3220 EXAM #4 NCLEX STYLE QUESTIONS

अब Quizwiz के साथ अपने होमवर्क और परीक्षाओं को एस करें!

A diagnosis of Hodgkin's disease is suspected in a 12-year-old child. Several diagnostic studies are performed to determine the presence of this disease. Which diagnostic test result will confirm the diagnosis of Hodgkin's disease? 1.Elevated vanillylmandelic acid urinary levels 2.The presence of blast cells in the bone marrow 3.The presence of Epstein-Barr virus in the blood 4.The presence of Reed-Sternberg cells in the lymph nodes

Correct Answer: 4 Rationale: Hodgkin's disease (a type of lymphoma) is a malignancy of the lymph nodes. The presence of giant, multinucleated cells (Reed-Sternberg cells) is the classic characteristic of this disease. Elevated levels of vanillylmandelic acid in the urine may be found in children with neuroblastoma. The presence of blast cells in the bone marrow indicates leukemia. Epstein-Barr virus is associated with infectious mononucleosis.

Which immunization should not be given to a child receiving chemotherapy for cancer? a. Tetanus vaccine c. Diphtheria, pertussis, tetanus (DPT) b. Inactivated poliovirus vaccine d. Measles, rubella, mumps

D

An example of a disease process that is based on a "two-hit" hypothesis leading to a cancer diagnosis is: A. Fanconi anemia. B. Wiskott Aldrich syndrome. C. Klinefelter syndrome. D. Retinoblastoma.

D.

Which characteristics would the nurse associate with aplastic anemia? Select all that apply. One, some, or all responses may be correct. 1) Decreased B12 2) Increased pain 3) Increased platelets 4) Decreased red blood cells 5) Decreased white blood cells

Decreased red blood cells Decreased white blood cells

d

For children who do not have a matched sibling bone marrow donor, the therapeutic management of aplastic anemia includes what intervention? a. Antibiotics b. Antiretroviral drugs c. Iron supplementation d. Immunosuppressive therapy

The nurse understands that the patient with megaloblastic (pernicious) anemia has which deficiencies? (Select all that apply. One, some, or all responses may be correct.) 1) Erythropoietin 2) Intrinsic factor 3) Ferrous sulfate 4) Cyanocobalamin 5) White blood cells

Intrinsic factor Cyanocobalamin

c

Persons diagnosed with sickle cell trait a. have 50% or more of the total hemoglobin in HgbS b. cannot pass the trait to their children c. can have painful gross hematuria as a major complication d. never develop symptoms of anemia

In taking care of a pediatric oncology patient, which diagnostic finding would indicate a critical concern for the development of bleeding? A. Absolute neutrophil count of 1000 mm3 B. Temperature of 99.2° F C. White blood cell count 18,000 mm3 D. Platelet count 50,000 mm3

Platelet count 50,000 mm3 The platelet count indicates a critical concern relative to coagulation and the patient is at risk for bleeding episodes. An absolute neutrophil count of 1000 mm3 is within normal range. Although the recorded temperature is elevated, it does not indicate a critical concern relative to infection as it can be due to other hypermetabolic processes in the body. A white count of this value suggests an infectious process.

Which disorder would the nurse associate with a patient who presents to the clinic with a reddish complexion and complains of dizziness, headache, and fatigue? 1) Leukemia 2) Anaphylaxis 3) Polycythemia vera 4) Thrombocytopenia

Polycythemia vera

Which clinical manifestations would the nurse anticipate when providing care for a patient with thrombocytopenia? (Select all that apply. One, some, or all responses may be correct.) 1) Purpura 2) Stomatitis 3) Ecchymoses 4) Defective S hemoglobin 5) Gastrointestinal bleeding

Purpura Ecchymoses Gastrointestinal bleeding

d

The clinic nurse instructs parents of a child with sickel cell anemia about the precipitating factors related to sickle cell crisis. Which, if identified by the parents as a precipitating factor, indicates the need for further instruction? a. stress b. trauma c. infection d. fluid overload

a c d

The clinic nurse is evaluating causes for iron deficiency caused by inadequate supply of iron. What should the nurse recognize as causes for iron deficiency caused by an inadequate iron supply? Select all that apply a. Prematurity b. Slow growth rate c. Excessive milk intake d. Severe iron deficiency in the mother e. Exclusive breastfeeding of infant from birth to 3 months

b c e

The clinic nurse is evaluating causes for iron deficiency due to impaired iron absorption. What should the nurse recognize as causes for iron deficiency due to impaired iron absorption? Select all that apply a. Gastric acidity b. Chronic diarrhea c. Lactose intolerance d. Absence of phosphates e. Inflammatory bowel disease

c

The clinical manifestations of sickle cell anemia (SCA) are primarily the result of which physiologic alteration? a. Decreased blood viscosity b. Deficiency in coagulation c. Increased red blood cell (RBC) destruction d. Greater affinity for oxygen

d

The nurse analyzes the laboratory results of a child with hemophilia. The nurse understands that which result will most likely be abnormal in this child? a. platelet count b. hematocrit level c. hemoglobin level d. partial thromboplastin time

a b c e

The nurse is administering a unit of blood to a child. What are signs and symptoms of a transfusion reaction? Select all that apply a. Chills b. Shaking c. Flank pain d. Hypothermia e. Sudden severe headache

b

The nurse is caring for a child with sickle cell disease who is scheduled to have a splenectomy. What information should the nurse explain to the parents regarding the reason for a splenectomy a. to decrease potential for infection b. to prevent splenic sequestration c. to prevent sickling of red blood cells d. to prevent sickle cell crisis

a

The nurse is planning care for an adolescent with acquired immunodeficiency syndrome. What is the priority nursing goal? a. Prevent infection. b. Prevent secondary cancers. c. Identify source of infection. d. Restore immunologic defenses.

d

The parent of a child receiving an iron preparation tells the nurse that the child's stools are a tarry green color. What information does the nurse included in discussion with this parent? a. Symptom of iron-deficiency anemia b. Adverse effect of the iron preparation c. Indicator of an iron preparation overdose d. Normally expected change resulting from the iron preparation

When participating in a patient care planning conference regarding an autologous bone marrow transplantation, the nurse understands that the plan of care includes preparing the patient for a transplant from which source? 1) An animal 2) The patient 3) Another person 4) A twin or sibling

The patient

c

The school nurse is caring for a child with hemophilia who fell on his arm during recess. What supportive measures would the nurse implement first? a. Apply warm, moist compresses. b. Apply a tourniquet for at least 5 minutes. c. Elevate the arm above the level of the heart. d. Begin passive range of motion unless pain is severe.

Which unique characteristics would the nurse link to a patient with sickle cell disease? (Select all that apply. One, some, or all responses may be correct.) 1) There is no cure for this disease. 2) Extreme pain occurs during a crisis. 3) Sickle cell patients have abnormally shaped red blood cells. 4) Adequate hydration can cause a crisis. 5) Leukapheresis is a common treatment.

There is no cure for this disease. Extreme pain occurs during a crisis. Sickle cell patients have abnormally shaped red blood cells.

b

What condition occurs when the normal adult hemoglobin is partly or completely replaced by abnormal hemoglobin? a. Aplastic anemia b. Sickle cell anemia c. Thalassemia major d. Iron deficiency anemia

a

What rationale explains why prolonged use of oxygen should be discouraged in a child with anemia? a. Prolonged use of oxygen can decrease erythropoiesis. b. Prolonged use of oxygen can interfere with iron production. c. Prolonged use of oxygen interferes with a childs appetite. d. Prolonged use of oxygen can affect the synthesis of hemoglobin.

a

Which of the following is the most frequent form of internal bleeding in the child with hemophilia? a. hemarthrosis b. epistaxis c. intracranial hemorrhage d. gastrointestinal tract hemorrhage

d

Which of the following measures should the nurse teach the parent of a child with hemophilia to do first if the child sustains an injury to a joint causing bleeding? a. give the child a dose of acetaminophen b. immobilize the joint and elevate the extremity c. apply heat to the area d. administer factor per the home-care protocol

The nurse is teaching parents about the importance of iron in a toddler's diet. Which explains why iron deficiency anemia is common during toddler-hood? a. Milk is a poor source of iron. b. Iron cannot be stored during fetal development. c. Fetal iron stores are depleted by age 1 month. d. Dietary iron cannot be started until age 12 months.

a. Milk is a poor source of iron. Children between the ages of 12 and 36 months are at risk for anemia because cow's milk is a major component of their diet and it is a poor source of iron. Iron is stored during fetal development, but the amount stored depends on maternal iron stores. Fetal iron stores are usually depleted by age 5 to 6 months. Dietary iron can be introduced by breastfeeding, iron-fortified formula, and cereals during the first 12 months of life.

Parents of a child with sickle cell anemia ask the nurse, "What happens to the hemoglobin in sickle cell anemia?" Which statement by the nurse explains the disease process?" a. Normal adult hemoglobin is replaced by abnormal hemoglobin. b. There is a lack of cellular hemoglobin being produced. c. There is a deficiency in the production of globulin chains. d. The size and depth of the hemoglobin are affected.

a. Normal adult hemoglobin is replaced by abnormal hemoglobin. Sickle cell anemia is one of a group of diseases collectively called hemoglobinopathies, in which normal adult hemoglobin is replaced by abnormal hemoglobin. Aplastic anemia is a lack of cellular elements being produced. Thalassemia major refers to a variety of inherited disorders characterized by deficiencies in production of certain globulin chains. Iron deficiency anemia affects the size and depth of the color.

The nurse is administering an IV chemotherapeutic agent to a child with leukemia. The child suddenly begins to wheeze and have severe urticaria. Which is the most appropriate nursing action? a. Stop drug infusion immediately. b. Recheck rate of drug infusion. c. Observe child closely for next 10 minutes. d. Explain to child that this is an expected side effect.

a. Stop drug infusion immediately.

The family of a child receiving chemotherapy for leukemia should be taught to focus on which aspect(s) of the child's care? (Select all that apply.) a. Using a support group b. Stimulating appetite c. Maintaining adequate hydration d. Continuing with scheduled immunizations e. Reporting exposure to infectious diseases

a. Using a support group b. Stimulating appetite c. Maintaining adequate hydration e. Reporting exposure to infectious diseases

A child is diagnosed with iron deficiency anemia. What will the nurse explain can occur if this disorder goes untreated? a. Hemorrhage b. Heart failure c. Infection d. Pulmonary embolism

b. Heart failure

Which is often administered to prevent or control hemorrhage in a child with cancer? a. Nitrosoureas b. Platelets c. Whole blood d. Corticosteroids

b. Platelets Most bleeding episodes can be prevented or controlled with the administration of platelet concentrate or platelet-rich plasma. Nitrosoureas, whole blood, and corticosteroids would not prevent or control hemorrhage.

Which immunization should be given with caution to children infected with human immunodeficiency virus (HIV)? a. Influenza b. Varicella c. Pneumococcal d. Inactivated poliovirus (IPV)

b. Varicella The children should be carefully evaluated before being given live viral vaccines such as varicella, measles, mumps, and rubella. The child must be immunocompetent and not have contact with other severely immunocompromised individuals. Influenza, pneumococcal, and inactivated poliovirus (IPV) are not live vaccines.

What will the nurse administer with ferrous sulfate drops when providing them to a child on the pediatric unit? a. With milk b. With orange juice c. With water d. On a full stomach

b. With orange juice

The nurse understands a primary step toward achievement of a long range goal associated with the rehabilitation of a client with a new colostomy is: "A. Mastery of techniques of colostomy care B. Readiness to accept an altered body function C. Awareness of available community resources D.Knowledge of the neccessary dietary modifications.

"ANSWER: B The client must be ready to accept changes in body image and function; this acceptance will facilitate mastery of the techniques of colosotomy care and optimal use of community resources."

The nurse is completing a care plan for a client diagnosed with leukemia. Which independent problem should be addressed? "1.Infection. 2.Anemia. 3.Nutrition. 4.Grieving."

"Correct: 4. Grieving is an independent problem, and the nurse can assess and treat this problem with or without collaboration."

Which of the following can be a manifestation of leukemia in a child? Select all that apply. 1. Leg pain. 2. Fever. 3. Excessive weight gain. 4. Bruising. 5. Enlarged lymph nodes.

1, 2, 4, 5 1. The proliferation of cells in the bone marrow can cause leg pain. 2. Fever is a result of the neutropenia. 4. A decrease in platelets causes the bruising. 5. The lymph nodes are enlarged by the infi ltration of leukemic cells.

Which of the following confi rms a diagnosis of Hodgkin disease in a 15-year-old? 1. Reed-Sternberg cells in the lymph nodes. 2. Blast cells in the blood. 3. Lymphocytes in the bone marrow. 4. VMA in the urine.

1. A lymph node biopsy is done to confi rm a histological diagnosis and staging of Hodgkin disease. The presence of Reed- Sternberg cells is characteristic of the disease.

A nurse educator is providing a teaching session for the nursing staff. Which of the following individuals is at greatest risk for developing beta-thalassemia (Cooley anemia)? 1. A child of Mediterranean descent. 2. A child of Mexican descent. 3. A child whose mother has chronic anemia. 4. A child who has a low intake of iron.

1. Beta-thalassemia is an inherited recessive disorder that is found primarily in individuals of Mediterranean descent. The disease has also been reported in Asian and African populations.

The parent of a 2-year-old who is HIV + questions the nurse about placing the child in day care. Which of the following is the best response? 1. The child should not go to day care until older, because there is a high risk for transmission of the disease. 2. The child can go to day care without restrictions and should be allowed to participate in all activities. 3. The child can go to day care but should avoid physical activity. 4. The child may go to day care, but the parent must inform all the parents at the day care that the child is HIV + .

2. The child can attend day care without any limitations but should not attend with a fever

When caring for a child with lymphoma, the nurse needs to be aware of which of the following? 1. The same staging system is used for lymphoma and Hodgkin disease. 2. Aggressive chemotherapy with central nervous system prophylaxis will give the child a good prognosis. 3. All children with lymphoma need a bone marrow transplant for a good prognosis. 4. Despite high-dose chemotherapy, the prognosis is very poor for most children.

2. The use of aggressive combination chemotherapy has a major impact on the survival rates for children with a diagnosis of lymphoma. Because there is usually bone marrow involvement, there is a need for central nervous system prophylaxis.

Which of the following laboratory tests will be ordered to determine the presence of the human immunodefi ciency virus antigen in an infant whose mother is HIV + ? 1. CD4 cell count. 2. Western blot. 3. IgG levels. 4. p24 antigen assay.

4. Detection of HIV in infants is confi rmed by a p24 antigen assay, viral culture of HIV, or polymerase chain reaction.

Which of the following is the most common opportunistic infection in children infected with human immunodefi ciency virus (HIV)? 1. CMV. 2. Encephalitis. 3. Meningitis. 4. Pneumocystic pneumonia.

4. Pneumocystis jiroveci pneumonia is the most common opportunistic infection that can occur in HIV-infected children, and such children are treated prophylactically for this.

16. In teaching family members about their child's von Willebrand disease, what is the priority outcome for the child that the nurse should discuss? a. Prevention of injury b. Maintaining adequate hydration c. Compliance with chronic transfusion therapy d. Prevention of respiratory infections

ANS: A A Hemorrhage as a result of injury is the child's greatest threat to life. B Fluid volume status becomes a concern when hemorrhage has occurred. C The treatment of von Willebrand disease is desmopressin acetate (DDAVP), which is administered intranasally or intravenously. D Respiratory infections do not constitute a major threat to the child with von Willebrand disease.

Which nursing intervention should not be included in the postoperative plan of care for a child undergoing surgery for a brain tumor? a. Place the child in Trendelenburg position. b. Perform neurologic assessments. c. Assess dressings for drainage. d. Monitor temperature.

ANS: A Feedback A The child is never placed in the Trendelenburg position because it increases intracranial pressure and the risk of bleeding. B Increased intracranial pressure is a risk in the postoperative period. The nurse would assess the child's neurologic status frequently. C Hemorrhage is a risk in the postoperative period. The child's dressing would be inspected frequently for bleeding. D Temperature is monitored closely because the child is at risk for infection in the postoperative period.

Which clinical finding is an overt sign of retinoblastoma in children? a. Whitish reflex in the eye b. Lymphadenopathy c. Bone pain d. Change in gait

ANS: A Feedback A A whitish reflex in the eye, leukocoria, is a common finding of retinoblastoma. It is an overt sign of cancer in children. B Persistent lymphadenopathy is a manifestation of several forms of childhood cancer. It is a covert sign of cancer in children. C Bone pain is a covert symptom of cancer in children. D A change in gait may be a sign of a brain tumor. It is considered a covert sign of cancer in children.

Children with non-Hodgkin lymphoma are at risk for complications resulting from tumor lysis syndrome (TLS). The nurse should assess for a. Liver failure b. CNS deficit c. Kidney failure d. Respiratory distress

ANS: C Feedback A TLS is related to intracellular electrolytes overloading the kidney as a response to the rapid lysis of tumor cells. This does not affect the liver. B TLS does not affect the CNS. C In TLS, the tumor's intracellular contents are dumped into the child's extracellular fluid as the tumor cells are lysed in response to chemotherapy. Because of the large volume of these cells, their intracellular electrolytes overload the kidneys and, if not monitored, can cause kidney failure. D TLS does not affect the lungs and cause respiratory distress.

A child with a history of fever of unknown origin, excessive bruising, lymphadenopathy, anemia, and fatigue is exhibiting symptoms most suggestive of a. Ewing sarcoma b. Wilms' tumor c. Neuroblastoma d. Leukemia

ANS: D Feedback A Symptoms of Ewing sarcoma involve pain and soft tissue swelling around the affected bone. B Wilms' tumor usually manifests as an abdominal mass with abdominal pain and may include renal symptoms, such as hematuria, hypertension, and anemia. C Neuroblastoma manifests primarily as an abdominal, chest, bone, or joint mass. Symptoms are dependent on the extent and involvement of the tumor. D These symptoms reflect bone marrow failure and organ infiltration, which occur in leukemia.

Which laboratory results would the nurse expect when a patient with a history of extreme fatigue, spontaneous nosebleeds, and frequent infections also has shortness of breath, pulse of 104 beats/min, and a blood pressure of 90/60 mm Hg? 1) RBCs 4.5, WBCs 3500, and platelets 95,000 2) RBCs 3.7, WBCs 4000, and platelets 80,000 3) RBCs 3.8, WBCs 4500, and platelets 100,000 4) RBCs 4.6, WBCs 5000, and platelets 125,000

Aplastic anemia signs/symptoms RBCs 3.7, WBCs 4000, and platelets 80,000

Which type of blood transfusion would the patient receive when stating, "I've already donated my own blood in case I need a transfusion during my joint replacement surgery"? 1) Autologous 2) Leukapheresis 3) Cryoprecipitate 4) Fresh Frozen Plasma

Autologous

Which transfusion option would the nurse discuss with a patient who practices the Jehovah's Witness religion and the surgeon has a concern about potential blood loss during the planned cardiac surgery? 1) Platelet transfusion 2) Allogenic transfusion 3) Autologous transfusion 4) Cryoprecipitate transfusion

Autologous transfusion

A school-age child is admitted in vaso-occlusive sickle cell crisis. The childs care should include: a. Correction of acidosis. b. Adequate hydration and pain management. c. Pain management and administration of heparin. d. Adequate oxygenation and replacement of factor VIII

B

A young boy will receive a bone marrow transplant (BMT). This is possible because one of his older siblings is a histocompatible donor. This type of BMT is termed: Test Bank - Maternal Child Nursing Care by Perry (6th Edition, 2017) 721 a. Syngeneic. c. Monoclonal. b. Allogeneic. d. Autologous.

B

Chelation therapy is begun on a child with b-thalassemia major. The purpose of this therapy is to: a. Treat the disease. c. Decrease the risk of hypoxia. b. Eliminate excess iron. d. Manage nausea and vomiting.

B

Which immunization should be given with caution to children infected with human immunodeficiency virus? a. Influenza c. Pneumococcus b. Varicella d. Inactivated poliovirus

B

The nurse is planning care for a school-age child admitted to the hospital with hemophilia. Which interventions should the nurse plan to implement for this child (Select all that apply)? a. Fingersticks for blood work instead of venipunctures b. Avoidance of intramuscular (IM) injections c. Acetaminophen (Tylenol) for mild pain control d. Soft toothbrush for dental hygiene e. Administration of packed red blood cells

B C D

A child is receiving fluorouracil as part of a chemotherapy protocol. Which of the following would be most important for the nurse to include in the child's plan of care? A) Monitoring for visual changes B) Maintaining adequate hydration C) Using prescribed eye drops to prevent conjunctivitis D) Avoiding administration with food or meals

B) Maintaining adequate hydration

A child with iron-deficiency anemia is prescribed ferrous fumarate, 3 mg/kg/day in two divided doses. The nurse interprets this order as indicating which of the following? A) The child requires a prophylactic dose of iron. B)The child has mild to moderate iron deficiency. C) The child has severe iron deficiency. D) The child is being prepared for packed red blood cell administration.

B)The child has mild to moderate iron deficiency.

A child is brought to the emergency department after being accidentally struck in the lower back region with a baseball bat. When gathering assessment data, the nurse discovers that the child has hemophilia. The nurse should immediately assess for which data? 1. Slurred speech 2. Presence of hematuria 3. Complaints of headache 4. Change in respiratory rate

Correct Answer: 2 Rationale: Because the kidneys are located in the flank region of the body, trauma to the back area can cause hematuria, particularly in a child with hemophilia. The nurse would be most concerned about the child's airway and respiratory rate if the child had sustained an injury to the neck region. Slurred speech and headache are associated with head trauma.

Which teaching point would the nurse identify in the plan of care for a patient recently diagnosed with thrombocytopenia? 1) Instruct the patient to avoid crowds. 2) Instruct the patient to increase fluid intake. 3) Instruct the patient to use a soft toothbrush. 4) Instruct the patient to alternate periods of activity with periods of rest.

Instruct the patient to use a soft toothbrush.

d

The nurse suspects a child is having an adverse reaction to a blood transfusion. What is the initial action by the nurse? a. Notify the physician. b. Take the vital signs and blood pressure and compare them with baseline levels. c. Dilute infusing blood with equal amounts of normal saline. d. Stop transfusion and maintain a patent intravenous line with normal saline and new tubing.

What assessment finding would the nurse expect to find specifically for a client admitted with Hodgkin's disease? "1. Fatigue 2. weakness. 3. Weight gain 4. Enlarged lymph nodes."

Correct: 4. Enlarged lymph nodes with progression to extralymphatic sites. This is a characteristic specifically to lymphoma, where as fatigue and weakness can occur with other diseases. Weight loss is more likely than weight gain.

A pediatric oncology patient has developed a nose bleed. Which finding would account for this occurrence? A. Increased white blood cell count B. Increased neutrophils C. Decreased hemoglobin and hematocrit D. Decreased platelet count

D

Several complications can occur when a child receives a blood transfusion. An immediate sign or symptom of an air embolus is: a. Chills and shaking. c. Irregular heart rate. b. Nausea and vomiting. d. Sudden difficulty in breathing

D

The nurse is preparing a child for possible alopecia from chemotherapy. Which suggestion should be included in the teaching? a. Explaining to the child that hair usually regrows in 1 year. b. Advising the child to expose the head to sunlight to minimize alopecia. c. Explaining to the child that wearing a hat or scarf is preferable to wearing a wig. d. Explaining to the child that, when hair regrows, it may have a slightly different color or texture.

D

The nurse is recommending how to prevent iron deficiency anemia in a healthy, term, breastfed infant. What should she or he suggest? a. Iron (ferrous sulfate) drops after age 1 month. b. Iron-fortified commercial formula can be used by ages 4 to 6 months. c. Iron-fortified infant cereal can be introduced at age 2 months. d. Iron-fortified infant cereal can be introduced at approximately 6 months of age.

D

A preschool-age child undergoing chemotherapy experiences nausea and vomiting. Which of the following would be the best intervention to include in the child's plan of care? a. Administer tube feedings. b. Offer small, frequent meals. c. Offer fluids only between meals. d. Allow the child to choose what to eat for meals.

D (While all options can be done to encourage nutrition, allowing the preschooler choices meets two issues: nutrition and developmental tasks.)

In taking care of a pediatric oncology patient, which diagnostic finding would indicate a critical concern for the development of bleeding? A. Absolute neutrophil count of 1000 mm3 B. Temperature of 99.2° F C. White blood cell count 18,000 mm3 D. Platelet count 50,000 mm3

D.

The primary care pediatric nurse practitioner reviews a child's complete blood count with differential white blood cell values and recognizes a "left shift" because of A. a decreased eosinophil count. B. a decreased lymphocyte count. C. an elevated monocyte count. D. an elevated neutrophil count.

D. an elevated neutrophil count.

Which disorder would the nurse identify as a grave complication when the patient experiences bleeding and intravascular clotting at the same time? 1) Thrombocytopenia 2) Multiple myeloma 3) Von Willebrand disease 4) Disseminated intravascular coagulation (DIC)

Disseminated intravascular coagulation (DIC)

Which clinical manifestation would the nurse monitor for potential development in a patient who is at risk for circulatory overload after receiving a blood transfusion 2 hours ago? 1) Dyspnea 2) Back pain 3) Bradycardia 4) Increased temperature

Dyspnea

Which focus of care would the nurse evaluate when providing care for a patient with pernicious anemia? 1) Maintenance of monthly phlebotomy appointments 2) Ensuring administration of monthly B12 injections for life 3) Obtaining positive Coombs test results on a scheduled basis 4) Interventions related to an increased risk for aplastic anemia

Ensuring administration of monthly B12 injections for life

c

Gary Berringer is a 5 year old being admitted because of diminished RBC production triggered by a viral infection. What type of sickle cell crisis is he most likely experiencing? a. vasoocclusive crisis b. splenic sequestration crisis c. aplastic crisis d. hyperhemolytic crisis

b

Hereditary spherocytosis is a. always transmitted as an autosomal recessive disease b. a hemolytic disorder caused by a defect in the proteins that form the RBC membrane c. rarely evident until the infant is 4-6 months of age d. usually resolved when additional folic acid supplements are administered

When a patient with sickle cell anemia complains of experiencing a crisis at least once a week and wants to reduce the frequency, which treatment would the nurse anticipate the primary health care provider prescribing? 1) Oxygen therapy 2) Hydroxyurea 3) Erythropoietin 4) Intravenous (IV) morphine

Hydroxyurea

After consulting with the patient's dietician regarding an oral iron supplement, which liquid would the nurse request to increase absorption of the medication? 1) Tea 2) Milk 3) Coffee 4) Orange juice

Orange juice

Which clinical manifestation would the nurse report immediately when providing care for a patient with severe anemia who is receiving a blood transfusion? 1) Euphoria 2) Pain in the lower back 3) Temperature of 100°F 4) Decreased urine output

Pain in the lower back

Which clinical manifestations would the nurse associate with severe anemia? (Select all that apply. One, some, or all responses may be correct.) 1) Pallor 2) Tachypnea 3) Bradycardia 4) Smooth nails 5) Smooth tongue

Pallor Tachypnea Smooth tongue

Which condition would the nurse include in the teaching for a patient diagnosed with an excessive production of red blood cells? 1) Leukemia 2) Leukocytosis 3) Thrombocytopenia 4) Polycythemia vera

Polycythemia vera

Which intervention would the nurse implement when the patient receiving a blood transfusion complains of fever, chills, lower back pain, and chest tightness? 1) Stopping the blood transfusion process 2) Maintaining airways through suctioning 3) Notifying the primary health care provider 4) Positioning the patient in the supine position

Stopping the blood transfusion process

a c d

The nurse is caring for a child who is receiving a transfusion of PRBCs. The nurse is aware that if the child has a hemolytic reaction to the blood, the signs and symptoms would include which of the following? Select all that apply a. fever b. rash c. oliguria d. hypotension e. chills

d

The nurse is caring for a child with hemophilia A. The childs activity is as tolerated. What activity is contraindicated for this child? a. Ambulating to the cafeteria b. Active range of motion c. Ambulating to the playroom d. Passive range of motion exercises

d

The nurse is explaining blood components to an 8-year-old child. What is the nurse's best description and action of platelets? a. Make up the liquid portion of blood. b. Help keep germs from causing infection. c. Carry the oxygen you breathe from your lungs to all parts of your body. d. Help your body stop bleeding by forming a clot (scab) over the hurt area.

a d e

What activity should the school nurse recommend for a child with hemophilia A? Select all that apply a. Golf b. Soccer c. Rugby d. Jogging e. Swimming

a b e

What are signs and symptoms of anemia? Select all that apply a. Pallor b. Fatigue c. Dilute urine d. Bradycardia e. Muscle weakness

d

What is the most appropriate method to use when drawing blood from a child with hemophilia? a. use finger punctures for lab draws b. prepare to administer platelets c. apply heat to the extremity before venipunctures d. schedule all labs to be drawn at one time

c

Which is the most accurate genetic explanation for a family with hemophilia? a. it is a y-linked dominant disorder b. it is equally distributed among males and females c. it is an x-linked recessive disorder d. it is an autosommal recessive disorder

The nurse is teaching parents about the importance of iron in a toddler's diet. Which explains why iron deficiency anemia is common during toddlerhood? a. Milk is a poor source of iron. b. Iron cannot be stored during fetal development. c. Fetal iron stores are depleted by age 1 month. d. Dietary iron cannot be started until age 12 months.

a. Milk is a poor source of iron.

The most recent blood count for a child who received chemotherapy last week shows neutropenia. What is the priority nursing diagnosis for this child? a. Risk for infection b. Risk for hemorrhage c. Altered skin integrity d. Disturbance in body image

a. Risk for infection

A possible cause of acquired aplastic anemia in children is: a. drugs. b. injury. c. deficient diet. d. congenital defect.

a. drugs. Drugs, such as chemotherapeutic agents and several antibiotics (e.g., chloramphenicol), can cause aplastic anemia. Injury, deficient diet, and congenital defect are not causative agents in acquired aplastic anemia.

What would the nurse include in a teaching plan about mouth care of a child receiving chemotherapy? a. Use commercial mouthwash. b. Clean teeth with a soft toothbrush. c. Avoid use of a Water-Pik. d. Inspect the mouth weekly for ulcerations.

b. Clean teeth with a soft toothbrush.

The nurse is teaching the parents of a young child with iron deficiency anemia about nutrition. What food would the nurse emphasize as being a rich source of iron? a. An egg white b. Cream of Wheat c. A banana d. A carrot

b. Cream of Wheat

Which should the nurse teach about prevention of sickle cell crises to parents of a preschool child with sickle cell disease? (Select all that apply.) a. Limit fluids at bedtime. b. Notify the health care provider if a fever of 38.5° C (101.3° F) or greater occurs. c. Give penicillin as prescribed. d. Use ice packs to decrease the discomfort of vasoocclusive pain in the legs. e. Notify the health care provider if your child begins to develop symptoms of a cold.

b. Notify the health care provider if a fever of 38.5° C (101.3° F) or greater occurs. c. Give penicillin as prescribed. e. Notify the health care provider if your child begins to develop symptoms of a cold.

Which is often administered to prevent or control hemorrhage in a child with cancer? a. Nitrosoureas b. Platelets c. Whole blood d. Corticosteroids

b. Platelets

"A child is admitted to the hospital with a diagnosis of Wilm's tumor, Stage II. Which of the following statements most accurately describes this stage? "A) The tumor is less than 3 cm. in size and requires no chemotherapy. B) The tumor did not extend beyond the kidney and was completely resected. C) The tumor extended beyond the kidney but was completely resected. D) The tumor has spread into the abdominal cavity and cannot be resected."

"1. Answer: C The staging of Wilm's tumor is confirmed at surgery as follows: Stage I, the tumor is limited to the kidney and completely resected; stage II, the tumor extends beyond the kidney but is completely resected; stage III, residual nonhematogenous tumor is confined to the abdomen; stage IV, hematogenous metastasis has occurred with spread beyond the abdomen; and stage V, bilateral renal involvement is present at diagnosis."

Which should the nurse include when teaching the mother of a 9-month-old infant about administering liquid iron preparations? a. They should be given with meals. b. They should be stopped immediately if nausea and vomiting occur. c. Adequate dosage will turn the stools a tarry green color. d. Allow preparation to mix with saliva and bathe the teeth before swallowing.

c. Adequate dosage will turn the stools a tarry green color.

A school-age child with leukemia experienced severe nausea and vomiting when receiving chemotherapy for the first time. Which is the most appropriate nursing action to prevent or minimize these reactions with subsequent treatments? a. Encourage drinking large amounts of favorite fluids. b. Encourage child to take nothing by mouth (remain NPO) until nausea and vomiting subside. c. Administer an antiemetic before chemotherapy begins. d. Administer an antiemetic as soon as child has nausea.

c. Administer an antiemetic before chemotherapy begins.

A toddler with leukemia is on intravenous chemotherapy treatments. The toddlers lab results are white blood cell count (WBC): 1000; neutrophils: 7%; nonsegmented neutrophils (bands): 7%. What is this childs absolute neutrophil count (ANC)? _____ Record your answer as a whole number.

140

The parent of a child diagnosed with Wilms tumor asks the nurse what the treatment plan will be. The nurse explains the usual protocol for this condition. Which information should the nurse give to the parent? 1. The child will have chemotherapy and, after that has been completed, radiation. 2. The child will need to have surgery to remove the tumor. 3. The child will go to surgery for removal of the tumor and the kidney and will then start chemotherapy. 4. The child will need radiation and later surgery to remove the tumor.

3. Combination therapy of surgery and chemotherapy is the primary therapeutic management. Radiation is done depending on clinical stage and histological pattern.

Which of the following should be done to protect the central nervous system from the invasion of malignant cells in a child newly diagnosed with leukemia? 1. Cranial and spinal radiation. 2. Intravenous steroid therapy. 3. Intrathecal chemotherapy. 4. High-dose intravenous chemotherapy.

3. Giving chemotherapy via lumbar puncture allows the drugs to get to the brain and helps prevent metastasis of the disease.

The nurse is instructing the parent of a child with HIV about immunizations. Which of the following should the nurse tell the parent? Select all that apply. 1. Hepatitis B vaccine will not be given to this child. 2. Members of the family should be cautioned not to receive the varicella vaccine. 3. The child will need to have a Western blot test done prior to all immunizations. 4. Pneumococcal and infl uenza vaccines are recommended. 5. Meningitis immunization.

4, 5 4. Immunizations against childhood illnesses are recommended for children exposed to or infected with HIV. Pneumococcal and infl uenza vaccines are recommended at their scheduled times 5. Meningococcal immunizations can be administered on the regular schedule.

Which intervention should be implemented after a bone marrow aspiration? 1. Ask the child to remain in a supine position. 2. Place the child in an upright position for 4 hours. 3. Keep the child nothing by mouth for 6 hours. 4. Administer analgesics as needed for pain.

4. Children may experience minor discomfort after the procedure, and analgesics should be given as needed

Prednisone is given to children who are being treated for leukemia. Why is this medication given as part of the treatment plan? 1. Enhances protein metabolism. 2. Enhances sodium excretion. 3. Increases absorption of the chemotherapy. 4. Destroys abnormal lymphocytes.

4. Prednisone is used in many of the treatment protocols for leukemia because there is abnormal lymphocyte production. Prednisone is thought to destroy abnormal lymphocytes.

A possible cause of acquired aplastic anemia in children is: a. Drugs. c. Deficient diet. b. Injury. d. Congenital defect.

A

A young child with leukemia has anorexia and severe stomatitis. The nurse should suggest that the parents try which intervention? a. Relax any eating pressures. b. Firmly insist that child eat normally. c. Begin gavage feedings to supplement diet. d. Serve foods that are either hot or cold.

A

An inherited immunodeficiency disorder characterized by absence of both humoral and cell-mediated immunity is: a. Severe combined immunodeficiency syndrome (SCIDS). b. Acquired immunodeficiency syndrome. c. Wiskott-Aldrich syndrome. d. Fanconi syndrome.

A

In which condition are all the formed elements of the blood simultaneously depressed? Test Bank - Maternal Child Nursing Care by Perry (6th Edition, 2017) 718 a. Aplastic anemia c. Thalassemia major b. Sickle cell anemia d. Iron deficiency anemia

A

What is the most common mode of transmission of human immunodeficiency virus (HIV) in the pediatric population? a. Perinatal transmission c. Blood transfusions b. Sexual abuse d. Poor hand washing

A

A child with lymphoma is receiving extensive radiotherapy. Which of the following is the most common side effect of this treatment? A. Malaise B. Seizures C. Neuropathy D. Lymphadenopathy

A (Malaise is the most common side effect of radiotherapy. For children, the fatigue may be especially distressing because it means they cannot keep up with their peers.)

b

A nurse instructs the parent of a child with sickle cell disease about factors that might precipitate a pain crisis in the child. Which of the following factors identified by the parent as being able to cause a pain crisis indicates a need for further teaching? a. infection b. overhydration c. stress at school d. cold environment

a c

A nurse is providing teaching about epistaxis to the parent of a school-age child. Which of the following should the nurse include as an action to take when managing an episode of epistaxis? Select all that apply a. press the nares together for at least 10 minutes b. breathe through the nose until bleeding stops c. pack cotton or tissue into the naris that is bleeding d. apply a warm cloth across the bridge of the nose e. insert petroleum into the naris after the bleeding stops

d

A nurse is providing teaching to the parent of a child who has a new prescription for liquid oral iron supplements. Which of the following statements by the parent indicates an understanding of teaching? a. "I should take my child to the ER if his stools become dark" b. "my child should avoid eating citrus fruits while taking the supplements" c. "I should give the iron with milk to help prevent an upset stomach" d. "my child should take the supplement through a straw"

Which clinical manifestation would the nurse associate with a patient who has polycythemia vera? 1) A patient with pallor and activity intolerance 2) A thin patient with a pale complexion 3) A patient with hypotension and syncope 4) A patient with a reddish face appearance

A patient with a reddish face appearance

A 14-year-old boy is diagnosed with Hodgkin disease. When palpating for enlarged lymph nodes, the nurse would expect to find which nodes as most commonly enlarged? Select all answers that apply. A) Cervical B) Axillary C) Supraclavicular D) Occipital E) Inguinal

A) Cervical C) Supraclavicular

The nurse is teaching the parents of a child diagnosed with iron-deficiency anemia about ways to increase their child's intake of iron. The parents demonstrate understanding of the teaching when they identify which foods as good choices for the child? Select all that apply. A) Tuna B) Salmon C) Tofu D) Cow's milk E) Dried fruits

A) Tuna B) Salmon C) Tofu E) Dried fruits

A toddler who presents with anemia and reticulocytopenia has a history of a gradual decrease in energy and increase in pallor beginning after a recent viral infection. How will the primary care pediatric nurse practitioner treat this child? A. Closely observe the child's symptoms and lab values. B. Consult with a pediatric hematologist. C. Prescribe supplemental iron for 4 to 6 months. D. Refer for transfusions to correct the anemia.

A. Closely observe the child's symptoms and lab values.

The primary care pediatric nurse practitioner reviews hematology reports on a child with betathalassemia minor and notes an Hgb level of 8 g/dL. What will the nurse practitioner do? A. Evaluate serum ferritin. B. Order Hgb electrophoresis. C. Prescribe supplemental iron. D. Refer for RBC transfusions.

A. Evaluate serum ferritin.

Which statement best describes -thalassemia major (Cooley anemia)? a. All formed elements of the blood are depressed. b. Inadequate numbers of red blood cells are present. c. Increased incidence occurs in families of Mediterranean extraction. d. Increased incidence occurs in persons of West African descent.

c. Increased incidence occurs in families of Mediterranean extraction. Individuals who live near the Mediterranean Sea and their descendants have the highest incidence of thalassemia. An overproduction of red cells occurs. Although numerous, the red cells are relatively unstable. Sickle cell disease is common in persons of West African descent.

A child diagnosed with lymphoma is receiving extensive radiation therapy. The MOST common side effect of this treatment is: A. fatigue. B. seizures. C. neuropathy. D. lymphadenopathy.

A. fatigue. Fatigue is the most common side effect of radiation therapy. For children the fatigue may be especially distressing because it means that they cannot keep up with their peers. Seizures are unlikely because irradiation would not usually involve the cranial area for treatment of lymphoma. Neuropathy is a side effect of certain chemotherapeutic agents. Lymphadenopathy is one of the findings of lymphoma.

When an adolescent with a new diagnosis of Ewing sarcoma asks the nurse about treatment, the nurse's response is based on the knowledge that (select all that apply) a. This type of tumor invades the bone. b. Management includes chemotherapy, surgery, and radiation. c. Ewing sarcoma is usually not responsive to either chemotherapy or radiation. d. Affected bones such as ribs and proximal fibula may be removed to excise the tumor. e. Is the most common bone tumor seen in children.

ANS: A, B, D Feedback Correct: Ewing sarcoma invades the bone and is found most often in the midshaft of long bones, especially the femur, vertebrae, ribs, and pelvic bones. Treatment for Ewing sarcoma begins with chemotherapy to decrease tumor bulk, followed by surgical resection of the primary tumor. Local control of the tumor can be achieved with surgery or radiation. The affected bone may be removed if it will not affect the child's functioning. Ribs and the proximal fibula are considered expendable and may be removed to excise the tumor without affecting function. Incorrect: Ewing sarcoma is responsive to both chemotherapy and radiation. Osteosarcoma is the most common primary bone malignancy in children. The second most common bone tumor seen in children is Ewing sarcoma.

6. A condition in which the normal adult hemoglobin is partly or completely replaced by abnormal hemoglobin is known as a. Aplastic anemia b. Sickle cell anemia c. Thalassemia major d. Iron-deficiency anemia

ANS: B A Aplastic anemia is a lack of cellular elements being produced. B Sickle cell anemia is one of a group of diseases collectively called hemoglobinopathies, in which normal adult hemoglobin is replaced by an abnormal hemoglobin. C Hemophilia refers to a group of bleeding disorders in which there is deficiency of one of the factors necessary for coagulation. D Iron-deficiency anemia affects size and depth of color and does not involve an abnormal hemoglobin.

d

An 8-year-old girl is receiving a blood transfusion when the nurse notes that she has developed precordial pain, dyspnea, distended neck veins, slight cyanosis, and a dry cough. These manifestations are most suggestive of what complication? a. Air embolism b. Allergic reaction c. Hemolytic reaction d. Circulatory overload

A boy with leukemia screams whenever he needs to be turned or moved. The most probable cause of this pain is: a. Edema. c. Petechial hemorrhages. Test Bank - Maternal Child Nursing Care by Perry (6th Edition, 2017) 720 b. Bone involvement. d. Changes within the muscles.

B

A condition in which the normal adult hemoglobin is partly or completely replaced by abnormal hemoglobin is: a. Aplastic anemia. c. Thalassemia major. b. Sickle cell anemia. d. Iron deficiency anemia.

B

The nurse has initiated a blood transfusion on a preschool child. The child begins to exhibit signs of a transfusion reaction. Place in order the interventions the nurse should implement, sequencing from the highest priority to the lowest. a. Take the vital signs. b. Stop the transfusion. c. Notify the practitioner. d. Maintain a patent intravenous (IV) line with normal saline

B A D C

An example of a disease process with underlying immune adaptation l potentially leading to a cancer diagnosis is? A. Fanconi anemia B. Wiskott Aldrich syndrome C. Klinefelter syndrome D. Retinoblastoma

B. Wiskott Aldrich syndrome is an example of an immunodeficiency state may place the individual at increased risk to develop certain cancers. Fanconi anemia and Klinefelter syndrome are examples of chromosomal abnormalities which can potentially lead to development of cancer. Retinoblastoma is an example of "two-hit" hypothesis of inheritance leading to development of cancer states.

The primary care pediatric nurse practitioner is performing a well child examination on a schoolage child who has a history of cancer treated with cranial irradiation. What will the nurse practitioner monitor in this child? A. Cardiomyopathy and arrhythmias B. Leukoencephalopathy C. Obesity and gonadal dysfunction D. Peripheral neuropathy and hearing loss

B. Leukoencephalopathy

An adolescent will receive a bone marrow transplant (BMT). The nurse should explain that the bone marrow will be administered by which route? a. Bone grafting c. Intravenous infusion b. Bone marrow injection d. Intraabdominal infusion

C

As related to inherited disorders, which statement is descriptive of most cases of hemophilia? a. Autosomal dominant disorder causing deficiency in a factor involved in the blood-clotting reaction b. X-linked recessive inherited disorder causing deficiency of platelets and prolonged bleeding c. X-linked recessive inherited disorder in which a blood-clotting factor is deficient d. Y-linked recessive inherited disorder in which the red blood cells become moon shaped

C

The nurse is caring for a 13-year-old girl with von Willebrand disease. After teaching the adolescent and her parents about this disorder and care, which response by the parents indicates a need for additional teaching? A) "We need to administer Stimate prior to dental work." B) "We should be aware that she may suffer from menorrhagia." C) "We should administer desmopressin as often as needed." D) "We understand that she may have frequent nosebleeds."

C) "We should administer desmopressin as often as needed."

A nurse is conducting a physical examination of a 5-year-old with suspected iron- deficiency anemia. How would the nurse evaluate for changes in neurologic functioning? A) "Open your mouth so I can look inside your cheeks and lips." B) "Do you have any bruises on your feet or shins?" C) "Will you show me how you walk across the room?" D) "Let me see the palms of your hands and soles of your feet."

C) "Will you show me how you walk across the room?"

The nurse is caring for a 9-year-old boy who is having chemotherapy. The nurse is developing a teaching plan for the child and family about nutrition. Which of the following would the nurse be least likely to include? A) Emphasizing the intake of grains, fruits, and vegetables B) Featuring high-fiber foods if opioid analgesics are being taken C) Concentrating on consuming primarily high-calorie shakes and puddings D) Avoiding milk products if diarrhea is a problem

C) Concentrating on consuming primarily high-calorie shakes and puddings

A pediatric oncology patient is undergoing chemotherapy. Which observation would lead the nurse to suspect that the patient has developed sterile hemorrhagic cystitis? A. Absence of hematuria B. Presence of proteinuria C. Complaints by the patient that it burns upon urination D. Increased sensation of thirst.

C. Complaints by the patient that it burns upon urination Clinical symptoms associated with sterile hemorrhagic cystitis include dysuria and hematuria. Presence of proteinuria is associated with an unrelated finding. Increased sensation of thirst is associated with an unrelated finding.

A child with lymphoma is receiving extensive radiotherapy. Which of the following is the most common side effect of this treatment? "A. Malaise B. Seizures C. Neuropathy D. Lymphadenopathy"

"Answer A is Correct Malaise is the most common side effect of radiotherapy. For children, the fatigue may be especially distressing because it means they cannot keep up with their peers."

"The pediatric nurse understands that the most common cancer found in children is: "1. Non-hodgkin's lymphoma 2. Acute lymphocytic leukemia 3. Chronic lymphocytic leukemia 4. Ewing's sarcoma"

"Correct: 2. 1. No - this is not a common cancer in children 2. YES! this is the most common form of cancer found in children is acute lymphocytic leukemia. 3. No - this is not a common cancer in children 4. No - this is not a common cancer in children"

"A nurse is performing an assessment on a 10-year old child suspected having Hodgkin's disease. The nurse understands that which of the following assessment findings is characteristic of this disease? "a) fever and malaise b) anorexia and weight loss c) painful, enlarged inguinal lymph nodes d) painless, firm, and movable adenopathy in the cervical area"

"D. painless, firm, and movable adenopathy in the cervical area - Clinical manifestations specifically associated with Hodgkin's disease include painless, firm, and movable adenopathy in the cervical and supraclavicular areas. Hepatosplenomegaly also is noted. Although fever, malaise, anorexia, and weight loss are associated with Hodgkin's disease, these manifestations are seen in many disorders."

Which of the following activities should a nurse suggest for a client diagnosed with hemophilia? Select all that apply. 1. Swimming. 2. Golf. 3. Hiking. 4. Fishing. 5. Soccer.

1, 2, 3, 4 1. Children with hemophilia should be encouraged to take part in noncontact activities that allow for social, psychological, and physical growth, such as swimming. 2. Children with hemophilia should be encouraged to take part in noncontact activities that allow for social, psychological, and physical growth, such as golf. 3. Children with hemophilia should be encouraged to take part in noncontact activities that allow for social, psychological, and physical growth, such as hiking. 4. Children with hemophilia should be encouraged to take part in noncontact activities that allow for social, psychological, and physical growth, such as fishing

The nurse is caring for a child who is receiving a transfusion of PRBCs. The nurse is aware that if the child has a hemolytic reaction to the blood, the signs and symptoms would include which of the following? Select all that apply. 1. Fever. 2. Rash. 3. Oliguria. 4. Hypotension. 5. Chills.

1, 3, 4. 1. Hemolytic reactions include fever, pain at insertion site, hypotension, renal failure, tachycardia, oliguria, and shock. 3. Hemolytic reactions include fever, pain at insertion site, hypotension, renal failure, tachycardia, oliguria, and shock. 4. Hemolytic reactions include fever, pain at insertion site, hypotension, renal failure, tachycardia, oliguria, and shock.

The nurse is caring for a child with a diagnosis of ALL who is receiving chemotherapy. The nurse notes that the child ' s platelet count is 20,000/mm 3 . Based on this laboratory fi nding, what information should the nurse provide to the child and parents? 1. A soft toothbrush should be used for mouth care. 2. Isolation precautions should be started immediately. 3. The child ' s vital signs, including blood pressure, should be monitored every 4 hours. 4. All visitors should be discouraged from coming to see the family.

1. Because the platelet count is decreased, there is a signifi cant risk of bleeding, especially in soft tissue. The use of the soft toothbrush should help prevent bleeding of the gums.

Which of the following is correct regarding prognostic factors for determining survival for a child newly diagnosed with ALL? 1. The initial white blood cell count on diagnosis. 2. The race of the child. 3. The amount of time needed to initiate treatment. 4. Children aged 12 to 15 years.

1. Children with a normal or low white blood cell count who do not have non-T, non-B acute lymphoblastic leukemia and who are CALLA-positive have a much better prognosis than those with high cell counts or other cell types.

The nurse is taking care of a child with sickle cell disease. The nurse is aware that which of the following problems is (are) associated with sickle cell disease? Select all that apply. 1. Polycythemia. 2. Hemarthrosis. 3. Aplastic crisis. 4. Thrombocytopenia. 5. Vaso-occlusive crisis.

3, 5 3. Aplastic crisis, temporary cessation of red blood cell production, is associated with sickle cell anemia. 5. Vaso-occlusive crisis is the most common problem in children with sickle cell disease.

A teen is seen in clinic for a possible diagnosis of Hodgkin disease. The nurse is aware that which of the following symptoms should make the health-care provider suspect Hodgkin disease? 1. Fever, fatigue, and pain in the joints. 2. Anorexia with weight loss. 3. Enlarged, painless, and movable lymph nodes in the cervical area. 4. Enlarged liver with jaundice.

3. Enlarged, painless, and movable lymph nodes in the cervical area are the most common presenting manifestations of Hodgkin disease

The most important reason health-care providers prescribe a combination of antiretroviral drugs to children with HIV is to delay: 1. Progression to AIDS. 2. Enable treatment of multiple symptoms. 3. Recurrence of symptoms. 4. Drug resistance

4. A combination of antiretroviral medications is prescribed for a child who is HIV positive to delay development of drug resistance. HIV drugs work on different stages of the HIV life cycle to prevent reproduction of new virus particles.

Which of the following measures should the nurse teach the parent of a child with hemophilia to do fi rst if the child sustains an injury to a joint causing bleeding? 1. Give the child a dose of acetaminophen (Tylenol). 2. Immobilize the joint and elevate the extremity. 3. Apply heat to the area. 4. Administer factor per the home-care protocol.

4. Administration of factor should be the fi rst intervention if home-care transfusions have been initiated.

A child with leukemia is receiving chemotherapy and is complaining of nausea. The nurse has been giving the scheduled antiemetic. Which of the following should the nurse do when the child is nauseated? 1. Encourage low-protein foods. 2. Encourage low-caloric foods. 3. Offer the child ' s favorite foods. 4. Offer cool, clear liquids.

4. Cool, clear liquids are better tolerated. Milk-based products cause secretions to be thick and can cause vomiting

A nurse is doing discharge education with a parent who has a child with betathalassemia (Cooley anemia). The nurse informs the parent that the child is at risk for which of the following conditions? 1. Hypertrophy of the thyroid. 2. Polycythemia vera. 3. Thrombocytopenia. 4. Chronic hypoxia and iron overload.

4. In beta-thalassemia, there is increased destruction of red blood cells, causing anemia. This results in chronic anemia and hypoxia. The children are treated with multiple blood transfusions, which can cause iron overload and damage to major organs.

The nurse expects which of the following clinical manifestations in a child diagnosed with SCID? 1. Prolonged bleeding. 2. Failure to thrive. 3. Fatigue and malaise. 4. Susceptibility to infection

4. SCID is characterized by an absence of cell-mediated immunity, with the most common clinical manifestation being infection in children from age 3 months. These children do not usually recover from these infections.

Which statement best explains why iron deficiency anemia is common during toddlerhood? a. Milk is a poor source of iron. b. Iron cannot be stored during fetal development. c. Fetal iron stores are depleted by age 1 month. d. Dietary iron cannot be started until age 12 months.

A

b

A nurse is caring for an infant whose screening test reveals a possible diagnosis of sickle cell disease. Which of the following tests should be performed to distinguish if the infant has the trait or the disease? a. sickle solubility test b. hemoglobin electrophoresis c. complete blood count d. transcranial doppler

a

A school-age child is admitted in vasoocclusive sickle cell crisis (pain episode). The childs care should include which therapeutic interventions? a. Hydration and pain management b. Oxygenation and factor VIII replacement c. Electrolyte replacement and administration of heparin d. Correction of alkalosis and reduction of energy expenditure

a

A toddler is diagnosed with chronic benign neutropenia. The parents are being taught about caring for their child. What information is important to include? a. Avoid large indoor crowds and people who are ill. b. Parenteral antibiotics are necessary to control disease. c. Frequent rest periods are needed during the daytime. d. List the side effects of corticosteroids used to decrease inflammation.

The mother of a 5-year-old girl brings the child to the clinic for an evaluation. The mother tells the nurse, "She seems to be so tired and irritable lately. And she looks so pale." Further assessment reveals pale conjunctiva and oral mucous membranes. The nurse suspects iron-deficiency anemia. Which additional finding would help provide additional evidence for this suspicion? A) Spooned nails B) Negative splenomegaly C) Oxygen saturation: 99% D) Bradycardia

A) Spooned nails

A less common malignancy of muscle or striated tissue is known as ______________.

ANS: rhabdomyosarcoma This sarcoma occurs periorbitally or in the head and neck of younger children and in the trunk and extremities of older children. Long-term survival rates are variable based upon the age of the child.

A toddler with leukemia is on intravenous chemotherapy treatments. The toddler's lab results are WBC: 1000; neutrophils: 7%; nonsegmented neutrophils (bands): 7%. What is this child's absolute neutrophil count (ANC)? (Record your answer in a whole number.)

ANS: 140 To calculate an ANC for a WBC = 1000; neutrophils = 7%; and nonsegmented neutrophils (bands) = 7%, the steps are Step 1: 7% + 7% = 14%. Step 2: 0.14 1000 = 140 ANC.

5. What is true about the genetic transmission of sickle cell disease? a. Both parents must carry the sickle cell trait. b. Both parents must have sickle cell disease. c. One parent must have the sickle cell trait. d. Sickle cell disease has no known pattern of inheritance.

ANS: A A In this scenario, there is a 50% risk of having a child with sickle cell disease. B The sickle cell trait, not the disease itself, must be present in the parents for the child to have the disease. C An autosomal recessive pattern of inheritance means that both parents must be carriers of the sickle cell trait. D Sickle cell disease is known to have an autosomal recessive pattern of inheritance.

The nurse notes that a child's gums bleed easily and he has bruising and petechiae on his extremities. What laboratory values are consistent with these symptoms? a. Platelet count of 19,000/mm3 b. Prothrombin time of 11 to 15 seconds c. Hematocrit of 34 d. Leukocyte count of 14,000/mm3

ANS: A Feedback A The normal platelet count is 150,000 to 400,000/mm3. This finding is very low, indicating an increased bleeding potential. The child should be monitored closely for signs of bleeding. B The prothrombin time of 11 to 15 seconds is within normal limits. C The normal hematocrit is 35 to 45 and, although this finding is low, it would not create the symptoms presented. D This value indicates the probable presence of infection, but it is not a reflection of bleeding tendency.

A child with acute myeloblastic leukemia is scheduled to have a bone marrow transplant (BMT). The donor is the child's own umbilical cord blood that had been previously harvested and banked. This type of BMT is termed a. Autologous b. Allogeneic c. Syngeneic d. Stem cell

ANS: A Feedback A In an autologous transplant, the child's own marrow or previously harvested and banked cord blood is used. B In an allogeneic BMT, histocompatibility has been matched with a related or an unrelated donor. C In a syngeneic transplant, the child receives bone marrow from an identical twin. D A stem cell transplantation uses a unique immature cell present in the peripheral circulation.

The nurse notes that a child's gums bleed easily and he has bruising and petechiae on his extremities. What laboratory values are consistent with these symptoms? a. Platelet count of 19,000/mm3 b. Prothrombin time of 11 to 15 seconds c. Hematocrit of 34 d. Leukocyte count of 14,000/mm3

ANS: A Feedback A The normal platelet count is 150,000 to 400,000/mm3. This finding is very low, indicating an increased bleeding potential. The child should be monitored closely for signs of bleeding. B The prothrombin time of 11 to 15 seconds is within normal limits. C The normal hematocrit is 35 to 45 and, although this finding is low, it would not create the symptoms presented. D This value indicates the probable presence of infection, but it is not a reflection of bleeding tendency.

What should the nurse recognize as symptoms of a brain tumor in a school-age child for whom she is caring? (Select all that apply.) a. Blurred vision b. Increased head circumference c. Vomiting when getting out of bed d. Intermittent headache e. Declining academic performance

ANS: A, C, D, E Feedback Correct Visual changes such as nystagmus, diplopia, and strabismus are manifestations of a brain tumor. The change in position on awakening causes an increase in intracranial pressure, which is manifested as vomiting. Vomiting on awakening is considered a hallmark symptom of a brain tumor. Increased intracranial pressure resulting from a brain tumor is manifested as a headache. School-age children may exhibit declining academic performance, fatigue, personality changes, and symptoms of vague, intermittent headache. Other symptoms may include seizures or focal neurologic deficits. Incorrect Manifestations of brain tumors vary with tumor location and the child's age and development. Infants with brain tumors may have increased head circumference with a bulging fontanel. School-age children have closed fontanels and therefore their head circumferences do not increase with brain tumors.

The nurse is caring for a child with iron-deficiency anemia. What should the nurse expect to find when reviewing the results of the complete blood count (CBC)? Select all that apply. a. Low hemoglobin levels b. Elevated red blood cell (RBC) levels c. Elevated mean cell volume (MCV) levels d. Low reticulocyte count e. Decreased MCV levels

ANS: A, D, E Correct The results of the complete blood count in a child with iron-deficiency anemia will show low hemoglobin levels (6 to 11 g/dL) and microcytic, hypochromic RBCs; this manifests as decreased MCV and decreased mean cell hemoglobin. The reticulocyte count is usually slightly elevated or normal. Incorrect: The reticulocyte count is usually slightly elevated or normal, and mean cell volume levels are decreased, not increased.

What is an expected physical assessment finding for an adolescent with a diagnosis of Hodgkin disease? a. Protuberant, firm abdomen b. Enlarged, painless, firm cervical lymph nodes c. Soft tissue swelling d. Soft to hard, nontender mass in pelvic area

ANS: B Feedback A A protuberant, firm abdomen is present in many cases of neuroblastoma. B Painless, firm, movable adenopathy (enlarged lymph nodes) palpated in the cervical region is an expected assessment finding in Hodgkin disease. Other systemic symptoms include unexplained fevers, weight loss, and night sweats. C Soft tissue swelling around the affected bone is a manifestation of Ewing sarcoma. D A soft to hard, nontender mass can be palpated when rhabdomyosarcoma is present.

While completing an assessment on a 6-month-old infant, which finding should the nurse recognize as a symptom of a brain tumor in an infant? a. Blurred vision b. Increased head circumference c. Vomiting when getting out of bed d. Headache

ANS: B Feedback A Visual changes such as nystagmus, diplopia, and strabismus are manifestations of a brain tumor but would not be able to be verbalized by an infant. B Manifestations of brain tumors vary with tumor location and the child's age and development. Infants with brain tumors may be irritable or lethargic, feed poorly, and have increased head circumference with a bulging fontanel. C The change in position on awakening causes an increase in intracranial pressure, which is manifested as vomiting. Vomiting on awakening is considered a hallmark symptom of a brain tumor, but infants do not get themselves out of bed in the morning. D Increased intracranial pressure resulting from a brain tumor is manifested as a headache but could not be verbalized by an infant.

The nurse is caring for a child with aplastic anemia. What nursing diagnoses are appropriate? Select all that apply. a. Acute Pain related to vaso-occlusion b. Risk for Infection related to inadequate secondary defenses or immunosuppression c. Ineffective Protection related to thrombocytopenia d. Ineffective Tissue Perfusion related to anemia e. Ineffective Protection related to abnormal clotting

ANS: B, C, D Correct These are appropriate nursing diagnosis for the nurse planning care for a child with aplastic anemia. Aplastic anemia is a condition in which the bone marrow ceases production of the cells it normally manufactures, resulting in pancytopenia. The child will have varying degrees of the disease depending on how low the values are for absolute neutrophil count (affecting the body's response to infection), platelet count (putting the child at risk for bleeding), and absolute reticulocyte count (causing the child to have anemia). Incorrect: Acute pain related to vaso-occlusion is an appropriate nursing diagnosis for sickle cell anemia for the child in vaso-occlusive crisis, but it is not applicable to a child with aplastic anemia. Ineffective protection related to abnormal clotting is an appropriate diagnosis for von Willebrand disease.

1. What is the best response to a parent who asks the nurse whether her 5-month-old infant can have cow's milk? a. "You need to wait until she is 8 months old and eating solids well." b. "Yes, if you think that she will eat enough meat to get the iron she needs." c. "Infants younger than 12 months need iron-rich formula to get the iron they need." d. "Try it and see how she tolerates it."

ANS: C A A 5-month-old infant cannot get adequate iron without drinking an iron-fortified formula or taking an iron supplement. B The American Academy of Pediatrics recommends beginning solid foods at 4 to 6 months of age. Meats are typically introduced in later infancy. Iron-fortified formula is still recommended. C Infants younger than 12 months need iron-fortified formula or breast milk. Infants who drink cow's milk do not get adequate iron and are at risk for iron deficiency anemia. D Counseling a parent to give a 5-month-old infant cow's milk is inappropriate.

11. How should the nurse respond when asked by the mother of a child with beta-thalassemia why the child is receiving deferoxamine? a. "To improve the anemia." b. "To decrease liver and spleen swelling." c. "To eliminate excessive iron being stored in the organs." d. "To prepare your child for a bone marrow transplant."

ANS: C A Chronic transfusion therapy is the treatment for anemia. Deferoxamine is administered to prevent complications from repeated transfusions. B Deferoxamine is used to prevent organ damage, not as a treatment for existing conditions such as hepatosplenomegaly. C Multiple transfusions result in hemosiderosis. Deferoxamine is given to chelate iron and prevent organ damage. D Preparation for a bone marrow transplant does not include administration of deferoxamine.

Children with non-Hodgkin lymphoma are at risk for complications resulting from tumor lysis syndrome (TLS). The nurse should assess for: a. Liver failure b. CNS deficit c. Kidney failure d. Respiratory distress

ANS: C Feedback A TLS is related to intracellular electrolytes overloading the kidney as a response to the rapid lysis of tumor cells. This does not affect the liver. B TLS does not affect the CNS. C In TLS, the tumor's intracellular contents are dumped into the child's extracellular fluid as the tumor cells are lysed in response to chemotherapy. Because of the large volume of these cells, their intracellular electrolytes overload the kidneys and, if not monitored, can cause kidney failure. D TLS does not affect the lungs and cause respiratory distress.

What fluid is the best choice when a child with mucositis asks for something to drink? a. Hot chocolate b. Lemonade c. Popsicle d. Orange juice

ANS: C Feedback A A hot beverage can be irritating to mouth ulcers. B Citrus products may be very painful to an ulcerated mouth. C Cool liquids are soothing and ice pops are usually well tolerated. D Citrus products may be very painful to an ulcerated mouth

18. What is the priority in the discharge plan for a child with immune thrombocytopenic purpura (ITP)? a. Teaching the parents to report excessive fatigue to the physician b. Monitoring the child's hemoglobin level every 2 weeks c. Providing a diet that contains iron-rich foods d. Establishing a safe, age-appropriate home environment

ANS: D A Excessive fatigue is not a significant problem for the child with ITP. B ITP is associated with low platelet levels. C Increasing the child's intake of iron in the diet will not correct ITP. D Prevention of injury is a priority concern for a child with ITP.

Hematopoietic stem cell transplantation (HSCT) is the standard treatment for a child in his or her first remission with what cancer? a. ALL b. Non-Hodgkin lymphoma c. Wilms' tumor d. Acute myeloblastic leukemia (AML)

ANS: D Feedback A The standard treatment for ALL is combination chemotherapy. B Standard treatment for non-Hodgkin lymphoma is chemotherapy. Bone marrow transplantation is used to treat non-Hodgkin lymphoma that is resistant to conventional chemotherapy and radiation. C The treatment for Wilms' tumor consists of surgery and chemotherapy alone or in combination with radiation therapy. D HSCT is often used interchangeably with bone marrow transplantation and is currently standard treatment for children in their first remission with AML.

A child with leukemia is being discharged after beginning chemotherapy. Which of the following instructions will the nurse include when teaching the parents of this child? "A. provide a diet low in protein and high in carboydrates B. avoid fresh vegetables that are not cooked or peeled C. notify the doctor if the child's temp exceeds 101 degrees F D. increase the use of humidifiers throughout the house"

Answer B - fresh vegetables harbor microorganisms, which can cause infections in immune-compromised children, fruit or vegetables should be either peeled or cooked. The physician should be notified of a temp above 100 degrees F. A diet low in protein is not indicated. Humidifiers harbor fungi in the water containers.

When caring for an 11-month-old infant with dehydration and metabolic acidosis, the nurse expects to see which of the following? "a. A reduced white blood cell count b. A decreased platelet count c. Shallow respirations d. Tachypnea"

Answer D. The body compensates for metabolic acidosis via the respiratory system, which tries to eliminate the buffered acids by increasing alveolar ventilation through deep, rapid respirations, altered white blood cell or platelet counts are not specific signs of metabolic imbalance.

When teaching a patient with thrombocytopenia about medications, which medications would the nurse advise the patient to avoid? Select all that apply. One, some, or all responses may be correct. 1) Aspirin 2) Contact sports 3) Iron supplements 4) Anticoagulant therapy 5) Nonsteroidal anti-inflammatory drugs (NSAIDs)

Aspirin Contact sports Anticoagulant therapy Nonsteroidal anti-inflammatory drugs (NSAIDs)

The pediatric nurse understands that the most common cancer found in children is: A. Non-hodgkin's lymphoma B. Acute lymphocytic leukemia C. Chronic lymphocytic leukemia D. Ewing's sarcoma"

B (1. No - this is not a common cancer in children 2. YES! this is the most common form of cancer found in children is acute lymphocytic leukemia. 3. No - this is not a common cancer in children 4. No - this is not a common cancer in children)

The nurse is caring for an 8-year-old girl who has been diagnosed with leukemia and will have a variety of tests, including a lumbar puncture, before beginning chemotherapy. Which of the following would be the priority? A) Applying EMLA to the lumbar puncture site B) Educating the child and family about the testing procedures C) Administering promethazine as ordered for nausea D) Educating the family about chemotherapy and its side effects

B) Educating the child and family about the testing procedures

The nurse is assessing a 3-year-old boy whose parents brought him to the clinic when they noticed that the right side of his abdomen was swollen. Which of the following findings would suggest this child has a neuroblastoma? A) The child has a maculopapular rash on his palms. B) The parents report that their son is vomiting and not eating well. C) The parents report that their son is irritable and not gaining weight. D) Auscultation reveals wheezing with diminished lung sounds.

B) The parents report that their son is vomiting and not eating well.

A pediatric oncology patient is undergoing chemotherapy. Which observation would lead the nurse to suspect that the patient has developed sterile hemorrhagic cystitis? A. Absence of hematuria B. Presence of proteinuria C. Complaints by the patient that it burns upon urination D. Increased sensation of thirst.

B.

What is most descriptive of the pathophysiology of leukemia? a. Increased blood viscosity occurs. b. Thrombocytopenia (excessive destruction of platelets) occurs. c. Unrestricted proliferation of immature white blood cells (WBCs) occurs. d. The first stage of the coagulation process is abnormally stimulated

C

The nurse is teaching the parents of a child with a hematologic disorder about the functions of the various blood cells. The nurse determines that the teaching was successful when the parents state which blood cell as being primarily responsible for blood clotting? A)Granulocytes B)Erythrocytes C)Thrombocytes D)Leukocytes

C)Thrombocytes

Which findings would the nurse suspect to be observe during a work up for in a pediatric patient suspected of having a supratentorial brain tumor? A. Vomiting not related to feeding. B. Headache upon arising. C. Personality changes. D. Seizures. E. Visual deficits.

C, D, E The presence of a supratentorial brain tumor typically presents with behavior or personality changes, seizures, visual disturbances or hemiparesis. Vomiting not related to feeding and headache upon arising are typically seen with infratentorial brain tumors.

A pediatric oncology patient has been discharged home following a course of chemotherapy. Which information should be included as part of discharge planning with regard to health promotion? A. No further treatments are needed and the patient can resume routine health assessments as developmentally appropriate. B. There are no restrictions based on activity and/or contacts with friends and family members. C. Certain restrictions will be in place related to immunizations that can be administered. D. The patient should limit fluid intake for several months in order to prevent overhydration from occurring.

C.

Administration of colony stimulating agents for the pediatric oncology patient are used to: A.decrease nausea. B. shrink tumor size. C. increase bone marrow response. D. decrease production of stem cells.

C.

A pediatric oncology patient is undergoing chemotherapy. Which treatment option would the nurse anticipate being included in the plan of care in order to prevent the development of sterile hemorrhagic cystitis? A. Restrictive fluid intake. B. Inclusion of dairy foods in the diet. C. Implementing a frequent voiding plan throughout the course of the day to the patient. D. Limiting mobility during course of chemotherapy.

C. Providing a frequent voiding plan to encourage the patient to void upon urge, immediately upon arising, before bedtime and one nighttime void will help to prevent possibility of urinary stasis. Encouraging fluid intake rather than restricting fluid is the mainstay of treatment. Dairy foods in the diet provide no effective treatment against the development of sterile hemorrhagic cystitis. Similarly, limiting mobility is not indicated.

The nurse is preparing to administer ondansetron (Zofran) to a pediatric patient. For which clinical symptom is this considered to be the drug of choice? A. Headache relief B. To promote increased energy C. Nausea and vomiting D. Pain relief

C. Nausea and vomiting Zofran is a 5-hydroxytryptamine-3 receptor antagonist and is considered the antiemetic of choice for oncology patients as it produces no extrapyramidal side effects. This medication does not treat headaches. This medication does not promote energy. This medication does not provide pain relief.

Which physical finding would suggest that a pediatric oncology patient has stomatitis? A. Intact buccal mucosa B. Presence of epistaxis C. Red painful area noted in the pharynx D. Nasal congestion

C. Red painful area noted in the pharynx Stomatitis is the presence of ulcers within the oral cavity. They are associated with red, eroded painful areas in the mouth or pharynx. An intact buccal mucosa is normal finding. Presence of epistaxis indicates a nosebleed. Nasal congestion is an unrelated finding.

A child being treated for Acute Lymphocytic Leukemia (ALL) has a white blood cell (WBC) count of 7,000/mm3. the nursing care plan lists risk for infection as a priority nursing diagnosis, and measures are being taken to reduce the child's exposure to infection. the nurse determines that the plan has been successful when which outcome has been met? "1. child's WBC count goes up. 2. child's WBC count goes down. 3. child's temperature remains within normal range. 4. parents demonstrate good hand washing technique."

CORRECT is #3 - RATIONALE: in leukemia, the WBCs that are present are immature and incapable of fighting infection. increases or decreases in the number of WBCs can be related to the disease process and treatment, and not related to infection. the only value that indicates the child is infection-free is the temperature. the use of proper handwashing technique is a measure or intervention used to meet a goal. but is not a goal itself. STRATEGY: the core issue of the question is knowledge of an indicator of infection in a client who is immunosuppressed from leukemia. recall that temperature and WBC counts are frequently used as indicators of infection. recall that in leukemia the WBCs are abnormal so choose the option related to temperature.

A 12 year old boy seen in the clinic, and a diagnosis of Hodgkin's disease is suspected . Which diagnostic test results confirm the diagnosis of Hodgkin's disease? 1 . Elevated vanillylmandelic acid urinary level. 2. The presence of blast cells in the bone marrow 3. The presence of Epsetin-Barr virus in the blood. 4. The presence of Reed-Sternberg cells in the lymph nodes

Correct Answer 4 . Hodgkin's disease is a neoplasm of lymphatic tissue. The presence of gaint multinucleated cells ( Reed- Sternbergs cells) is the hallmark of this disease. The presence of blast cells in the bone marrow indicates leukemia. The Epstein-Barr virus is associated with infectious mononucleosis . Elevated levels of vanillylmandelic acid in the urine may be found in children with neroblastoma.

Oral iron is prescribed for a child with iron deficiency anemia. The nurse provides instructions to the mother regarding the administration of the iron. The nurse should instruct the mother to administer the medication in which way? 1.Between meals 2.Just before a meal 3.Just after the meal 4.With a fruit low in vitamin C

Correct Answer: 1 Rationale: The mother should be instructed to administer oral iron supplements between meals. The iron should be given with a citrus fruit or juice high in vitamin C because vitamin C increases the absorption of iron by the body.

The mother of a 4-year-old child tells the pediatric nurse that the child's abdomen seems to be swollen. During further assessment, the mother tells the nurse that the child is eating well and that the activity level of the child is unchanged. The nurse, suspecting the possibility of Wilms' tumor, should avoid which during the physical assessment? 1.Palpating the abdomen for a mass 2.Assessing the urine for the presence of hematuria 3.Monitoring the temperature for the presence of fever 4.Monitoring the blood pressure for the presence of hypertension

Correct Answer: 1 Rationale: Wilms' tumor is the most common intraabdominal and kidney tumor of childhood. If Wilms' tumor is suspected, the tumor mass should not be palpated by the nurse. Excessive manipulation can cause seeding of the tumor and spread of the cancerous cells. Hematuria, fever, and hypertension are clinical manifestations associated with Wilms' tumor.

The nurse is reviewing a health care provider's prescriptions for a child with sickle cell anemia who was admitted to the hospital for the treatment of vaso-occlusive crisis. Which prescriptions documented in the child's record should the nurse question? Select all that apply. 1.Restrict fluid intake. 2.Position for comfort. 3.Avoid strain on painful joints. 4.Apply nasal oxygen at 2 L/minute. 5.Provide a high-calorie, high-protein diet. 6.Give meperidine, 25 mg intravenously, every 4 hours for pain.

Correct Answer: 1,6 Rationale:Sickle cell anemia is one of a group of diseases termed hemoglobinopathies, in which hemoglobin A is partly or completely replaced by abnormal sickle hemoglobin S. It is caused by the inheritance of a gene for a structurally abnormal portion of the hemoglobin chain. Hemoglobin S is sensitive to changes in the oxygen content of the red blood cell; insufficient oxygen causes the cells to assume a sickle shape, and the cells become rigid and clumped together, obstructing capillary blood flow. Oral and intravenous fluids are an important part of treatment. Meperidine is not recommended for a child with sickle cell disease because of the risk for normeperidine-induced seizures. Normeperidine, a metabolite of meperidine, is a central nervous system stimulant that produces anxiety, tremors, myoclonus, and generalized seizures when it accumulates with repetitive dosing. The nurse would question the prescription for restricted fluids and meperidine for pain control. Positioning for comfort, avoiding strain on painful joints, oxygen, and a high-calorie and high-protein diet are also important parts of the treatment plan.

The pediatric nurse assists the health care provider in performing a lumbar puncture on a 3-year-old child with leukemia and suspected central nervous system metastasis. The nurse should place the child in which position for this procedure? 1.Lithotomy position 2.Modified Sims' position 3.Lateral recumbent, knees flexed to the abdomen and the head bent, chin down 4.Prone, with the knees flexed to the abdomen and the head bent, the chin resting on the chest

Correct Answer: 3 Rationale: A lateral recumbent position, with the knees flexed to the abdomen and the head bent with the chin resting on the chest, is assumed for a lumbar puncture. This position separates the spinal processes and facilitates needle insertion into the subarachnoid space. The remaining options are incorrect positions.

The nurse is providing home care instructions to the parents of a 10-year-old child with hemophilia. Which sport activity should the nurse suggest for this child? 1.Soccer 2.Basketball 3.Swimming 4.Field hockey

Correct Answer: 3 Rationale: Hemophilia refers to a group of bleeding disorders resulting from a deficiency of specific coagulation proteins. Children with hemophilia need to avoid contact sports and to take precautions such as wearing elbow and knee pads and helmets with other sports. The safe activity for them is swimming.

A child with a diagnosis of sickle cell disease is being admitted for the treatment of vaso-occlusive crisis. The nurse prepares for the admission anticipating which prescription for the child? 1.NPO status 2.Meperidine for pain 3.Intravenous fluids 4.Intubation to administer oxygen

Correct Answer: 3 Rationale: Intravenous fluid and increased oral fluids are a component of the treatment plan for the child with vaso-occlusive crisis. Management of the severe pain that occurs with vaso-occlusive crisis includes the use of opioid analgesics, such as morphine sulfate and hydromorphone. Meperidine is contraindicated because of its side effects and the increased risk of seizures with its use. Oxygen is administered when hypoxia is present and the oxygen saturation level is less than 95%. Intubation is not necessary to treat vaso-occlusive crisis.

A nursing student is assigned to care for a child with sickle cell disease (SCD). The nursing instructor asks the student to describe the causative factors related to this disease. Which statement by the student indicates a need for further research? 1.SCD is an autosomal recessive disease. 2.Children with the HbS (sickle cell hemoglobin) trait are not symptomatic. 3.If each parent carries the trait, the child will carry the trait, and the probability of the child having the disease is 75%. 4.If one parent has the HbS trait and the other parent is normal, there is a 50% chance that each offspring will inherit the trait.

Correct Answer: 3 Rationale: SCD is an autosomal recessive disease. Children with the HbS trait are not symptomatic. If one parent has the HbS trait and the other parent is normal, there is a 50% chance that each offspring will inherit the trait. If each parent carries the trait, there is a 25% chance that their child will be normal, a 50% chance that the child will carry the trait, and a 25% chance that each child will have the disease.

The nurse provides instructions regarding home care to the parents of a 3-year-old child hospitalized with hemophilia. Which statement, if made by the parent, indicates a need for further instructions? 1."We will supervise our child closely." 2."We will pad corners of the furniture." 3."We will avoid having our child receive immunizations." 4."We will remove household items that can easily fall over."

Correct Answer: 3 Rationale: The nurse needs to stress the importance of immunizations, dental hygiene, and routine well-child care. The remaining options are appropriate. The parents also are instructed in the measures to implement in the event of blunt trauma, especially trauma involving the joints, and taught to apply prolonged pressure to superficial wounds until the bleeding has stopped.

The pediatric nurse educator provides a teaching session to the nursing staff regarding hemophilia. Which statement regarding this disorder should the nurse plan to include in the discussion? 1.Males inherit hemophilia from their fathers. 2.Hemophilia is a Y-linked hereditary disorder. 3.Females inherit hemophilia from their mothers. 4.Hemophilia A results from deficiency of factor VIII.

Correct Answer: 4 Rationale: Hemophilia refers to a group of bleeding disorders resulting from a deficiency of specific coagulation proteins. Hemophilia A results from a deficiency of factor VIII. Males inherit hemophilia from their mothers, and females inherit the carrier status from their fathers. Hemophilia is inherited in a recessive manner via a genetic defect on the X chromosome. Hemophilia B (Christmas disease) is a deficiency of factor IX.

The nurse is asked to prepare for the admission of a child to the pediatric unit with a diagnosis of Wilms' tumor. The nurse is creating a plan of care for the child and should include which intervention in the plan? 1.Monitor the temperature for hypothermia. 2.Monitor the blood pressure for hypotension. 3.Palpate the abdomen for an increase in the size of the tumor. 4.Inspect the urine for the presence of hematuria at each voiding.

Correct Answer: 4 Rationale: If Wilms' tumor is suspected, the tumor mass should not be palpated. Excessive manipulation can cause seeding of the tumor and cause spread of the cancerous cells. Fever (not hypothermia), hematuria, and hypertension (not hypotension) are clinical manifestations associated with Wilms' tumor.

An 8-year-old girl is receiving a blood transfusion when the nurse notes that she has developed precordial pain, dyspnea, distended neck veins, slight cyanosis, and a dry cough. These manifestations are most suggestive of: a. Air embolism. c. Hemolytic reaction. b. Allergic reaction. d. Circulatory overload

D

An accurate description of anemia is: a. Increased blood viscosity. b. Depressed hematopoietic system. c. Presence of abnormal hemoglobin. d. Decreased oxygen-carrying capacity of blood.

D

Which clinical manifestation should the nurse expect when a child with sickle cell anemia experiences an acute vaso-occlusive crisis? a. Circulatory collapse Test Bank - Maternal Child Nursing Care by Perry (6th Edition, 2017) 716 b. Cardiomegaly, systolic murmurs c. Hepatomegaly, intrahepatic cholestasis d. Painful swelling of hands and feet, painful joints

D

The nurse is completing a care plan for a client diagnosed with leukemia. Which independent problem should be addressed? A. Infection. B. Anemia. C. Nutrition. D. Grieving.

D (Grieving is an independent problem, and the nurse can assess and treat this problem with or without collaboration)

he nurse is caring for a 7-year-old girl who is undergoing a stem cell transplant. Which of the following would the nurse include in the child's postoperative plan of care? A) Assessing for petechiae, purpura, bruising, or bleeding B) Limiting blood draws to the minimum volume required C) Administering antiemetics around the clock as ordered D) Monitoring for severe diarrhea and maculopapular rash

D) Monitoring for severe diarrhea and maculopapular rash

You are reviewing information relative to a patient's medical history for treatment of leukemia. Patient is exhibiting no clinical symptoms at this point in the treatment plan. In comparing bone marrow reports prior to and 6 months following chemotherapy, what information do you hope to obtain that would assist in evaluating the plan of care? A. Expectation that the results will be consistent indicating that goals have been met. B. Increased likelihood that atypical cells will be present suggesting a revision of the plan of care. C. Pancytopenic response indicating that chemotherapy treatment was successful. D. Determination of response to clinical therapy comparing pre and post procedure that will provide evidence to interpret whether medical treatment has been effective.

D. Bone marrow biopsies are used both to diagnose as well as evaluate clinical response to chemotherapy (therapeutic management) used in the treatment of leukemia. While one would hope that the intervention was successful, until the results are compared and read by the pathologist, there is no way to state equivocally what the results will be at the histological level. Consistent findings pre and post treatment would indicate that treatment goals have not been met. Similarly, if clinical response is favorable, then one would not expect to see atypical cells. As the patient is not experiencing any symptoms, a diagnosis of pancytopenia would not be expected as this would indicate bone marrow failure.

The nurse in planning care for the pediatric oncology patient anticipates implementing which action with regard to the administration of an antiemetic in a chemotherapy protocol? A. Providing the medication on a prn basis based on patient's presenting symptoms of nausea and/or vomiting. B. Administering the medication via the oral route following infusion of chemotherapy protocol. C. Providing medication with sips of water following clinical symptoms of nausea and/or vomiting. D. Administering 30 to 60 minutes prior to initiation of therapy.

D. Anticipatory management of an antiemetic is part of chemotherapy and/or radiation protocols. It is typically given 30 to 60 minutes prior to the infusion and administered in a scheduled sequence rather than based on a prn or when the patient presents symptomatically. Preferred route of administration is via parenteral route especially if the anticipated risk for nausea and/or vomiting is increased.

c

In a child with sickle cell anemia (SCA), adequate hydration is essential to minimize sickling and delay the vasoocclusion and hypoxiaischemia cycle. What information should the nurse share with parents in a teaching plan? a. Encourage drinking. b. Keep accurate records of output. c. Check for moist mucous membranes. d. Monitor the concentration of the childs urine.

Patients with hypochromic, microcytic anemia need to increase their intake of which nutrient? 1) B12 2) Iron 3) Vitamin D 4) Cyanocobalamin

Iron

b

Lucas, age 7 years, is receiving a transfusion of packed RBCs. After 45 minutes, he begins to have chills, fever, a sensation of tightness in the chest, and headache. The priority action of the nurse is to a. stop the transfusion, maintain a patent IV line with normal saline and new tubing, and administer acetaminophen b. stop the transfusion, maintain a patent IV line with normal saline and new tubing, and notify the practitioner c. slow the transfusion rate until the symptoms subside d. slow the transfusion and send a sample of the patient's blood and urine to the laboratory

Which example would the nurse provide for a patient who is experiencing disseminated intravascular coagulation (DIC) and asks about the pathologic cause of the disorder? 1) Sepsis 2) Anemia 3) Hemophilia 4) Minor trauma

Sepsis

c

The parents of a child with sickle cell anemia (SCA) are concerned about subsequent children having the disease. What statement most accurately reflects inheritance of SCA? a. SCA is not inherited. b. All siblings will have SCA. c. Each sibling has a 25% chance of having SCA. d. There is a 50% chance of siblings having SCA.

Which frequently used therapy would the nurse explain when teaching a patient with a hematologic disorder? (Select all that apply. One, some, or all responses may be correct.) 1) Transfusions 2) Splenectomy 3) Erythropheresis 4) Magnesium therapy 5) Stem cell transplantation

Transfusions Splenectomy Stem cell transplantation

d

What statement best describes iron deficiency anemia in infants? a. It is caused by depression of the hematopoietic system. b. Diagnosis is easily made because of the infants emaciated appearance. c. It results from a decreased intake of milk and the premature addition of solid foods. d. Clinical manifestations are related to a reduction in the amount of oxygen available to tissues.

b

Which test provides a definitive diagnosis of aplastic anemia? a. complete blood count with differential b. bone marrow aspiration c. serum IgG levels d. basic metabolic panel

When both parents have sickle cell trait, which is the chance their children will have sickle cell anemia? a. 25% b. 50% c. 75% d. 100%

a. 25%

When both parents have sickle cell trait, which is the chance their children will have sickle cell anemia? a. 25% b. 50% c. 75% d. 100%

a. 25% Sickle cell anemia is inherited in an autosomal recessive pattern. If both parents have sickle cell trait (one copy of the sickle cell gene), then for each pregnancy, a 25% chance exists that their child will be affected with sickle cell disease. With each pregnancy, a 50% chance exists that the child will have sickle cell trait. Percentages of 75% and 100% are too high for the children of parents who have sickle cell trait.

The nurse is caring for a child with a low platelet count. What skin assessments would alert the nurse to bleeding? (Select all that apply.) a. Petichiae b. Purpura c. Ecchymosis d. Hematoma e. Lymphadenopathy

a. Petichiae b. Purpura c. Ecchymosis d. Hematoma

The parents of a child hospitalized with sickle cell anemia tell the nurse that they are concerned about narcotic analgesics causing addiction. Which is appropriate for the nurse to explain about narcotic analgesics? a. Are often ordered but not usually needed b. Rarely cause addiction because they are medically indicated c. Are given as a last resort because of the threat of addiction d. Are used only if other measures, such as ice packs, are ineffective

b. Rarely cause addiction because they are medically indicated The pain of sickle cell anemia is best treated by a multidisciplinary approach. Mild to moderate pain can be controlled by ibuprofen and acetaminophen. When narcotics are indicated, they are titrated to effect and are given around the clock. Patient-controlled analgesia reinforces the patient's role and responsibility in managing the pain and provides flexibility in dealing with pain. Few, if any, patients who receive opioids for severe pain become behaviorally addicted to the drug. Narcotics are often used because of the severe nature of the pain of vasoocclusive crisis. Ice is contraindicated because of its vasoconstrictive effects.

The nurse has initiated a blood transfusion on a preschool child. The child begins to exhibit signs of a transfusion reaction. Place in order the interventions the nurse should implement sequencing from the highest priority to the lowest. Provide answer using lowercase letters separated by commas (e.g., a, b, c, d). a. Take the vital signs. b. Stop the transfusion. c. Notify the practitioner. d. Maintain a patent IV line with normal saline..

b. Stop the transfusion. a. Take the vital signs. d. Maintain a patent IV line with normal saline. c. Notify the practitioner. ANS: b, a, d, c If a blood transfusion reaction of any type is suspected, stop the transfusion, take vital signs, maintain a patent IV line with normal saline and new tubing, notify the practitioner, and do not restart the transfusion until the child's condition has been medically evaluated.

A school-age child with leukemia experienced severe nausea and vomiting when receiving chemotherapy for the first time. Which is the most appropriate nursing action to prevent or minimize these reactions with subsequent treatments? a. Encourage drinking large amounts of favorite fluids. b. Encourage child to take nothing by mouth (remain NPO) until nausea and vomiting subside. c. Administer an antiemetic before chemotherapy begins. d. Administer an antiemetic as soon as child has nausea.

c. Administer an antiemetic before chemotherapy begins. The most beneficial regimen to minimize nausea and vomiting associated with chemotherapy is to administer the antiemetic before the chemotherapy is begun. The goal is to prevent anticipatory symptoms. Drinking fluids will add to the discomfort of the nausea and vomiting. Remaining until nausea and vomiting subside will help with this episode, but the child will have the discomfort and be at risk for dehydration. Administering an antiemetic as soon as the child has nausea does not prevent anticipatory nausea.

Which is most descriptive of the pathophysiology of leukemia? a. Increased blood viscosity occurs. b. Thrombocytopenia (excessive destruction of platelets) occurs. c. Unrestricted proliferation of immature white blood cells (WBCs) occurs. d. First stage of coagulation process is abnormally stimulated.

c. Unrestricted proliferation of immature white blood cells (WBCs) occurs. Leukemia is a group of malignant disorders of the bone marrow and lymphatic system. It is defined as an unrestricted proliferation of immature WBCs in the blood-forming tissues of the body. Increased blood viscosity may occur secondary to the increased number of WBCs. Thrombocytopenia may occur secondary to the overproduction of WBCs in the bone marrow. The coagulation process is unaffected by leukemia.

A child has just been diagnosed with acute lymphoblastic leukemia. What is the result of an overproduction of immature white blood cells in the bone marrow? a. Decreased T-cell production b. Decreased hemoglobin c. Increased blood clotting d. Increased susceptibility to infection

d. Increased susceptibility to infection An overproduction of immature white blood cells increases the child's susceptibility to infection.

Which clinical manifestation should the nurse expect when a child with sickle cell anemia experiences an acute vasoocclusive crisis? a. Circulatory collapse b. Cardiomegaly, systolic murmurs c. Hepatomegaly, intrahepatic cholestasis d. Painful swelling of hands and feet; painful joints

d. Painful swelling of hands and feet; painful joints

Where is the primary site of origin of the tumor in children who have neuroblastoma? 1. Bone. 2. Kidney. 3. Abdomen. 4. Liver.

3. Neuroblastoma tumors originate from embryonic neural crest cells that normally give rise to the adrenal medulla and the sympathetic nervous system. The majority of the tumors arise from the adrenal gland or from the retroperitoneal sympathetic chain. Therefore, the primary site is within the abdomen

Which intervention would the nurse identify in the plan of care when the laboratory report reflects an increase in the uric acid level for a patient receiving chemotherapy for leukemia? 1) Maintaining adequate hydration 2) Performing meticulous mouth care 3) Holding the next dose of chemotherapy and contact the health care provider 4) Encouraging the patient to alternate periods of activity with periods of rest

Maintaining adequate hydration

Which characteristics would the nurse associate with hemophilia? (Select all that apply. One, some, or all responses may be correct.) 1) A deficiency of factor IX 2) A deficiency of factor XI 3) A deficiency of factor VIII 4) An inherited X-linked disorder 5) An inherited Y-linked disorder

A deficiency of factor IX A deficiency of factor VIII An inherited X-linked disorder

b

Patricia Marshall, age 12 years, is admitted to your unit with a diagnosis of sickle cell crisis. Which of the following activities is most likely to have precipitated this episode? a. attending the football game with her friends b. going camping and hiking in the mountains with her friends c. going to the beach and surfing with her friends d. staying indoors and reading for several hours

Which clinical manifestation would alert the nurse to a potential problem when the patient begins to hemorrhage from an incision during the immediate postoperative period? 1) Flushed, hot skin 2) Rapid, weak pulse 3) Increased urine output 4) Rising systolic blood pressure

Rapid, weak pulse

A child diagnosed with stage IV neuroblastoma has undergone abdominal surgery to remove the tumor. He is now receiving chemotherapy. Which nursing diagnosis would be most important? A) Risk for infection related to chemotherapy B) Impaired skin integrity related to abdominal surgery C) Grieving related to advanced disease and poor prognosis D) Imbalanced nutrition related to adverse effects of chemotherapy

C) Grieving related to advanced disease and poor prognosis

Which clinical manifestations would the nurse associate with an acute hemolytic transfusion reaction? (Select all that apply. One, some, or all responses may be correct.) 1) Chills 2) Fever 3) Bradycardia 4) Hypertension 5) Low back pain

Chills Fever Low back pain

"What is a characteristic manifestation of Hodgkin's Disease? "1.) petechiae 2.) erythematous rash 3.) enlarged lymph nodes 4.) pallor"

Correct: #3 "Knowledge of the usual pattern of spread of this lymphoma, with its orderly progression through lymph node groups and its typical forms of extranodal involvement, facilitates timely diagnosis, staging, and treatment planning".

d

What condition precipitates polycythemia? a. Dehydration b. Severe infections c. Immunosuppression d. Prolonged tissue hypoxia

The parent of a 4-year-old brings the child to the clinic and tells the nurse the child ' s abdomen is distended. After a complete examination, a diagnosis of Wilms tumor is suspected. Which of the following is most important when doing a physical examination on this child? 1. Avoid palpation of the abdomen. 2. Assess the urine for the presence of blood. 3. Monitor vital signs, especially the blood pressure. 4. Obtain an accurate height and weight.

1. Palpating the abdomen of the child in whom a diagnosis of Wilms tumor is suspected should be avoided because manipulation of the abdomen may cause seeding of the tumor.

The nurse is caring for a child being treated for ALL. Laboratory results indicate that the child has a white blood cell count of 5000/mm 3 with 5% polys and 3% bands. Which of the following analyses is most appropriate? 1. The absolute neutrophil count is 400/mm 3 , and the child is neutropenic. 2. The absolute neutrophil count is 800/mm 3 , and the child is neutropenic. 3. The absolute neutrophil count is 4000/mm 3 , and the child is not neutropenic. 4. The absolute neutrophil count is 5800/mm 3 , and the child is not neutropenic.

1. The calculated absolute neutrophil count is 400/mm 3 (0.08 Å~ 5000), and the child is neutropenic because the count is less than 500/mm 3 .

The nurse is caring for a 10-year-old with leukemia who is receiving chemotherapy. The child is on neutropenic precautions. Friends of the child come to the desk and ask for a vase for the fl owers they have brought with them. Which of the following is the best response? 1. "I will get you a special vase that we use on this unit." 2. "The fl owers from your garden are beautiful but should not be placed in the room at this time." 3. "As soon as I can wash a vase, I will put the fl owers in it and bring it to the room." 4. "Get rid of the fl owers immediately. You could harm the child."

2. A neutropenic client should not have fl owers in the room because the fl owers may harbor Aspergillus or Pseudomonas aeruginosa. Neutropenic children are susceptible to infection. Precautions need to be taken so that the child does not come in contact with any potential sources of infection. Fresh fruits and vegetables can also harbor molds and should be avoided. Telling the friend that the fl owers are beautiful but that the child cannot have them is a tactful way not to offend the friend.

b

A child with sickle cell disease is in a vasoocclusive crisis. What nonpharmacologic pain intervention should the nurse plan? a. Exercise as a distraction b. Heat to the affected area c. Elevation of the extremity d. Cold compresses to the affected area

Which of the following will be abnormal in a child with the diagnosis of hemophilia? 1. Platelet count. 2. Hemoglobin level. 3. White blood cell count. 4. Partial thromboplastin time (PTT).

4. The abnormal laboratory results in hemophilia are related to decreased clotting function. Partial thromboplastin time is prolonged.

The most appropriate nursing diagnosis for a child with anemia is: Test Bank - Maternal Child Nursing Care by Perry (6th Edition, 2017) 713 a. Activity Intolerance related to generalized weakness. b. Decreased Cardiac Output related to abnormal hemoglobin. c. Risk for Injury related to depressed sensorium. d. Risk for Injury related to dehydration and abnormal hemoglobin

A

The nurse is administering an intravenous chemotherapeutic agent to a child with leukemia. The child suddenly begins to wheeze and have severe urticaria. Which is the most appropriate nursing action? a. Stop drug infusion immediately. b. Recheck rate of drug infusion. c. Observe child closely for next 10 minutes. d. Explain to child that this is an expected side effect.

A

The nurse is planning care for an adolescent with acquired immunodeficiency syndrome. The priority nursing goal is to: a. Prevent infection. c. Restore immunologic defenses. b. Prevent secondary cancers. d. Identify source of infection.

A

The school nurse is informed that a child with human immunodeficiency virus (HIV) will be attending school soon. Which is an important nursing intervention? Test Bank - Maternal Child Nursing Care by Perry (6th Edition, 2017) 729 a. Carefully follow universal precautions. b. Determine how the child became infected. c. Inform the parents of the other children. d. Reassure other children that they will not become infected

A

What is the priority nursing intervention for a child hospitalized with hemarthrosis resulting from hemophilia? a. Immobilization and elevation of the affected joint b. Administration of acetaminophen for pain relief c. Assessment of the childs response to hospitalization d. Assessment of the impact of hospitalization on the family system

A

d

A child with hemophilia A will have which abnormal laboratory result? a. PT (ProTime) b. Platelet count c. Fibrinogen level d. PTT (partial thromboplastin time)

Children receiving long-term systemic corticosteroid therapy are most at risk for: a. Hypotension. b. Dilation of blood vessels in the cheeks. c. Growth delays. d. Decreased appetite and weight loss.

C

A common clinical manifestation of Hodgkins disease is: a. Petechiae. b. Bone and joint pain. c. Painful, enlarged lymph nodes. d. Enlarged, firm, nontender lymph nodes.

D

a

In which condition are all the formed elements of the blood simultaneously depressed? a. Aplastic anemia b. Sickle cell anemia c. Thalassemia major d. Iron deficiency anemia

Which of the following measures should the nurse implement to help with the nausea and vomiting caused by chemotherapy? Select all that apply. 1. Give an antiemetic 30 minutes prior to the start of therapy. 2. Continue the antiemetic as ordered until 24 hours after the chemotherapy is complete. 3. Remove food that has a lot of odor. 4. Keep the child on a nothing-by-mouth status. 5. Wait until the nausea begins to start the antiemetic.

1, 2, 3 1. The first dose should be given 30 minutes prior to the start of the therapy. 2. Antiemetic should be administered around the clock until 24 hours after the chemotherapy is completed. 3. It is also helpful to remove foods with odor so that the smell of the food does not make the child nauseated.

The nurse is caring for a child with leukemia. The nurse should be aware that children being treated for leukemia may experience which of the following complications? Select all that apply. 1. Anemia. 2. Infection. 3. Bleeding tendencies. 4. Bone deformities. 5. Polycythemia.

1, 2, 3. 1. Anemia is caused by decreased production of red blood cells. 2. Infection risk in leukemia is secondary to the neutropenia. 3. Bleeding tendencies are from decreased platelet production.

A nurse is caring for a child with von Willebrand disease. The nurse is aware that which of the following is a clinical manifestation of von Willebrand disease? Select all that apply. 1. The child bruises easily. 2. Excessive menstruation. 3. The child has frequent nosebleeds. 4. Elevated creatinine levels. 5. Elevated blood pressure.

1, 2, 3. 1. Von Willebrand disease is a hereditary bleeding disorder characterized by defi ciency of or defect in a protein. The disorder causes adherence of platelets to damaged endothelium and a mild defi ciency of factor VIII. One of the manifestations of this disease is bleeding of the mucous membranes. 2. Excessive menstruation may be a manifestation of this disease. 3. Frequent nosebleeds are a common manifestation of this disease.

A child with hemophilia A fell and injured a knee while playing outside. The knee is swollen and painful. Which of the following measures should be taken to stop the bleeding? Select all that apply. 1. The extremity should be immobilized. 2. The extremity should be elevated. 3. Warm moist compresses should be applied to decrease pain. 4. Passive range-of-motion exercises should be administered to the extremity. 5. Factor VIII should be administered.

1, 2, 5 1. Measures are needed to induce vasoconstriction and stop the bleeding, including immobilization of the extremity. 2. Measures are needed to induce vasoconstriction and stop the bleeding. Treatment should include elevating the extremity. 5. Hemophilia A is a defi ciency in factor VIII, which causes delay in clotting when there is a bleed. Giving a dose of Factor VIII concentrate will assist in the clotting process.

A child diagnosed with leukemia is receiving allopurinol (Zyloprim) as part of the treatment plan. The parents ask why their child is receiving this medication. What information about the medication should the nurse provide? 1. Helps reduce the uric acid level caused by cell destruction. 2. Helps make the chemotherapy more effective. 3. Helps reduce the nausea and vomiting associated with chemotherapy. 4. Helps decrease pain in the bone marrow

1. Allopurinol (Zyloprim) reduces serum uric acid. When there is lysis of cells from chemotherapy, there will be an increase in serum uric acid.

Which of the following is a reason to perform a lumbar puncture on a child with a diagnosis of leukemia? Select all that apply. 1. Rule out meningitis. 2. Assess the central nervous system for infiltration. 3. Give intrathecal chemotherapy. 4. Determine increased intracranial pressure. 5. Stage the leukemia.

2, 3 2. A lumbar puncture is done to determine whether the cancer cells have entered the CNS, but this would not be routine unless the child was symptomatic. 3. Chemotherapy can also be given through a lumbar puncture (spinal tap).

The nurse receives a call from a parent of a child with leukemia in remission. The parent says the child has been exposed to chickenpox and has never had it. Which of the following responses is most appropriate for the nurse? Select all that apply. 1. "You need to monitor the child ' s temperature frequently and call back if the temperature is greater than 101°F (38.3°C)." 2. "The child has had two varicella immunizations as an infant but is no longer immune after chemotherapy." 3. "You need to bring the child to the clinic for a varicella immunoglobulin vaccine." 4. "Your child will need to be isolated for the next 2 weeks." 5. "Your child may develop chicken pox lesions about 14 to 21 days after exposure."

2, 3 2. Chickenpox exposure is a real concern for a child who is immunocompromised, and action needs to be taken. 3. The child should receive varicella zoster immune globulin within 96 hours of the exposure.

Which of the following describes idiopathic thrombocytopenia purpura (ITP)? Select all that apply. 1. ITP is a congenital hematological disorder. 2. ITP causes excessive destruction of platelets. 3. Children with ITP have normal bone marrow. 4. Platelets are small in ITP. 5. Purpura is observed in ITP.

2, 3, 5 2. ITP is characterized by excessive destruction of platelets. 3. The bone marrow is normal in children with ITP 5. ITP is characterized by purpura, which are areas of hemorrhage under the skin.

Which of the following should the nurse expect to administer to a child with ITP and a platelet count of 5000/mm 3 ? Select all that apply. 1. Platelets. 2. Intravenous immunoglobulin. 3. Packed red blood cells (PRBCs). 4. White blood cells. 5. Prednisolone.

2, 5 2. Intravenous immunoglobulin is given because the cause of platelet destruction is believed to be an autoimmune response to disease-related antigens. Treatment is usually supportive. Activity is restricted at the onset because of the low platelet count and risk for injury that could cause bleeding. 5. Treatment in the acute phase is often symptomatic, and prednisolone, IVIG, and anti-D antibody are often given. This tends to shorten the course because the disease tends to resolve over time. Focus on the cause of ITP and which cells are affected.

The nurse is caring for a child diagnosed with thalassemia major who is receiving the fi rst chelation therapy. What information should the nurse provide to the parent regarding the therapy? Select all that apply. 1. Decreases the risk of bleeding. 2. Eliminates excess iron. 3. Prevents further sickling of the red blood cells. 4. Provides an iron supplement. 5. Hydration is necessary for the process to be effective.

2, 5. 2. Chelation therapy is used to rid the body of excess iron stores that result from frequent blood transfusions 5. Hydration is necessary for the process to be effective.

A 10-year-old with severe factor VIII defi ciency falls, injures an elbow, and is brought to the ED. The nurse should prepare which of the following? 1. An IM injection of factor VIII. 2. An IV infusion of factor VIII. 3. An injection of desmopressin. 4. An IV infusion of platelets

2. The child is treated with an IV infusion of factor VIII to replace the missing factor and help stop the bleeding

Which test provides a defi nitive diagnosis of aplastic anemia? 1. Complete blood count with differential. 2. Bone marrow aspiration. 3. Serum IgG levels. 4. Basic metabolic panel.

2. Definitive diagnosis is determined from bone marrow aspiration, which demonstrates the conversion of red bone marrow to yellow, fatty marrow.

Which of the following is the best method to prevent the spread of infection to an immunosuppressed child? 1. Administer antibiotics prophylactically to the child. 2. Have people wash their hands prior to contact with the child. 3. Assign the same nurses to care for the child each day. 4. Limit visitors to family members only.

2. Hand washing is the best method to prevent the spread of germs and protect the child from infection.

The parent of a child with hemophilia is asking the nurse what caused the hemophilia. Which is the nurse ' s best response? 1. It is an X-linked dominant disorder. 2. It is an X-linked recessive disorder. 3. It is an autosomal dominant disorder. 4. It is an autosomal recessive disorder

2. Hemophilia is transmitted as an X-linked recessive disorder. About 60% of children have a family history of hemophilia. The usual transmission is by a female with the trait and an unaffected male.

A nurse instructs the parent of a child with sickle cell disease about factors that might precipitate a pain crisis in the child. Which of the following factors identifi ed by the parent as being able to cause a pain crisis indicates a need for further instruction? 1. Infection. 2. Overhydration. 3. Stress at school. 4. Cold environment.

2. Overhydration does not cause a crisis.

The nurse is caring for a child with sickle cell disease who is scheduled to have a splenectomy. What information should the nurse explain to the parents regarding the reason for a splenectomy? 1. To decrease potential for infection. 2. To prevent splenic sequestration. 3. To prevent sickling of red blood cells. 4. To prevent sickle cell crisis.

2. Splenic sequestration is a life-threatening situation in children with sickle cell disease. Once a child is considered to be at high risk of splenic sequestration or has had this in the past, the spleen will be removed.

A nurse is caring for a 5-year-old with sickle cell vaso-occlusive crisis. Which of the orders should the nurse question? Select all that apply. 1. Position the child for comfort. 2. Apply hot packs to painful areas. 3. Give meperidine (Demerol) 25 mg intravenously every 4 hours as needed for pain. 4. Restrict oral fl uids. 5. Apply oxygen per nasal cannula to keep oxygen saturations above 94%.

3, 4, 5 3. Tissue hypoxia is very painful. Narcotics such as morphine are usually given for pain when the child is in a crisis. Meperidine (Demerol) should be avoided because of the risk of Demerol-induced seizures. 4. The child should receive hydration because when the child is in crisis, the abnormal S-shaped red blood cells clump, causing tissue hypoxia and pain. 5. Oxygen is of little value unless the tissue is hypoxic. The objective of treatment is to minimize hypoxia.

An 18-month-old male is brought to the clinic by his mother. His height is in the 50th percentile, and his weight is in the 80th percentile. The child is pale. The physical examination is normal, but his hematocrit level is 20%. Which of the following questions should assist the nurse in making a diagnosis? Select all that apply. 1. "How many bowel movements a day does your child have?" 2. "How much did your baby weigh at birth?" 3. "What does your child eat every day?" 4. "Has the child been given any new medications?" 5. "How much milk does your child drink per day?"

3, 5 3. A diet history is necessary to determine the nutritional status of the child and whether the child is getting suffi cient sources of iron. 5. By asking how much milk the child consumes, the nurse can determine whether the child is fi lling up on milk and then not wanting to take food.

The nurse is discharging a child who has just received chemotherapy for neuroblastoma. Which of the following statements made by the child ' s parent indicates a need for additional teaching? 1. "I will inspect the skin often for any lesions." 2. "I will do mouth care daily and monitor for any mouth sores." 3. "I will wash my hands before caring for my child." 4. "I will take a rectal temperature daily and report a temperature greater than 101°F (38.3°C) immediately to the health-care provider."

4. Monitoring the child ' s temperature and reporting it to the physician are important, but the temperature should not be taken rectally. The risk of injury to the mucous membranes is high. Rectal abscesses can occur in the damaged rectal tissue. The best method for taking the temperature is axillary, especially if the child has mouth sores.

The nurse is planning activity for a 4-year-old child with anemia. Which activity should the nurse plan for this child? a. Game of hide and seek in the childrens outdoor play area b. Participation in dance activities in the playroom c. Puppet play in the childs room d. A walk down to the hospital lobby

C

"The female client recently diagnosed with Hodgkin's lymphoma asks the nurse about her prognosis. Which is the nurse's best response? "1. Survival for Hodgkin's disease is relatively good with standard therapy. 2. Survival depends on becoming involved in an investigational therapy program. 3. Survival is poor, with more than 50% of clients dying within six (6) months. 4. Survival is fine for primary Hodgkin's, but secondary cancers occur within a year."

"1. Up to 90% of clients respond well to standard treatment with chemotherapy and radiation therapy, and those who relapse usually respond to a change of chemotherapy medications. Survival depends on the individual client and the stage of disease at diagnosis (correct). 2. Investigational therapy regimens would not be recommended for clients initially diagnosed with Hodgkin's disease because of the expected prognosis with standard therapy 3. Clients usually achieve a significantlylonger survival rate than six (6) months.Many clients survive to develop long-termsecondary complications. 4. Secondary cancers can occur as long as 20 years after a remission of the Hodgkin'sdisease has occurred."

A pediatric nurse specialist provides a teaching session to the nursing staff regarding osteogenic sarcoma. Which statement by a member of the nursing staff indicates a need for clarification of the information presented? "1.) ""The femur is the most common site of this sarcoma."" 2.) ""The child does not experience pain at the primary tumor site."" 3.) ""Limping, if a weight-bearing limb is affected, is a clinical manifestation."" 4.) ""The symptoms of the disease in the early stage are almost always attributed to normal growing pains."""

"2.) ""The child does not experience pain at the primary tumor site."" (CORRECT ANSWER--Need for further clarification). Osteogenic sarcoma is the most common bone cancer in children. Cancer usually is found in the metaphysis of long bones, especially in the lower extremities, with most tumors occurring in the femur (omit #1). Osteogenic sarcoma is manifested clinically by progressive, insidious, and intermittent pain at the tumor site (correct answer: #2). By the time these children receive medical attention, they may be in considerable pain from the tumor. All options: 1, 3, 4 are accurate regarding osteogenic sarcoma. "

A client is admitted to the hospital for a colon resection and in preparation for surgery the physician orders neomycin. The nurse understands the main reason why this antibiotic is especially useful before colon surgery is because it: "A. Will not affect the kidneys B. Acts systemically without delay C.Has limited absorption from the GI tract. D.Is effective against many different organisms "

"ANSWER:C Because neomycin is limited absorption form the GI tract, it exerts it antibiotic effect on the intestinal mucosa. In preparation of GI surgery, the level of microbial organisms will be reduced."

An adolescent with a history of surgical repair for undescended testes comes to the clinic for a sport physical. Anticipatory guidance for the parents and adolescent would focus on which of the following as most important? "a) the adolescent sterility b) the adolescent future plans c) technique for monthly testicular self-examinations d) need for a lot of psychosocial support"

"Answer C Because the incidence of testicular cancer is increased in adulthood among children who have undescended testes. It is extremely important to teach the adolescent how to perform the testicular self-examination monthly."

"The mother of a 4 year old child brings the child to the clinic and tells the pediatric nurse specialist that the child's abdomen seems to be swollen. During further assessment of the subjective data, the mother tells the nurse that the child has been eating well and that the activity level of the child is unchanged. The nurse, suspecting the possibility of a Wilm's tumor, would avoid which of the following during the physical assessment? "1. Palpating the abdomen for a mass. 2. Assessing the urine for hematuria 3. Monitoring the temperature for presence of fever 4. Monitoring the blood pressure for presence of hypertension"

"Answer: 1 Rationale: Wilm's tumor is the most common intra-abdominal and kidney tumor of childhood. If Wilm's tumor is suspected, the tumor mass should not be palpated by the nurse. Excessive manipulation can cause the seeding of the tumor and spread of cancerous cells. Fever, hematuria, and hypertension are all clinical manifestations of Wilm's tumor."

"Chemotherapy dosage is frequently based on total body surFace area (BSA), so it is important for the nurse to do which of the following before administering chemotherapy? "1. Measure abdominal girth 2. Claculate BMI 3. Ask the client about his/her height and weight 4. Weigh and measure the client on the day of medication administration"

"Answer: 4 To ensure that the client receives optimal doses of chemotherapy, dosing is usually based on the total Body surface area(BSA) which requires accurate height and weight before each med administration. Simply asking the client about height/weight may lead to inaccuracies in determining BSA. Calculating BMI and measuring abdominal girth does not provide the data needed."

A preschool-aged child is to undergo several painful procedures. Which of the following techniques is most-appropriate for the nurse to use in preparing the child? "A. Allow the child to practice injections on a favorite doll. B. Explain the procedure in simple terms. C. Allow a family member to explain the procedure to the child. D. Allow the child to watch an educational video."

"Answer: B Preschoolers have the cognitive ability to understand simple terms. Use of a favorite doll is contraindicated because it is ""part"" of that child and he/she might perceive the doll is experiencing pain."

"When assessing a child with Wilm's tumor, the nurse should keep in mind that it is most important to avoid which of the following? "A. Measuring the child's chest circumference B. Palpating the child's abdomen C. Placing the child in an upright position D. Measuring the child's occipitofrontal circumference"

"Answer: B. The abdomen of the child with Wilm's tumor should not be palpated because of the danger of disseminating tumor cells. Children with Wilm's tumor should always be handled gently and carefully. Other answers. The child's head and chest measuring will not affect Wilm's tumor. Repositioning a child in the upright position may cause more pain to the child, but priority this is not worse than disseminating tumor cells."

A school-age child is being seen in the oncology clinic for possible Hodgkin's disease. During the course of the nursing assessment, which findings would be expected? Select all that apply. "a) fever. b) painless cervical nodes. c) painful cervical nodes. d) poor appetite. e) complaints of night sweats"

"Answers: b and d (complaints of night sweats, painless cervical nodes.) Painless cervical nodes are a hallmark sign of HD. In addition to this, night sweats also are characteristic. Fever, poor appetite, and painful cervical nodes are more characteristic of infection."

Which information would the nurse include when teaching a patient with a discharge instruction to take iron supplements? (Select all that apply. One, some, or all responses may be correct.) 1) "Antacids interfere with the absorption of iron." 2) "Vitamin C interferes with the absorption of iron." 3) "Increase daily fluids because of the risk for diarrhea." 4) "Take iron with orange juice to assist with absorption." 5) "Milk fortified with vitamin D assists absorption of iron more readily."

"Antacids interfere with the absorption of iron." "Take iron with orange juice to assist with absorption."

"Nursing considerations related to the administration of chemotherapeutic drugs include which of the following? "a) Anaphylaxis cannot occur, since the drugs are considered toxic to normal cells. b) Infiltration will not occur unless superficial veins are used for the intravenous infusion. c) Many chemotherapeutic agents are vesicants that can cause severe cellular damage if drug infiltrates. d) Good hand washing is essential when handling chemotherapeutic drugs, but gloves are not necessary."

"CORRECT c. Chemotherapeutic agents can be extremely damaging to cells. Nurses experienced with the administration of vesicant drugs should be responsible for giving these drugs and be prepared to treat extravasations if necessary. a. Anaphylaxis is a possibility with some chemotherapeutic and immunologic agents. b. Infiltration and extravasations are always a risk, especially with peripheral veins. d. Gloves are worn to protect the nurse when handling the drugs, and the hands should be thoroughly washed afterward."

"A child is undergoing remission induction therapy to treat leukemia. Allopurinol is included in the regimen. The main reason for administering allopurinol as part of the client's chemotherapy regimen is to: "a. Prevent metabolic breakdown of xanthine to uric acid b. Prevent uric acid from precipitating in the ureters c. Enhance the production of uric acid to ensure adequate excretion of urine d. Ensure that the chemotherapy doesn't adversely affect the bone marrow"

"CORRECT: Answer A. The massive cell destruction resulting from chemotherapy may place the client at risk for developing renal calculi; adding allopurinol decreases this risk by preventing the breakdown of xanthine to uric acid. Allopurinol doesn't act in the manner described in the other options."

"After teaching the parents of a child newly diagnosed with leukemia about the disease, which of the following descriptions given by the mother best indicates that she understands the nature of leukemia?" "A) ""The disease is an infection resulting in increased white blood cell production."" B) ""The disease is a type of cancer characterized by an increase in immature white blood cells."" C) ""The disease is an inflammation associated with enlargement of the lymph nodes."" D) ""The disease is an allergic disorder involving increased circulating antibodies in the blood."""

"CORRECT: B. Leukemia is a neoplastic, or cancerous, disorder of blood-forming tissues that is characterized by a proliferation of immature white blood cells."

"A 10 year old child with hemophilia A has slipped on the ice and bumped his knee. The nurse should prepare to administer an: "A. injection of factor X B. intravenous infusion of iron C. intravenous infusion of factor VIII D. intramuscular injection of iron using the Z track method"

"CORRECT: C Hemophila refers to a group of bleeding disorders resulting from a deficiency of specific coagulation proteins. the primary meds used are to replace missing clotting factor. Factor VIII will be prescribed intravenously to replace the missing clotting factor and minimize the bleeding,"

"A child is diagnosed with Wilms' tumor. During assessment, the nurse in charge expects to detect: "a. Gross hematuria b. Dysuria c. Nausea and vomiting d. An abdominal mass"

"CORRECT: D The most common sign of Wilms' tumor is a painless, palpable abdominal mass, sometimes accompanied by an increase in abdominal girth. Gross hematuria is uncommon, although microscopic hematuria may be present. Dysuria is not associated with Wilms' tumor. Nausea and vomiting are rare in children with Wilms' tumor."

"Which nursing diagnosis is highest-priority for a child undergoing chemotherapy and experiencing nausea and vomiting? "A. Fluid and Electrolyte Imbalance B. Alterations in Nutrition C. Alterations in Skin Integrity D. Body Image Disturbances"

"Correct Answer: A While all of the nursing diagnoses listed here are important, dehydration and fluid and electrolyte loss secondary to vomiting is the priority for this client."

"The mother of a child diagnosed with a potentially life-threatening form of cancer says to the nurse, ""I don't understand how this could happen to us. We have been so careful to make sure our child is healthy."" Which response by the nurse is most appropriate? "A. ""This must be a difficult time for you and your family. Would you like to talk about how you are feeling?"" B. ""Why do you say that? Do you think that you could have prevented this?"" C. ""You shouldn't feel that you could have prevented the cancer. It is not your fault."" D. ""Many children are diagnosed with cancer. It is not always life-threatening."""

"Correct Answer: A Parents of children diagnosed with cancer require major emotional support, and should be allowed to express their feelings. Prevention and blaming oneself is not supportive, nor is telling the parents that there are many other children with cancer."

"After a client is admitted to the pediatric unit with a diagnosis of acute lymphocytic leukemia, the laboratory test indicates that the client is neutropenic. The nurse should perform which of the following? "a. Advise the client to rest and avoid exertion b. Prevent client exposure to infections c. Monitor the blood pressure frequently d. Observe for increased bruising"

"Correct Answer: B. Prevent client exposure to infections Rationale: Neutropenia is a decreased number of neutrophil cells in the blood which are responsible for the body's defense against infection. Rest and avoid exertion would be related to erythrocytes and oxygen carrying properties. Monitoring the blood pressure, and observing for bruising would be related to platelets and sign and symptoms of bleeding."

A patient who has been told by the health care provider that the cells in a bowel tumor are poorly differentiated asks the nurse what is meant by "poorly differentiated." Which response should the nurse make? "a. ""The cells in your tumor do not look very different from normal bowel cells."" b. ""The tumor cells have DNA that is different from your normal bowel cells."" c. ""Your tumor cells look more like immature fetal cells than normal bowel cells."" d. ""The cells in your tumor have mutated from the normal bowel cells."""

"Correct Answer: C Rationale: An undifferentiated cell has an appearance more like a stem cell or fetal cell and less like the normal cells of the organ or tissue. The DNA in cancer cells is always different from normal cells, whether the cancer cells are well differentiated or not. All tumor cells are mutations form the normal cells of the tissue."

"The most common signs and symptoms of leukemia related to bone marrow involvement are which of the following? "a. Petechiae, fever, fatigue b. Headache, papilledema, irritability c. Muscle wasting, weight loss, fatigue d. Decreased intracranial pressure, psychosis, confusion"

"Correct answer: A Signs of infiltration of the bone marrow are petechiae from lowered platelet count, fever related to infection from the depressed number of effective leukocytes, and fatigue from the anemia. The other options are not signs of bone marrow involvement."

"A child with cancer has the following lab result: WBC 10,000, RBC 5, and plts of 20,000. When planning this child's care, which risk should the nurse consider most significant? "A. Hemorrage B. Anemia C. Infection D. Pain"

"Correct answer: A Hemorrhage The lab values presented all are normal except for the platelet count. Decreases in platelet counts place the child at greatest risk for hemorrhage."

"The goals of cancer treatment are based on the principle that "a. surgery is the single most effective treatment for cancer. b. initial treatment is always directed toward cure of the cancer. c. a combination of treatment modalities is effective for controlling many cancers. d. although cancer cure is rare, quality of life can be increased with treatment modalities. "

"Correct answer: C Rationale: The goals of cancer treatment are cure, control, and palliation. When cure is the goal, treatment is offered that is expected to have the greatest chance of disease eradication. Curative cancer therapy depends on the particular cancer being treated and may involve local therapies (i.e., surgery or irradiation) alone or in combination, with or without periods of adjunctive systemic therapy (i.e., chemotherapy)."

"A preschool-age child undergoing chemotherapy experiences nausea and vomiting. Which of the following would be the best intervention to include in the child's plan of care? "a. Administer tube feedings. b. Offer small, frequent meals. c. Offer fluids only between meals. d. Allow the child to choose what to eat for meals."

"Correct answer: D While all options can be done to encourage nutrition, allowing the preschooler choices meets two issues: nutrition and developmental tasks."

"A 9-year old child with leukemia is in remission and has returned to school. The school nurse calls the mother of the child and tells the mother that a classmate has just been diagnosed with chickenpox. The mother immediately calls the clinic nurse because the leukemic child has never had chickenpox. The appropriate response by the clinic nurse to the mother is: "1. There is no need to be concerned. 2. Bring the child into the clinic for a vaccine. 3. Keep the child out of school for 2 week period. 4. Monitor the child for an elevated temperature, and call the clinic if this happens."

"Correct anwser: 2. Rationale: immunocompromised children are unable to fight varicella adequately. Chickenpox can be deadly to the them. If the child who has not had chickenpox is exposed to someone with varicella, the child should receive varicella zoster immune globulin within 96hrs of exposure. Options 1,3,4, are incorrect because they do nothing to minimize the chances of developing the disease."

"The nurse is admitting a patient who is jaundiced due to pancreatic cancer. The nurse should give the highest priority to which of the following needs? "1. Nutrition 2. Self-image 3. Skin integrity 4. Urinary elimination"

"Correct: 1. 1. profound weight loss and anorexia occur with pancreatic cancer. Correct. 2. jaundiced patients are concerned about how they look, but physiological needs take priority 3. jaundice causes dry skin and pruritis, scratching can lead to skin breakdown 4. urine is dark due to obstructive process, kidney function is not affected"

A nurse is teaching a client about the risk factors associated with colorectal cancer. The nurse determines that further teaching related to the colo-rectal cancer is necessary if the client identifies which of the following as an associated risk factor? "1. Age younger than 50 years 2. History of coloractal polyps 3. Family history of colorectal cancer 4. Chronic inflammatory bowel disease"

"Correct: 1. Colorectal cancer risk factors include age older than 50 years, a family history of the disease, colorectal polyps, and chronic inflammatory bowel disease."

Which condition assessed by the nurse would be an early warning sign of childhood cancer? 1. difficulty swallowing \ 2. nagging cough or hoarseness 3. slight changes in bowel and bladder function 4. swelling, lumps, masses on body

"Correct: 4. Swelling or lumps or masses anywhere on the body are early warning signs whereas difficulty swallowing or cough or hoarseness are signs of cancer in adults. there may be a marked sign in changes to bowel or bladder function, not a slight change"

A nurse is discussing childhood cancer with the parents of a child in an oncology unit. Which statement by the nurse would be the most accurate? "A. ""The most common site for children's cancer is the bone marrow."" B. ""All childhood cancers have a high mortality rate."" C. ""Children with leukemia have a higher survival rate if they are older than 11 when diagnosed."" D. ""The prognosis for children with cancer isn't affected by treatment strategies."""

"Correct: A. Childhood cancers occur most commonly in rapidly growing tissue, especially in the bone marrow. Mortality depends on the time of diagnosis, the type of cancer, and the age at which the child was diagnosed. Children who are diagnosed between the ages of 2 and 9 consistently demonstrate a better prognosis. Treatment strategies are tailored to produce the most favarable prognosis. (NCLEX-RN Questions & Answers, made Incredibly Easy)"

The mother of a 5-year-old child asks the nurse questions regarding the importance of vigilant use of sunscreen. Which information is most important for the nurse to convey to the mother? "a.) Appropriate use of sunscreen decreases the risk of skin cancer. b.) Repeated exposure to the sun causes premature aging of the skin. c.) A child's skin is delicate, and burns easily. d.) In addition to causing skin cancer, repeated sun exposure predisposes the child to other forms of cancer."

"Correct: A. While all of the answer choices are correct, recommending the use of sunscreen to decrease the incidence of skin cancer (a) is the best response."

"The parent of a child undergoing chemotherapy asks the nurse why the child must wear a mask in public places. Which of the following responses by the nurse would be most appropriate? "A) ""Chemotherapy causes dry mouth, and the mask will help contain moisture."" B) ""Chemotherapy decreases immune system function, increasing the risk of acquiring an infection."" C) ""Chemotherapy makes the oral mucous membranes deteriorate and makes them susceptible to infection."" D) ""Chemotherapy kills cancer cells, and your child might spread those cells to others."""

"Correct: B Chemotherapeutic agents decrease the immunity of the child. Proper use of the mask will decrease the chance of acquiring an infection. Cancer is not spread; a mask cannot contain moisture; and unsightly mouth sores are not a medical reason to wear a mask."

What are the needs of the patient with acute lymphocytic leukemia and thrombocytopenia? "(A) to a private room so she will not infect other patients and health care workers (B) to a private room so she will not be infected by other patients and health care workers (C) to a semiprivate room so she will have stimulation during her hospitalization (D to a semiprivate room so she will have the opportunity to express her feelings about her illness "

"Correct: B. a-— poses little or no threat B(CORRECT:)- protects patient from exogenous bacteria, risk for developing infection from others due to depressed WBC count, alters ability to fight infection c-— should be placed in a room alone d-ensure that patient is provided with opportunities to express feelings about illness"

"David, age 15 months, is recovering from surgery to remove Wilms' tumor. Which findings best indicates that the child is free from pain? "a. Decreased appetite b. Increased heart rate c. Decreased urine output d. Increased interest in play"

"Correct: D Answer D. One of the most valuable clues to pain is a behavior change: A child who's pain-free likes to play. A child in pain is less likely to consume food or fluids. An increased heart rate may indicate increased pain; decreased urine output may signify dehydration."

"The postoperative care of a preschool child who has had a brain tumor removed should include which of the following? "a. colorless drainage is to be expected b. analgesics are contraindicated because of altered consciousness c. positioning is on the operative side in the Trendelenberg position d. carefully monitor fluids due to cerebral edema"

"D CORRECT: Because of cerebral edema and the danger of increased intracranial pressure postoperatively, fluids are carefully monitored. A. Colorless drainage may be leakage of cerebrospinal fluid from the incision site. This needs to be reported as soon as possible. B. Analgesics can be used for postoperative pain. C. Child should not be positioned in Trendelenburg position postoperatively."

Which response by the parents of a 6-year-old patient recently diagnosed with hemophilia indicates appropriate knowledge of the child's disease? 1) "We keep our CPR up to date at all times." 2) "We carry an EpiPen with us all the time." 3) "We administer the monthly vitamin B12 shot." 4) "If he bleeds, he will need to have the replacement factor."

"If he bleeds, he will need to have the replacement factor."

Which patient statement would concern the nurse providing care for a patient with hemophilia? 1) "I eat a lot of red meat." 2) "If my pain increases, I can take aspirin." 3) "I understand that my sister could pass this on to her son." 4) "I make sure that I do not participate in any contact sports."

"If my pain increases, I can take aspirin."

Which content would the nurse include when providing discharge instructions for a patient with multiple myeloma? 1) "Obtain and document an apical pulse daily." 2) "Increase dietary calcium intake daily." 3) "Limit fluid intake to 1 L to 2 L/day." 4) "Obtain assistance when moving to prevent fractures."

"Obtain assistance when moving to prevent fractures."

Which of the following can lead to a possible diagnosis of human immunodefi ciency virus (HIV) in a child? Select all that apply. 1. Repeated respiratory infections. 2. Intermittent diarrhea. 3. Excessive weight gain. 4. Irregular heartbeat. 5. Poor weight gain.

1, 2, 5 1. Symptoms of HIV include frequent respiratory infections. The symptoms present based on the underlying cellular immunodefi ciency-related disease. 2. Symptoms of HIV include intermittent diarrhea. The symptoms present based on the underlying cellular immunodeficiency-related disease. 5. Symptoms of HIV include poor weight gain.

A nurse is caring for a 15-year-old who has just been diagnosed with non-Hodgkin lymphoma. Which of the following should the nurse include in teaching the parents about this lymphoma? Select all that apply. 1. The malignancy originates in the lymphoid system. 2. The presence of Reed-Sternberg cells in the biopsy is considered diagnostic. 3. Mediastinal involvement is typical. 4. The disease is diffuse rather than nodular. 5. Treatment includes chemotherapy and radiation.

1, 3, 4, 5 1. Non-Hodgkin disease originates in the lymphoid system. 3. Mediastinal involvement is typical. 4. The disease is diffuse rather than nodular. 5. Treatment includes chemotherapy and radiation.

Which of the following factors need(s) to be included in a teaching plan for a child with sickle cell disease? Select all that apply. 1. The child needs to be taken to a physician when sick. 2. The parent should make sure the child sleeps in an air-conditioned room. 3. Emotional stress should be avoided. 4. It is important to keep the child well hydrated. 5. It is important to make sure the child gets adequate nutrition.

1, 3, 4, 5 1. Seek medical attention for illness to prevent the child from going into a crisis. 3. Stress can cause a depressed immune system, making the child more susceptible to infection and crisis. Parents and children are advised to avoid stress. 4. The child needs good hydration and nutrition to maintain good health. 5. The child needs good hydration and nutrition to maintain good health

Which of the following best describes the action of chemotherapeutic agents used in the treatment of cancer in children? Select all that apply. 1. Suppress the function of normal lymphocytes in the immune system. 2. Are alkylating agents and are cell-specifi c. 3. Cause a replication of DNA and are cell-specifi c. 4. Interrupt cell cycle, thereby causing cell death. 5. Prednisone is a natural hormone.

1, 4, 5 1. All chemotherapy is immunosuppressive, because most childhood cancers affect the immune system. 4. Mitotic inhibitors, such as vincristine (Oncovin), stop cell division but can also damage cells in all phases of the cell cycle. 5. The corticosteroids are natural hormones that can be used to prevent nausea and allergic reactions. They are given with other chemotherapeutic agents.

School-age children with cancer often have a body image disturbance related to hair loss, moon face, or debilitation. Which of the following interventions is (are) most appropriate? Select all that apply. 1. Encourage them to wear a wig similar to their own hairstyle. 2. Emphasize the benefi ts of the therapy they are receiving. 3. Have them play only with other children with cancer. 4. Use diversional techniques to avoid discussing changes in the body because of the chemotherapy. 5. Help them fi nd a "special friend" who understands what they are experiencing.

1, 5 1. Wearing a wig is a good way for the child to keep personal identity despite the loss of hair. 5. Children with cancer and body-image changes oftentimes need assistance in expressing themselves, which a "special friend" can help with.

Which of the following measures should be implemented for a child with von Willebrand disease who has a nosebleed? 1. Apply pressure to the nose for at least 10 minutes. 2. Have the child lie supine and quiet. 3. Avoid packing of the nostrils. 4. Encourage the child to swallow frequently.

1. Applying pressure to the nose may stop the bleeding. In von Willebrand disease, there is an increased tendency to bleed from mucous membranes, leading to nosebleeds commonly from the anterior part of the nasal septum.

The nurse is caring for a child with sickle cell disease who is scheduled to have an exchange transfusion. What information should the nurse teach the family? 1. The procedure is done to prevent further sickling during a vaso-occlusive crisis. 2. The procedure reduces side effects from blood transfusions. 3. The procedure is a routine treatment for sickle cell crisis. 4. Once the child ' s spleen is removed, it is not necessary to do exchange transfusions.

1. Exchange transfusion reduces the number of circulating sickle cells and slows down the cycle of hypoxia, thrombosis, and tissue ischemia.

The parent of a teen with a diagnosis of Hodgkin disease asks what the child ' s prognosis will be with treatment. What information should the nurse give to the parent and child? 1. Clinical staging of Hodgkin disease will determine the treatment; long-term survival for all stages of Hodgkin disease is excellent. 2. There is a considerably better prognosis if the client is diagnosed early and is less than 5 years of age. 3. The prognosis for Hodgkin disease depends on the type of chemotherapy. 4. The only way to obtain a good prognosis is by chemotherapy and bone marrow transplant.

1. Long-term survival for all stages of Hodgkin disease is excellent. Early-stage disease can have a survival rate greater than 90%, with advanced stages having rates between 65% and 75%.

What are the clinical manifestations of non-Hodgkin lymphoma? 1. Basically the same as those in Hodgkin disease. 2. Depends on the anatomical site and extent of involvement. 3. Nausea, vomiting, abdominal pain. 4. Behavior changes, jaundice, dry mouth.

2. The clinical manifestations include symptoms of involvement. Rarely is a single sign or symptom diagnostic. Metastasis to the bone marrow or central nervous system may produce manifestations of leukemia

A child has completed treatment for leukemia and comes to the clinic for a checkup with the parents. The parents express to the nurse that they are glad their child has been cured of cancer and is safe from getting cancer later in life. Which of the following should the nurse consider in responding? 1. Childhood cancer usually instills immunity to all other cancers. 2. Children surviving one cancer are at higher risk for a second cancer. 3. The child may have a remission of the leukemia but is immune to all other cancers. 4. As long as the child continues to take steroids, there will be no other cancers

2. The most devastating late effect of leukemia treatment is development of secondary malignancy.

c

Infants are often not diagnosed with sickle cell anemia until they are 1 year of age. Why? a. usually there are no symptoms until after age 1 b. high intake of fluids from formula prevents sickle cell crises during this age c. fetal hemoglobin is present during the first year of life d. increased hemoglobin and hematocrit amounts compensate during this period

Which of the following is the most effective treatment for pain in a child with sickle cell crisis? Select all that apply. 1. Meperidine (Demerol). 2. Aspirin. 3. Morphine. 4. Behavioral techniques. 5. Acetaminophen (Tylenol) with codeine.

3, 4, 5. 3. Morphine is the drug of choice for a child with sickle cell crises. Usually the child is started on oral doses of acetaminophen (Tylenol) with codeine. When that is not suffi cient to alleviate pain, stronger narcotics are prescribed, such as morphine. Ketorolac (Toradol) may be indicated for short-term use for moderate-severe pain. 4. Behavioral techniques such as positive self-talk, relaxation, distraction, and guided imagery are helpful when pain is occurring. 5. Usually the child is started on oral doses of acetaminophen (Tylenol) with codeine when pain is described as mild to moderate

A 5-year-old is admitted to the hospital with complaints of leg pain and fever. On physical examination, the child is pale and has bruising over various areas of the body. The health-care provider suspects that the child has ALL. The nurse informs the parent that the diagnosis will be confi rmed by which of the following? 1. Lumbar puncture. 2. White blood cell count. 3. Bone marrow aspirate. 4. Bone scan.

3. The diagnostic test that confi rms leukemia is microscopic examination of the bone marrow aspirate.

The nurse is caring for a child who is receiving extensive radiation as part of the treatment for Hodgkin disease. Which intervention should be implemented? 1. Administer pain medication prior to the child ' s going to radiation therapy. 2. Assess the child for neuropathy since this is a common side effect. 3. Provide adequate rest, because the child may experience excessive malaise and lack of energy. 4. Encourage the child to eat a low-protein diet while on radiation therapy.

3. The most common side effect is extensive malaise, which may be from damage to the thyroid gland, causing hypothyroidism

A nurse analyzes the lab values of a child with leukemia who is receiving chemotherapy. The nurse notices that the platelet count is 19,500 cell/mm3. Based on this lab value which intervention would the nurse document in her plan of care. " "1. Monitor closely for signs of infection. 2. Temp every four hours. 3. Isolation precautions 4. Use a small toothbrush for mouth care"

4. **Correct... Rationale: Leukemia is a malignant increase in the number of leukocytes, usually at an immature stage, in the bone marrow. It affects the bone marrow, causing from decreased erythrocytes, infection from neutropenia, and bleeding from decreased platelet production. If the platelet count is les than 20,000 than bleeding precautions need to be taken.

A nurse is teaching a client about the risk factors associated with colorectal cancer. The nurse determines that further teaching related to the colo-rectal cancer is necessary if the client identifies which of the following as an associated risk factor? A. Age younger than 50 years B. History of coloractal polyps C. Family history of colorectal cancer D. Chronic inflammatory bowel disease"

A (Colorectal cancer risk factors include age older than 50 years, a family history of the disease, colorectal polyps, and chronic inflammatory bowel disease.)

A diagnostic workup is being performed on a 1-year-old child with suspected neuroblastoma. The nurse reviews the results of the diagnostic tests and understands that which of the following findings is most specifically related to this type of tumor? A. Elevated vanillylmandelic acid (VMA) urinary levels B. Presence of blast cells in the bone marrow C. Projectile vomiting, usually in the morning D. Postive Babinski's sign"

A (Rationale: Neuroblastoma is a solid tumor found only in children. It arises from neural crest cells that develop into the sympathetic nervous system and the adrenal medulla. Typically, the tumor compresses adjacent normal tissue and organs. Neuroblastoma cells may excrete catecholamines and their metabolites. Urine samples will indicate elevated VMA levels. The presence of blast cells in the bone marrow occurs in leukemia. Projectile vomiting occurring most often in the morning and a positive Babinski's sign are clinical manifestations of a brain tumor.)

A child with cancer has the following lab result: WBC 10,000, RBC 5, and plts of 20,000. When planning this child's care, which risk should the nurse consider most significant? A. Hemorrage B. Anemia C. Infection D. Pain

A (The lab values presented all are normal except for the platelet count. Decreases in platelet counts place the child at greatest risk for hemorrhage.)

A child is undergoing remission induction therapy to treat leukemia. Allopurinol is included in the regimen. The main reason for administering allopurinol as part of the client's chemotherapy regimen is to: a. Prevent metabolic breakdown of xanthine to uric acid b. Prevent uric acid from precipitating in the ureters c. Enhance the production of uric acid to ensure adequate excretion of urine d. Ensure that the chemotherapy doesn't adversely affect the bone marrow"

A (The massive cell destruction resulting from chemotherapy may place the client at risk for developing renal calculi; adding allopurinol decreases this risk by preventing the breakdown of xanthine to uric acid. Allopurinol doesn't act in the manner described in the other options.)

The mother of a 5-year-old child asks the nurse questions regarding the importance of vigilant use of sunscreen. Which information is most important for the nurse to convey to the mother? a.) Appropriate use of sunscreen decreases the risk of skin cancer. b.) Repeated exposure to the sun causes premature aging of the skin. c.) A child's skin is delicate, and burns easily. d.) In addition to causing skin cancer, repeated sun exposure predisposes the child to other forms of cancer.

A (While all of the answer choices are correct, recommending the use of sunscreen to decrease the incidence of skin cancer (a) is the best response.)

a

A 5-year-old child is admitted to the hospital in a sickle cell crisis. The child has been alert and oriented but in severe pain. The nurse notes that the child is complaining of a headache and is having unilateral hemiplegia. What action should the nurse implement? a. Notify the health care provider. b. Place the child on bed rest. c. Administer a dose of hydrocodone (Vicodin). d. Start O2 per the hospitals protocol.

The nurse is preparing to give oral care to a school-age child with mucositis secondary to chemotherapy administered to treat leukemia. Which preparations should the nurse use for oral care on this child (Select all that apply)? a. Chlorhexidine gluconate (Peridex) b. Lemon glycerin swabs c. Antifungal troches (lozenges) d. Lip balm (Aquaphor) e. Hydrogen peroxide

A C D

Which home care instructions should the nurse provide to the parents of a child with acquired immunodeficiency syndrome (AIDS) (Select all that apply)? a. Give supplemental vitamins as prescribed. b. Yearly influenza vaccination should be avoided. c. Administer trimethoprim-sulfamethoxazole (Bactrim) as prescribed. d. Notify the physician if the child develops a cough or congestion. e. Missed doses of antiretroviral medication do not need to be recorded.

A C D

a b e

A child with hemophilia A fell and injured a knee while playing outside. The knee is swollen and painful. Which of the following measures should be taken to stop the bleeding? Select all that apply a. the extremity should be immobilized b. the extremity should be elevated c. warm moist compresses should be applied to decrease pain d. passive ROM exercises should be administered to the extremity e. factor VIII should be administered

a

A child with hemophilia A is scheduled for surgery. What precautions should the nurse institute with this child? a. Handle the child gently when transferring to a cart. b. Caution the child not to brush his teeth before surgery. c. Use tape sparingly on postoperative dressings. d. Do not administer analgesics before surgery.

b

A child with hemophilia presents with a burning sensation in the knee and reluctance to move the body part. The nurse collaborates with the care team to provide factor replacement and a. administer an aspirin containing compound b. institute rest, ice, compression, and elevation (RICE) c. begin physical therapy with active ROM d. initiate skin traction

c

A child with severe anemia requires a unit of red blood cells (RBCs). The nurse explains to the child that the transfusion is necessary for which reason? a. Allow her parents to come visit her. b. Fight the infection that she now has. c. Increase her energy so she will not be so tired. d. Help her body stop bleeding by forming a clot (scab).

b

A child with sickle cell anemia (SCA) develops severe chest and back pain, fever, a cough, and dyspnea. What should be the first action by the nurse? a. Administer 100% oxygen to relieve hypoxia. b. Notify the practitioner because chest syndrome is suspected. c. Infuse intravenous antibiotics as soon as cultures are obtained. d. Give ordered pain medication to relieve symptoms of pain episode.

b d e

A child with sickle cell anemia is admitted in a vaso-occlusive crisis. Which of the following interventions should the nurse expect to see ordered? Select all that apply a. cold compresses to painful joints b. IV fluids started, and oral fluids encouraged c. meperidine ordered every 4 hours for pain d. high calorie, high protein diet e. antibiotics ordered for any existing infection

c

A mother states that she brought her child to the clinic because the 3-year-old girl was not keeping up with her siblings. During physical assessment, the nurse notes that the child has pale skin and conjunctiva and has muscle weakness. The hemoglobin on admission is 6.4 g/dl. After notifying the practitioner of the results, what nursing priority intervention should occur next? a. Reduce environmental stimulation to prevent seizures. b. Have the laboratory repeat the analysis with a new specimen. c. Minimize energy expenditure to decrease cardiac workload. d. Administer intravenous fluids to correct the dehydration.

c d e

A nurse is caring for a 5 year old with sickle cell vaso-occlusive crisis. Which of the following orders should the nurse question? Select all that apply a. position the child for comfort b. apply hot packs to painful areas c. give meperidine (Demerol) 25 mg IV every 4 hours as needed for pain d. restrict oral fluids e. apply oxygen per nasal cannula to keep oxygen saturations above 94%

b

A nurse is preparing to administer iron dextran IM to a school-age child who has iron deficiency anemia. Which of the following actions by the nurse is appropriate? a. administer the dose in the deltoid muscle b. use the Z-track method when administering the dose c. avoid injecting more than 2 mL with each dose d. massage the injection site for 1 minute after administering the dose

a

A nurse is providing teaching about the management of epistaxis to an adolescent. Which of the following positions should the nurse instruct the adolescent to take when experiencing a nosebleed? a. sit up and lean forward b. sit up and tilt the head c. lie in a supine position d. lie in a prone position

The nurse is assessing a child with aplastic anemia. Which of the following would the nurse expect to assess? Select all that apply. A) Ecchymoses B) Tachycardia C) Guaiac-positive stool D) Epistaxis E) Severe pain F) Warm tender joints

A) Ecchymoses B) Tachycardia C) Guaiac-positive stool D) Epistaxis

The nurse is caring for a 13-year-old boy with acute myelogenous leukemia who is experiencing feelings of powerlessness due to the effects of chemotherapy. Which of the following interventions will best help the teen's sense of control? A) Involving the boy in decisions whenever possible B) Acknowledging the boy's feelings of anger with the disease C) Providing realistic expectations of treatments and outcomes D) Recognizing abilities that are unaffected by the disease

A) Involving the boy in decisions whenever possible

The nurse is planning a discussion group for parents with children who have cancer. Which of the following would the nurse include when describing the differences between cancer in children and adults? A) Most childhood cancers affect the tissues rather than organs. B) Childhood cancers are usually localized when found. C) Unlike adult cancers, childhood cancers are less responsive to treatment. D) The majority of childhood cancers can be prevented.

A) Most childhood cancers affect the tissues rather than organs.

When providing care to a child with aplastic anemia, which nursing diagnosis most likely would be the priority? A) Risk for injury B) Imbalanced nutrition, less than body requirements C) Ineffective tissue perfusion D) Impaired gas exchange

A) Risk for injury

A pediatric oncology patient undergoing chemotherapy treatment is refusing to eat despite providing the child's "favorite foods" and allowing for alternate feeding patterns independent of meal time. What etiological theories might account for the child's loss of desire to eat? (Select all that apply.) A. The patient anticipates that he/she will be nauseous and/or vomit as part of the treatment sequence. B. The patient is refusing to eat in an attempt to gain control over his/her surroundings. C. The "correct" food has just not been found and more food selections should be offered. D. The patient is experiencing symptoms of depression. E. The patient is refusing to eat because his/her parents did not make the food.

A, B, D Anorexia and/or a refusal to eat sometimes accompanies chemotherapy interventions in patients. Thus, even in the context of being offered "favorite foods," the child may not want to eat. Theories proposed for this persistent anorexia and/or refusal include but are not limited to: possible depression, attempts at control, gaining control over one's environment, a conditioned response reflecting aversion during treatment and/or stress.

Which findings are not consistent with tumor lysis syndrome? A. Hypercalcemia and hyperkalemia B. Hypochloremia and hypokalemia C. Hyponatremia and hyperphosphatemia D. Hyperuricemia and hyperkalemia E. Hypercalcemia and hypokalemia

A, B, E The hallmark characteristics of tumor lysis syndrome are: hyperuricemia, hypocalcemia, hyperphosphatemia, and hyperkalemia.

In taking care of a pediatric oncology patient, which diagnostic finding would indicate a critical concern for the development of infection? A. Absolute neutrophil count of 250 mm3 B. Temperature of 99.2 degrees Fahrenheit C. White blood cell count 7,000 mm3 D. Platelet count 100,000 mm3

A. An absolute neutrophil count of less than 500 mm3 is of critical concern as it indicates the potential for overwhelming infection. None of the other measurement parameters are reflective of this fact.

The pediatric nurse is performing a well child assessment. Which finding if noted would require further investigation? A. Palpation of an abdominal mass without pain expression. B. No report of pain or tenderness in arms or legs. C. Buccal mucosa pink and intact. D. Grey appearance of tympanic membrane on otoscopic exam.

A. In a pediatric patient detection of an abdominal mass, regardless of pain expression requires further diagnostic work up as it may be evidence of Wilm's tumor. All of the other findings represent normal variations and as such do not require further investigation.

The pediatric nurse is performing a well child assessment. Which finding if noted would require further investigation? A. Palpation of an abdominal mass without pain expression. B. No report of pain or tenderness in arms or legs. C. Buccal mucosa pink and intact. D. Grey appearance of tympanic membrane on otoscopic exam. E. Bruises observed following light touch to the extremities. F. Report of a headache

A. In a pediatric patient detection of an abdominal mass, regardless of pain expression requires further diagnostic work up as it may be evidence of Wilm's tumor. All of the other findings represent normal variations and as such do not require further investigation.

The pediatric nurse practitioner provides primary care for a 30monthold child who has sickle cell anemia who has had one dose of 23valent pneumococcal vaccine. Which is an appropriate action for health maintenance in this child? A. Administer an initial meningococcal vaccine. B. Begin folic acid dietary supplementation. C. Decrease the dose of penicillin V prophylaxis. D. Give a second dose of 23valent pneumococcal vaccine.

A. Administer an initial meningococcal vaccine.

A 2yearold child who has SCA comes to the clinic with a cough and a fever of 101.5°C. The child currently takes penicillin V prophylaxis 125 mg orally twice daily. What will the primary care pediatric nurse practitioner do? A. Admit the child to the hospital to evaluate for sepsis. B. Give intravenous fluids and antibiotics in clinic. C. Increase the penicillin V dose to 250 mg. D. Order a chest radiograph to rule out pneumonia.

A. Admit the child to the hospital to evaluate for sepsis.

. The primary care pediatric nurse practitioner is managing care for a child diagnosed with irondeficiency anemia who had an initial hemoglobin of 8.8 g/dL and hematocrit of 32% who has been receiving ferrous sulfate as 3 mg/kg/day of elemental iron for 4 weeks. The child's current lab work reveals elevations in Hgb/Hct and reticulocytes with a hemoglobin of 10.5 g/dL and a hematocrit of 36%. What is the next step in management of this patient? A. Continue the current dose of ferrous sulfate and recheck labs in 1 to 2 months. B. Discontinue the supplemental iron and encourage an ironenriched diet. C. Increase the ferrous sulfate dose to 4 to 6 mg/kg/day of elemental iron. D. Refer the child to a pediatric hematologist to further evaluate the anemia.

A. Continue the current dose of ferrous sulfate and recheck labs in 1 to 2 months.

5. The primary care pediatric nurse practitioner performs a well baby examination on a 4monthold infant who is exclusively breastfed and whose mother plans to introduce only small amounts of fruits and vegetables in addition to breastfeeding. To ensure that the infant gets adequate amounts of iron, what will the nurse practitioner recommend? A. Elemental iron supplementation of 1 mg/kg/day until cereals are added B. Elemental iron supplementation of 3 mg/kg/day for the duration of breastfeeding C. Monitoring the infant's hemoglobin and hematocrit at every wellbaby checkup D. Offering ironfortified formula to ensure adequate iron intake

A. Elemental iron supplementation of 1 mg/kg/day until cereals are added

b

What is the cause for the signs and symptoms when hemoglobin falls sufficiently to produce clinical manifestations? a. Phagocytosis b. Tissue hypoxia c. Pulmonary hypertension d. Depressed bone marrow

3. The primary care pediatric nurse practitioner evaluates a 5yearold child who presents with pallor and obtains labs revealing a hemoglobin of 8.5 g/dL and a hematocrit of 31%. How will the nurse practitioner manage this patient? A. Prescribe elemental iron and recheck labs in 1 month. B. Reassure the parent that this represents mild anemia. C. Recommend a diet high in ironrich foods. D. Refer to a hematologist for further evaluation.

A. Prescribe elemental iron and recheck labs in 1 month.

The primary care pediatric nurse practitioner is examining a 5yearold child who has had recurrent fevers, bone pain, and a recent loss of weight. The physical exam reveals scattered petechiae, lymphadenopathy, and bruising. A complete blood count shows thrombocytopenia, anemia, and an elevated white cell blood count. The nurse practitioner will refer this child to a specialist for A. bone marrow biopsy. B. corticosteroids and IVIG. C. hemoglobin electrophoresis. D. immunoglobulin testing.

A. bone marrow biopsy.

Nursing considerations related to the administration of chemotherapeutic drugs include: A. many chemotherapeutic agents are vesicants that can cause severe cellular damage if the drug infiltrates. B. good hand washing is essential when handling chemotherapeutic drugs, but gloves are not necessary. C. infiltration will not occur unless superficial veins are used for the intravenous infusion. D. anaphylaxis cannot occur because the drugs are considered toxic to normal cells.

A. many chemotherapeutic agents are vesicants that can cause severe cellular damage if the drug infiltrates. Chemotherapeutic agents can be extremely damaging to cells. Nurses experienced with the administration of vesicant drugs should be responsible for giving these drugs and be prepared to treat extravasations if necessary. Gloves are worn to protect the nurse when handling the drugs, and the hands should be thoroughly washed afterward. Infiltration and extravasations are always a risk, especially with peripheral veins. Anaphylaxis is a possibility with some chemotherapeutic and immunologic agents.

A poor prognosis following surgical treatment for operable cancers is associated with: A. when there is evidence of metastasis. B. presence of postoperative nausea. C. the amount of pain medication that the patient takes in the first 24 hours postsurgery. D. if the tumor is encapsulated and localized.

A. when there is evidence of metastasis. Poor prognosis following surgical treatment for operable cancers is associated with cancers that have metastasized. Presence of postoperative nausea and/or the amount of pain medication provided during the first 24 hours postoperatively do not correlate with a poor prognosis but rather are dependent on individual tolerance and perception of nausea and pain. If a tumor is encapsulated and localized, this is associated with an easier surgical procedure and a better health outcome.

In recent years the use of _____________ stem cell transplantation has become the accepted therapy for the treatment of several hematologic and oncologic disorders.

ANS: hematopoietic HSCT allows extremely high doses of chemotherapy, with or without radiation, to be given without regard for bone marrow recovery because hematopoiesis will be restored through transplantation. Stem cells are harvested from bone marrow, peripheral blood, and umbilical cord blood. HSCT is often used interchangeably with bone marrow transplantation in the clinical setting.

15. The mother of a child with hemophilia asks the nurse how long her child will need to be treated for hemophilia. What is the best response to this question? a. "Hemophilia is a lifelong blood disorder." b. "There is a 25% chance that your child will have spontaneous remission and treatment will no longer be necessary." c. "Treatment is indicated until after your child has progressed through the toddler years." d. "It is unlikely that your child will need to be treated for his hemophilia because your first child does not have the disease."

ANS: A A Hemophilia is a lifelong hereditary blood disorder with no cure. Prevention by avoiding activities that induce bleeding and by treatment is lifelong. The management of hemophilia is highly individual and depends on the severity of the illness. B This is an untrue statement. Hemophilia is a lifelong hereditary blood disorder with no cure. Treatment is lifelong. C This is an untrue statement. Hemophilia is a lifelong hereditary blood disorder with no cure. Treatment is lifelong. D Because hemophilia has an X chromosome-linked recessive inheritance, there is a risk with each pregnancy that a child will either have the disease or be a carrier. Hemophilia is a life-long hereditary blood disorder with no cure. Treatment is lifelong.

13. What is the priority nursing intervention for a child hospitalized with hemarthrosis resulting from hemophilia? a. Immobilization and elevation of the affected joint b. Administration of acetaminophen for pain relief c. Assessment of the child's response to hospitalization d. Assessment of the impact of hospitalization on the family system

ANS: A A Immobilization and elevation of the joint will prevent further injury until bleeding is resolved. B Although acetaminophen may help with pain associated with the treatment of hemarthrosis, it is not the priority nursing intervention. C Assessment of a child's response to hospitalization is relevant to all hospitalized children; however, in this situation, psychosocial concerns are secondary to physiologic concerns. A priority nursing concern for this child is the management of hemarthrosis. D Assessing the impact of hospitalization on the family system is relevant to all hospitalized children, but it is not the priority in this situation.

19. What is a priority intervention in planning care for the child with disseminated intravascular coagulation (DIC)? a. Hospitalization at the first sign of bleeding b. Teaching the child relaxation techniques for pain control c. Management in the intensive care unit d. Provision of adequate hydration to prevent complications

ANS: C A DIC typically develops in a child who is already hospitalized. B Relaxation techniques and pain control are not high priorities for the child with DIC. C The child with DIC is seriously ill and needs to be monitored in an intensive care unit. D Hydration is not the major concern for the child with DIC.

2. An assessment of a 7-month-old infant with a hemoglobin level of 6.5 mg/dL is likely to reveal an infant who is a. Lethargic, pale, and irritable b. Thin, energetic, and sleeps little c. Anorexic, vomiting, and has watery stools d. Flushed, fussy, and tired

ANS: A A Pallor, lethargy, irritability, and tachycardia are clinical manifestations of iron deficiency anemia. A child with a hemoglobin level of 6.5 mg/dL has anemia. B A child with a hemoglobin level of 6.5 mg/dL has anemia. Infants with iron deficiency anemia are not typically thin and energetic but do tend to sleep a lot. C A child with a hemoglobin level of 6.5 mg/dL has anemia. Gastrointestinal symptoms are not clinical manifestations associated with iron deficiency anemia. D A child with a hemoglobin level of 6.5 mg/dL has anemia. Although the infant with iron deficiency anemia may be tired and fussy, pallor, rather than a flushed appearance, is characteristic of a low hemoglobin level.

Which nursing intervention should not be included in the postoperative plan of care for a child undergoing surgery for a brain tumor? a. Place the child in Trendelenburg position. b. Perform neurologic assessments. c. Assess dressings for drainage. d. Monitor temperature.

ANS: A Feedback A The child is never placed in the Trendelenburg position because it increases intracranial pressure and the risk of bleeding. B Increased intracranial pressure is a risk in the postoperative period. The nurse would assess the child's neurologic status frequently. C Hemorrhage is a risk in the postoperative period. The child's dressing would be inspected frequently for bleeding. D Temperature is monitored closely because the child is at risk for infection in the postoperative period.

What should the nurse recognize as symptoms of a brain tumor in a school-age child for whom she is caring? Select all that apply. a. Blurred vision b. Increased head circumference c. Vomiting when getting out of bed d. Intermittent headache e. Declining academic performance

ANS: A, C, D, E Feedback Correct Visual changes such as nystagmus, diplopia, and strabismus are manifestations of a brain tumor. The change in position on awakening causes an increase in intracranial pressure, which is manifested as vomiting. Vomiting on awakening is considered a hallmark symptom of a brain tumor. Increased intracranial pressure resulting from a brain tumor is manifested as a headache. School-age children may exhibit declining academic performance, fatigue, personality changes, and symptoms of vague, intermittent headache. Other symptoms may include seizures or focal neurologic deficits. Incorrect Manifestations of brain tumors vary with tumor location and the child's age and development. Infants with brain tumors may have increased head circumference with a bulging fontanel. School-age children have closed fontanels and therefore their head circumferences do not increase with brain tumors.

7. What are the nursing priorities for a child with sickle cell disease in vaso-occlusive crisis? a. Administration of antibiotics and nebulizer treatments b. Hydration and pain management c. Blood transfusions and an increased calorie diet d. School work and diversion

ANS: B A Antibiotics may be given prophylactically. Oxygen therapy rather than nebulizer treatments is used to prevent further sickling. B Hydration and pain management decrease the cells' oxygen demands and prevent sickling. C Although blood transfusions and increased calories may be indicated, they are not primary considerations for vaso-occlusive crisis. D School work and diversion are not major considerations when the child is in a vaso-occlusive crisis

20. What is the nurse's best response to a parent with questions about how her child's blood disorder will be treated? a. "Your child may be able to receive home care." b. "What did the physician tell you?" c. "Blood diseases are transient, so there is no need to worry." d. "Your child will be tired for awhile and then be back to her old self."

ANS: B A Treatment depends on the child's condition and the type of blood disorder. Although it is possible that the child could be treated in the home, the child may need to be treated as an outpatient or in the hospital. It is best to first assess what the parent has been told by the physician. B Providing the parent an opportunity to express what she was told by the physician allows the nurse to assess the parent's understanding and provide further information. C Minimizing the parent's concern is inappropriate. D The nurse needs to assess the parent's knowledge before teaching about the disease.

A child with non-Hodgkin lymphoma will be starting chemotherapy. What intervention is initiated before chemotherapy to prevent tumor lysis syndrome? a. Insertion of a central venous catheter b. Intravenous (IV) hydration containing sodium bicarbonate c. Placement of an externalized ventriculoperitoneal (VP) shunt d. Administration of pneumococcal and Haemophilus influenzae type B vaccines

ANS: B Feedback A A central venous catheter is placed to assist in delivering chemotherapy. B Intensive hydration with an IV fluid containing bicarbonate alkalinizes the urine to help prevent the formation of uric acid crystals, which damage the kidney. C An externalized VP shunt may be placed to relieve intracranial pressure caused by a brain tumor. D If a splenectomy is necessary for a child with Hodgkin disease, the pneumococcal and Haemophilus influenzae vaccines are administered before the surgery.

The nurse understands that the type of precautions needed for children receiving chemotherapy is based on which actions of chemotherapeutic agents? a. Gastrointestinal upset b. Bone marrow suppression c. Decreased creatinine level d. Alopecia

ANS: B Feedback A Although gastrointestinal upset may be an adverse effect of chemotherapy, it is not caused by all chemotherapeutic agents. No special precautions are instituted for gastrointestinal upset. B Chemotherapy agents cause bone marrow suppression, which creates the need to institute precautions related to reduced white blood cell, red blood cell, and platelet counts. These precautions focus on preventing infection and bleeding. C A decreased creatinine level is consistent with renal pathologic conditions, not chemotherapy. D Not all chemotherapeutic agents cause alopecia. No precautions are taken to prevent alopecia.

What is an appropriate nursing action before surgery when caring for a child diagnosed with a Wilms' tumor? a. Limit fluid intake. b. Do not palpate the abdomen. c. Force oral fluids. d. Palpate the abdomen every 4 hours.

ANS: B Feedback A Fluids are not routinely limited in a child with a Wilms' tumor. However, intake and output are important because of the kidney involvement. B Excessive manipulation of the tumor area can cause seeding of the tumor and spread of the malignant cells. C Fluids are not forced on a child with a Wilms' tumor. Normal intake for age is usually maintained. D The abdomen of a child with a Wilms' tumor should never be palpated because of the danger of seeding the tumor and spreading malignant cells.

What is a priority nursing diagnosis for the 4-year-old child newly diagnosed with leukemia? a. Ineffective Breathing Pattern related to mediastinal disease b. Risk for Infection related to immunosuppressed state c. Disturbed Body Image related to alopecia d. Impaired Skin Integrity related to radiation therapy

ANS: B Feedback A This nursing diagnosis applies to a child with non-Hodgkin lymphoma or any cancer involving the chest area. B Leukemia is characterized by the proliferation of immature white blood cells, which lack the ability to fight infection. C This is a nursing diagnosis related to chemotherapy, but it is not of the highest priority. Not all children have a body image disturbance as a result of alopecia, especially not preschoolers. This would be of more concern to an adolescent. D Radiation therapy is not a treatment for leukemia.

A child with a brain tumor is undergoing radiation therapy. What should the nurse include in the discharge instructions to the child's parents? (Select all that apply.) a. Apply over-the-counter creams to the area daily. b. Avoid excessive skin exposure to the sun. c. Use a washcloth when cleaning the area receiving radiation. d. Plan for adequate rest periods for the child. e. A darkening of the skin receiving radiation is expected.

ANS: B, D, E Feedback Correct: Children receiving cranial radiation are particularly affected by fatigue and an increased need for sleep during and shortly after completion of the course of radiation. Skin damage can include changes in pigmentation (darkening), redness, peeling, and increased sensitivity. Incorrect: Extra care must be taken to avoid excessive skin exposure to heat, sunlight, friction (such as rubbing with a towel or washcloth), and creams or moisturizers. Only topical creams and moisturizers prescribed by the radiation oncologist should be applied to the radiated skin.

A child with a brain tumor is undergoing radiation therapy. What should the nurse include in the discharge instructions to the child's parents? Select all that apply. a. Apply over-the-counter creams to the area daily. b. Avoid excessive skin exposure to the sun. c. Use a washcloth when cleaning the area receiving radiation. d. Plan for adequate rest periods for the child. e. A darkening of the skin receiving radiation is expected.

ANS: B, D, E Feedback Correct: Children receiving cranial radiation are particularly affected by fatigue and an increased need for sleep during and shortly after completion of the course of radiation. Skin damage can include changes in pigmentation (darkening), redness, peeling, and increased sensitivity. Incorrect: Extra care must be taken to avoid excessive skin exposure to heat, sunlight, friction (such as rubbing with a towel or washcloth), and creams or moisturizers. Only topical creams and moisturizers prescribed by the radiation oncologist should be applied to the radiated skin.

12. Which statement best describes beta-thalassemia major (Cooley anemia)? a. All formed elements of the blood are depressed. b. Inadequate numbers of red blood cells are present. c. Increased incidence occurs in families of Mediterranean extraction. d. Increased incidence occurs in persons of West African descent.

ANS: C A An overproduction of red cells occurs. Although numerous, the red cells are relatively unstable. B An overproduction of red cells occurs. Although numerous, the red cells are relatively unstable. C Individuals who live near the Mediterranean Sea and their descendants have the highest incidence of thalassemia. D Sickle cell disease is common in blacks of West African descent.

8. What describes the pathologic changes of sickle cell anemia? a. Sickle-shaped cells carry excess oxygen. b. Sickle-shaped cells decrease blood viscosity. c. Increased red blood cell destruction occurs. d. Decreased red blood cell destruction occurs.

ANS: C A Sickled red cells have decreased oxygen-carrying capacity and transform into the sickle shape in conditions of low oxygen tension. B When the sickle cells change shape, they increase the viscosity in the area where they are involved in the microcirculation. C The clinical features of sickle cell anemia are primarily the result of increased red blood cell destruction and obstruction caused by the sickle-shaped red blood cells. D Increased red blood cell destruction occurs.

14. What is descriptive of most cases of hemophilia? a. Autosomal dominant disorder causing deficiency is a factor involved in the blood-clotting reaction b. X-linked recessive inherited disorder causing deficiency of platelets and prolonged bleeding c. X-linked recessive inherited disorder in which a blood-clotting factor is deficient d. Y-linked recessive inherited disorder in which the red blood cells become moon shaped

ANS: C A The inheritance pattern is X-linked recessive. B The disorder involves coagulation factors, not platelets. C The inheritance pattern in 80% of all of the cases of hemophilia is X-linked recessive. The two most common forms of the disorder are factor VIII deficiency, hemophilia A or classic hemophilia; and factor IX deficiency, hemophilia B or Christmas disease. D The disorder does not involve red cells or the Y chromosome.

A nurse determines that parents understood the teaching from the pediatric oncologist if the parents indicate that which test confirms the diagnosis of leukemia in children? a. Complete blood cell count (CBC) b. Lumbar puncture c. Bone marrow biopsy d. Computed tomography (CT) scan

ANS: C Feedback A A CBC may show blast cells that would raise suspicion of leukemia. It is not a confirming diagnostic study. B A lumbar puncture is done to check for central nervous system involvement in the child who has been diagnosed with leukemia. C The confirming test for leukemia is microscopic examination of bone marrow obtained by bone marrow aspiration and biopsy. D A CT scan may be done to check for bone involvement in the child with leukemia. It does not confirm a diagnosis.

The nurse should base a response to a parent's question about the prognosis of acute leukemia (ALL) on the knowledge that: a. Leukemia is a fatal disease, although chemotherapy provides increasingly longer periods of remission. b. Research to find a cure for childhood cancers is very active. c. The majority of children go into remission and remain symptom free when treatment is completed. d. It usually takes several months of chemotherapy to achieve a remission.

ANS: C Feedback A With the majority of children surviving 5 years or longer, it is inappropriate to refer to leukemia as a fatal disease. B This statement is true, but it does not address the parent's concern. C Children diagnosed with the most common form of leukemia, ALL, can almost always achieve remission, with a 5-year disease-free survival rate approaching 85%. D About 95% of children achieve remission within the first month of chemotherapy. If a significant number of blast cells are still present in the bone marrow after a month of chemotherapy, a new and stronger regimen is begun.

A nurse determines that parents understood the teaching from the pediatric oncologist if the parents indicate that which test confirms the diagnosis of leukemia in children? a. Complete blood cell count (CBC) b. Lumbar puncture c. Bone marrow biopsy d. Computed tomography (CT) scan

ANS: C Feedback A A CBC may show blast cells that would raise suspicion of leukemia. It is not a confirming diagnostic study. B A lumbar puncture is done to check for central nervous system involvement in the child who has been diagnosed with leukemia. C The confirming test for leukemia is microscopic examination of bone marrow obtained by bone marrow aspiration and biopsy. D A CT scan may be done to check for bone involvement in the child with leukemia. It does not confirm a diagnosis.

A syndrome that leads to the deposition of platelets and fibrinogen plugs in the vasculature and the simultaneous depletion of platelets and clotting factor proteins is commonly known as DIC or _____________________.

ANS: disseminated intravascular coagulation The pathophysiology of DIC is complicated and not easily understood because both extreme bleeding and clotting occur at the same time.

The nurse is aware that an abdominal mass found in a 10-month-old infant corresponds with which childhood cancer? a. Osteogenic sarcoma b. Rhabdomyosarcoma c. Neuroblastoma d. Non-Hodgkin lymphoma

ANS: C Feedback A Osteogenic sarcoma is a bone tumor. Bone tumors typically affect older children. B Rhabdomyosarcoma is a malignancy of muscle, or striated tissue. It occurs most often in the periorbital area, in the head and neck in younger children, or in the trunk and extremities in older children. C Neuroblastoma is found exclusively in infants and children. In most cases of neuroblastoma, a primary abdominal mass and protuberant, firm abdomen are present. D Non-Hodgkin lymphoma is a neoplasm of lymphoid cells. Painless, enlarged lymph nodes are found in the cervical or axillary region. Abdominal signs and symptoms do not include a mass.

What is the nurse's best response to a mother whose child has a diagnosis of acute lymphoblastic leukemia and is expressing guilt about not having responded sooner to her boy's symptoms? a. "You should always call the physician when your child has a change in what is normal for him." b. "It is better to be safe than sorry." c. "It is not uncommon for parents not to notice subtle changes in their children's health." d. "I hope this delay does not affect the treatment plan."

ANS: C Feedback A The goal is to relieve the mother's guilt and build trust so that she can talk about her feelings. This statement will only reinforce her guilt. B This response is flippant and reinforces that the mother was negligent, which will only increase her guilt. C This statement minimizes the role the mother played in not seeking early medical attention. It also displays empathy, which helps to build trust, thereby enabling the mother to talk about her feelings. Identifying concerns and clarifying misconceptions will help families cope with the stress of chronic illness. D This statement shows a total lack of empathy and would increase the mother's feelings of guilt.

What should the nurse teach parents about oral hygiene for the child receiving chemotherapy? a. Brush the teeth briskly to remove bacteria. b. Use a mouthwash that contains alcohol. c. Inspect the child's mouth daily for ulcers. d. Perform oral hygiene twice a day.

ANS: C Feedback A The teeth should be brushed with a soft-bristled toothbrush. Excessive force with brushing should be avoided because delicate tissue could be broken, causing infection or bleeding. B Mouthwashes containing alcohol may be drying to oral mucosa, thus breaking down the protective barrier of the skin. C The child's mouth is inspected regularly for ulcers. At the first sign of ulceration, an antifungal drug is initiated. D Oral hygiene should be performed four times a day.

The nurse should base a response to a parent's question about the prognosis of acute lymphoblastic leukemia (ALL) on the knowledge that a. Leukemia is a fatal disease, although chemotherapy provides increasingly longer periods of remission. b. Research to find a cure for childhood cancers is very active. c. The majority of children go into remission and remain symptom free when treatment is completed. d. It usually takes several months of chemotherapy to achieve a remission.

ANS: C Feedback A With the majority of children surviving 5 years or longer, it is inappropriate to refer to leukemia as a fatal disease. B This statement is true, but it does not address the parent's concern. C Children diagnosed with the most common form of leukemia, ALL, can almost always achieve remission, with a 5-year disease-free survival rate approaching 85%. D About 95% of children achieve remission within the first month of chemotherapy. If a significant number of blast cells are still present in the bone marrow after a month of chemotherapy, a new and stronger regimen is begun.

A nurse is teaching home care instructions to parents of a child with sickle cell disease. Which instructions should the nurse include? Select all that apply. a. Limit fluid intake. b. Administer aspirin for fever. c. Administer penicillin as ordered. d. Avoid cold and extreme heat. e. Provide for adequate rest periods.

ANS: C, D, E Correct Parents should be taught to avoid cold, which can increase sickling, and extreme heat, which can cause dehydration. Adequate rest periods should be provided. Penicillin should be administered daily as ordered. Incorrect: The use of aspirin should be avoided; acetaminophen or ibuprofen should be used as an alternative. Fluids should be encouraged and an increase in fluid intake is encouraged in hot weather or when there are other risks for dehydration.

3. What action is not appropriate for a 14-month-old child with iron deficiency anemia? a. Decreasing the infant's daily milk intake to 24 oz or less b. Giving oral iron supplements between meals with orange juice c. Including apricots, dark-green leafy vegetables, and egg yolk in the infant's diet d. Allowing the infant to drink the iron supplement from a small medicine cup

ANS: D A A daily milk intake in toddlers of less than 24 oz will encourage the consumption of iron-rich solid foods. B Because food interferes with the absorption of iron, iron supplements are taken between meals. Administering this medication with foods rich in vitamin C facilitates absorption of iron. C Apricots, dark-green leafy vegetables, and egg yolks are rich sources of iron. Other iron-rich foods include liver, dried beans, Cream of Wheat, iron-fortified cereal, and prunes. D Iron supplements should be administered through a straw or by a medicine dropper placed at the back of the mouth because iron temporarily stains the teeth.

9. Which clinical manifestation should the nurse expect when a child with sickle cell anemia experiences an acute vaso-occlusive crisis? a. Circulatory collapse b. Cardiomegaly, systolic murmurs c. Hepatomegaly, intrahepatic cholestasis d. Painful swelling of hands and feet; painful joints

ANS: D A Circulatory collapse results from sequestration crises. B Cardiomegaly, systolic murmurs, hepatomegaly, and intrahepatic cholestasis result from chronic vaso-occlusive phenomena. C Cardiomegaly, systolic murmurs, hepatomegaly, and intrahepatic cholestasis result from chronic vaso-occlusive phenomena. D A vaso-occlusive crisis is characterized by severe pain in the area of involvement. If in the extremities, painful swelling of the hands and feet is seen; if in the abdomen, severe pain resembles that of acute surgical abdomen; and if in the head, stroke and visual disturbances occur.

4. An accurate description of anemia is a. Increased blood viscosity b. Depressed hematopoietic system c. Presence of abnormal hemoglobin d. Decreased oxygen-carrying capacity of blood

ANS: D A Increased blood viscosity is usually a function of too many cells or of dehydration, not of anemia. B A depressed hematopoietic system or abnormal hemoglobin can contribute to anemia, but the definition is dependent on the deceased oxygen-carrying capacity of the blood. C A depressed hematopoietic system or abnormal hemoglobin can contribute to anemia, but the definition is dependent on the decreased oxygen-carrying capacity of the blood. D Anemia is a condition in which the number of red blood cells or hemoglobin concentration is reduced below the normal values for age. This results in a decreased oxygen-carrying capacity of blood.

10. What should the discharge plan for a school-age child with sickle cell disease include? a. Restricting the child's participation in outside activities b. Administering aspirin for pain or fever c. Limiting the child's interaction with peers d. Administering penicillin daily as ordered

ANS: D A Sickle cell disease does not prohibit the child from outdoor play. Active and passive exercises help promote circulation. B Aspirin use should be avoided. Acetaminophen or ibuprofen should be administered for fever or pain. C The child needs to interact with peers to meet his developmental needs. D Children with sickle cell disease are at a high risk for pneumococcal infections and should receive long-term penicillin therapy and preventive immunizations.

17. A child who has been in good health has a platelet count of 45,000/mm3, petechiae, and excessive bruising that covers the body. The nurse is aware that these signs are clinical manifestations of a. Erythroblastopenia b. von Willebrand disease c. Hemophilia d. Immune thrombocytopenic purpura (ITP)

ANS: D A The clinical manifestations of erythroblastopenia are pallor, lethargy, headache, fainting, and a history of upper respiratory infection. B The clinical manifestations of von Willebrand disease are bleeding from the gums or nose, prolonged bleeding from cuts, and excessive bleeding after surgery or trauma. C Bleeding is the clinical manifestation of hemophilia and results from a deficiency of normal factor activity necessary to produce blood clotting. D Excessive bruising and petechiae, especially involving the mucous membranes and gums in a child who is otherwise healthy, are the clinical manifestations of ITP, resulting from decreased platelets. The etiology of ITP is unknown, but it is considered to be an autoimmune process.

Which statement, if made by a nurse to the parents of a child with leukemia, indicates an understanding of teaching related to home care associated with the disease? a. "Your son's blood pressure must be taken daily while he is on chemotherapy." b. "Limit your son's fluid intake just in case he has central nervous system involvement." c. "Your son must receive all of his immunizations in a timely manner." d. "Your son's temperature should be taken frequently."

ANS: D Feedback A The child's temperature must be taken daily because of the risk for infection, but it is not necessary to take a blood pressure daily. B Fluid is never withheld as a precaution against increased intracranial pressure. If a child had confirmed CNS involvement with increased intracranial pressure, this intervention might be more appropriate. C Children who are immunosuppressed should not receive any live virus vaccines. D An elevated temperature may be the only sign of an infection in an immunosuppressed child. Parents should be instructed to monitor their child's temperature as often as necessary.

What is the most appropriate nursing action when the nurse notes a reddened area on the forearm of a neutropenic child with leukemia? a. Massage the area. b. Turn the child more frequently. c. Document the finding and continue to observe the area. d. Notify the physician.

ANS: D Feedback A In a child with neutropenia, a reddened area may be the only sign of an infection. The area should never be massaged. B The forearm is not a typical pressure area; therefore the likelihood of the redness being related to pressure is very small. C The observation should be documented, but because it may be a sign of an infection and immunosuppression, the physician must also be notified. D Skin is the first line of defense against infection. Any signs of infection in a child who is immunosuppressed must be reported to the physician. When a child is neutropenic, pus may not be produced and the only sign of infection may be redness.

Which statement, if made by a nurse to the parents of a child with leukemia, indicates an understanding of teaching related to home care associated with the disease? a. "Your son's blood pressure must be taken daily while he is on chemotherapy." b. "Limit your son's fluid intake just in case he has central nervous system involvement." c. "Your son must receive all of his immunizations in a timely manner." d. "Your son's temperature should be taken frequently."

ANS: D Feedback A The child's temperature must be taken daily because of the risk for infection, but it is not necessary to take a blood pressure daily. B Fluid is never withheld as a precaution against increased intracranial pressure. If a child had confirmed CNS involvement with increased intracranial pressure, this intervention might be more appropriate. C Children who are immunosuppressed should not receive any live virus vaccines. D An elevated temperature may be the only sign of an infection in an immunosuppressed child. Parents should be instructed to monitor their child's temperature as often as necessary.

The nurse is evaluating lab results to determine if her patient is experiencing a diagnosis of DIC. The nurse should anticipate the following results: increased red blood cell count, low platelet counts, and an increased fibrinogen level. Is this statement true or false?

ANS: F The results indicate a decreased red blood cell count, low platelets, red blood cell fragments, prolonged prothrombin time, and a decreased fibrinogen level with an increased D-dimer.

Complementary and alternative medical therapies (CAM) are those that are scientifically proven or are not proven; however, they are deemed to be useful as an adjunct to treatment. It is not uncommon for families to try CAM without disclosing this information to the health care team. Is this statement true or false?

ANS: T Families should be asked about CAM therapies by the nurse in a nonthreatening manner. This is important information because some therapies can potentially decrease the efficacy of chemotherapy (such as folate in a child receiving methotrexate).

A diagnostic workup is being performed on a 1-year-old child with suspected neuroblastoma. The nurse reviews the results of the diagnostic tests and understands that which of the following findings is most specifically related to this type of tumor? "1. Elevated vanillylmandelic acid (VMA) urinary levels 2. Presence of blast cells in the bone marrow 3. Projectile vomiting, usually in the morning 4. Postive Babinski's sign"

ANSWER: 1 Rationale: Neuroblastoma is a solid tumor found only in children. It arises from neural crest cells that develop into the sympathetic nervous system and the adrenal medulla. Typically, the tumor compresses adjacent normal tissue and organs. Neuroblastoma cells may excrete catecholamines and their metabolites. Urine samples will indicate elevated VMA levels. The presence of blast cells in the bone marrow occurs in leukemia. Projectile vomiting occurring most often in the morning and a positive Babinski's sign are clinical manifestations of a brain tumor.

When monitoring a patient receiving a blood transfusion, which intervention would the nurse anticipate implementing after notifying the health care provider of the patient's new rash and itching at the intravenous insertion site? 1) Administer diuretics. 2) Administer antipyretics. 3) Prepare patient for dialysis. 4) Administer diphenhydramine hydrochloride.

Administer diphenhydramine hydrochloride.

Which intervention would the nurse implement after the charge nurse immediately stops a transfusion for a patient receiving a second unit of blood who becomes short of breath and has distended neck veins? 1) Administering oxygen and furosemide 2) Administering epinephrine and steroid agents 3) Providing supportive therapy to maintain heart rate and blood pressure 4) No intervention because the patient does not require additional treatment

Administering oxygen and furosemide

"A 4-year-old has a right nephrectomy to remove a Wilms tumor. The nurse knows that it is essential to: "A. Request a low-salt diet B. Restrict fluids C. Educate the family regarding renal transplants D. Prevent urinary tract infections"

Answer D is correct. Because the child has only one remaining kidney, it is important to prevent urinary tract infections. Answers A, B, and C are not necessary, so they are incorrect.

"Which diagnostic test should be performed annually after age 50 to screen for colon cancer? "a. Abdominal computed tomography (CT) b. Abdominal X-ray c. Colonoscopy d. Fecal occult blood test"

Answer d: Surface blood vessels of polyps and cancers are fragile and often bleed with the passage of stools, so a fecal occult blood test and CT scan can help establish tumor size and metastasis. A colonoscopy can help to locate a tumor as well as polyps, but is only recommended every 10 years.

A young child with human immunodeficiency virus is receiving several antiretroviral drugs. The purpose of these drugs is to: Test Bank - Maternal Child Nursing Care by Perry (6th Edition, 2017) 723 a. Cure the disease. b. Delay disease progression. c. Prevent spread of disease. d. Treat Pneumocystis jiroveci pneumonia.

B

The parents of a child hospitalized with sickle cell anemia tell the nurse that they are concerned about narcotic analgesics causing addiction. The nurse should explain that narcotic analgesics: a. Are often ordered but not usually needed. b. Rarely cause addiction because they are medically indicated. c. Are given as a last resort because of the threat of addiction. d. Are used only if other measures such as ice packs are ineffective

B

A 15-year-old has been admitted to the hospital with the diagnosis of acute lymphocytic leukemia. Which of the following signs and symptoms require the most immediate nursing intervention? A. Fatigue and Anorexia B. Fever and Petechiae C. Swollen lymph nodes in the neck and lethargy. D. Enlarged liver and spleen

B (Fever and petechiae associated with acute lymphocytic leukemia indicate a suppression of normal white blood cells and thrombocytes by the bone marrow and put the client at risk for other infections and bleeding. The nurse should initiate infection control and safety precautions to reduce these risks. Fatigue is a common symptom of leukemia due to red blood cell suppression. Although the client should be told about the need for rest and meal planning, such teaching is not the priority intervention. Swollen glands and lethargy may be uncomfortable but they do not require immediate intervention. An enlarged liver and spleen do require safety precautions that prevent injury to the abdomen; however, these precautions are not the priority.)

A preschool-aged child is to undergo several painful procedures. Which of the following techniques is most-appropriate for the nurse to use in preparing the child? A. Allow the child to practice injections on a favorite doll. B. Explain the procedure in simple terms. C. Allow a family member to explain the procedure to the child. D. Allow the child to watch an educational video.

B (Preschoolers have the cognitive ability to understand simple terms. Use of a favorite doll is contraindicated because it is ""part"" of that child and he/she might perceive the doll is experiencing pain.)

After a client is admitted to the pediatric unit with a diagnosis of acute lymphocytic leukemia, the laboratory test indicates that the client is neutropenic. The nurse should perform which of the following? a. Advise the client to rest and avoid exertion b. Prevent client exposure to infections c. Monitor the blood pressure frequently d. Observe for increased bruising

B (Rationale: Neutropenia is a decreased number of neutrophil cells in the blood which are responsible for the body's defense against infection. Rest and avoid exertion would be related to erythrocytes and oxygen carrying properties. Monitoring the blood pressure, and observing for bruising would be related to platelets and sign and symptoms of bleeding.)

A school-age child with leukemia experienced severe nausea and vomiting when receiving chemotherapy for the first time. The most appropriate nursing action to prevent or minimize these reactions with subsequent treatments is to: a. Encourage drinking large amounts of favorite fluids. b. Encourage child to take nothing by mouth (remain NPO) until nausea and vomiting subside. c. Administer an antiemetic before chemotherapy begins. d. Administer an antiemetic as soon as child has nausea

C

A 9-year old child with leukemia is in remission and has returned to school. The school nurse calls the mother of the child and tells the mother that a classmate has just been diagnosed with chickenpox. The mother immediately calls the clinic nurse because the leukemic child has never had chickenpox. The appropriate response by the clinic nurse to the mother is: A. There is no need to be concerned. B. Bring the child into the clinic for a vaccine. C. Keep the child out of school for 2 week period. D. Monitor the child for an elevated temperature, and call the clinic if this happens.

B (Rationale: immunocompromised children are unable to fight varicella adequately. Chickenpox can be deadly to the them. If the child who has not had chickenpox is exposed to someone with varicella, the child should receive varicella zoster immune globulin within 96hrs of exposure. Options 1,3,4, are incorrect because they do nothing to minimize the chances of developing the disease.)

The nurse is caring for a child with aplastic anemia. Which nursing diagnoses are appropriate (Select all that apply)? a. Acute Pain related to vaso-occlusion b. Risk for Infection related to inadequate secondary defenses or immunosuppression c. Ineffective Protection related to thrombocytopenia d. Ineffective Tissue Perfusion related to anemia e. Ineffective Protection related to abnormal clotting

B C D

Which should the nurse teach about prevention of sickle cell crises to parents of a preschool child with sickle cell disease (Select all that apply)? a. Limit fluids at bedtime. b. Notify the health care provider if a fever of 38.5 C (101.3 F) or greater occurs. c. Give penicillin as prescribed. Test Bank - Maternal Child Nursing Care by Perry (6th Edition, 2017) 732 d. Use ice packs to decrease the discomfort of vaso-occlusive pain in the legs. e. Notify the health care provider if your child begins to develop symptoms of a cold.

B C E

Parents of a school-age child with hemophilia ask the nurse, Which sports are recommended for children with hemophilia? Which sports should the nurse recommend (Select all that apply)? a. Soccer b. Swimming c. Basketball d. Golf e. Bowling

B D E

A nurse is leading a discussion with a group of new mothers about newborn nutrition and its importance for growth and development. One of the mothers asks, "Doesn't the baby get iron from me before birth?" Which response by the nurse would be most appropriate? A) "You give the baby some iron, but it is not enough to sustain him after birth." B) "Because the baby grows rapidly during the first months, he uses up what you gave him." C) "The iron you give him before birth is different from what he needs once he is born." D) "If the baby didn't use up what you gave him before birth, he excretes it soon after birth."

B) "Because the baby grows rapidly during the first months, he uses up what you gave him."

The nurse is caring for a 12-year-old boy with idiopathic thrombocytopenia. The nurse is providing discharge instructions about home care and safety recommendations to the boy and his parents. Which response indicates a need for further teaching? A) "We should avoid aspirin and drugs like ibuprofen." B) "He can resume participation in football in 2 weeks." C) "Swimming would be a great activity." D) "Our son cannot take any antihistamines."

B) "He can resume participation in football in 2 weeks."

The nurse is caring for a 5-year-old boy undergoing radiation treatment for a neuroblastoma. Which nursing diagnosis would be most applicable for this child? A) Activity intolerance related to anemia and weakness from medications B) Impaired skin integrity related to desquamation from cellular destruction C) Impaired oral mucosa related to the presence of oral lesions from malnutrition D) Imbalanced nutrition, less than body requirements related to nausea and vomiting

B) Impaired skin integrity related to desquamation from cellular destruction

Which findings would the nurse suspect to be observe during a work up for in a pediatric patient suspected of having a brain tumor? (Select all that apply.) A. Vomiting following eating B. Headaches upon arising that dissipates as the day progresses C. Decreased pulse pressure D. Abnormal cranial nerve examination E. Negative Babinksi sign

B, C, D The presence of brain tumor would cause a variety of clinical symptoms depending on the location and extent of the tumor. Expected physical findings are associated with headache upon arising that subsides as the day progresses, a decreased pulse pressure, abnormal neurological exam which includes cranial nerve abnormalities as well as a positive Babinski reflex. Vomiting specifically associated with eating is not directly correlated with a brain tumor. Vomiting can be present but may or not be associated with nausea or feeding.

A school-age child is being seen in the oncology clinic for possible Hodgkin's disease. During the course of the nursing assessment, which findings would be expected? Select all that apply. a) fever. b) painless cervical nodes. c) painful cervical nodes. d) poor appetite. e) complaints of night sweats"

B, D (Painless cervical nodes are a hallmark sign of HD. In addition to this, night sweats also are characteristic. Fever, poor appetite, and painful cervical nodes are more characteristic of infection.")

A pediatric patient has been diagnosed with leukemia and presents with a white blood cell (WBC) count of 80,000 mm3. Which statement if provided by a nursing student indicates that additional teaching is needed with regard to pathophysiological mechanisms of leukemia? A. The increase in WBC provides protection against bacterial infections. B. Although the WBC count is elevated, there are increased blast cells which help to protect the patient against infection. C. The amount of white blood cells is greatly increased, which affords protection against viral infections. D. Increases in white blood cells are expected but associated with a low leukocyte count.

B.

With regard to incidence of childhood cancer, which statement is accurate? A. In children there is a high incidence of cancer. B. Despite a low incidence, there is high morbidity in children under the age of 15. C. Gender does not affect incidence of childhood cancers. D. Higher incidence in found in African American children as compared to Caucasians.

B. Despite a lower incidence of childhood cancer, there is a higher morbidity associated with specific age groups. Different subtypes of cancer are affected by gender, age and ethnicity. A higher incidence of cancers are found in Caucasian children as compared to African American children.

A pediatric patient has been diagnosed with leukemia and presents with a white blood cell (WBC) count of 80,000 mm3. Which statement if provided by a nursing student indicates that additional teaching is needed with regard to pathophysiological mechanisms of leukemia? A. The increase in WBC provides protection against bacterial infections. B. Although the WBC count is elevated, there are increased blast cells which help to protect the patient against infection. C. The amount of white blood cells is greatly increased, which affords protection against viral infections. D. Increases in white blood cells are expected but associated with a low leukocyte count.

B. Although the WBC count is elevated, there are increased blast cells which help to protect the patient against infection. In leukemia, white blood cell (WBC) count is elevated with an increase in blast or immature cells, which limit the functional ability of WBCs being able to fight off infection. Increases in WBC do not provide protection against bacterial or viral infections. Although one expects to see an increase in the overall WBC, it is associated with a low leukocyte count.

Iron dextran is ordered for a young child with severe iron deficiency anemia. Nursing considerations include: a. Administering with meals. b. Administering between meals. c. Injecting deeply into a large muscle. d. Massaging injection site for 5 minutes after administration of drug

C

What is appropriate mouth care for a toddler with mucosal ulceration related to chemotherapy? A. Lemon glycerin swabs for cleansing B. Mouthwashes with normal saline C. Mouthwashes with hydrogen peroxide D. Local anesthetic such as viscous lidocaine before meals

B. Mouthwashes with normal saline Lemon glycerin swabs can irritate eroded tissue and decay teeth. Normal saline mouthwashes are the preferred mouth care for this age-group. The rinse will keep the mucosal surfaces clean without risking adverse effects on the mucosa or adverse effects caused by the child swallowing the rinse. Hydrogen peroxide delays healing by breaking down protein. Viscous lidocaine is not recommended for toddlers because it depresses the gag reflex.

The primary care pediatric nurse practitioner sees a 12monthold infant who is being fed goat's milk and a vegetarian diet. The child is pale and has a beefyred, sore tongue and oral mucous membranes. Which tests will the nurse practitioner order to evaluate this child's condition? A. Hemoglobin electrophoresis B. RBC folate, iron, and B12 levels C. Reticulocyte levels D. Serum lead levels

B. RBC folate, iron, and B12 levels

A complete blood count on a 12monthold infant reveals microcytic, hypochromic anemia with a hemoglobin of 9.5 g/dL. The infant has mild pallor with no hepatosplenomegaly. The primary care pediatric nurse practitioner suspects A. hereditary spherocytosis. B. irondeficiency anemia. C. lead intoxication. D. sicklecell anemia.

B. irondeficiency anemia.

Which standards of care would the nurse monitor to ensure a safe and effective transfusion when assisting in administration of a patient's blood transfusion? (Select all that apply. One, some, or all responses may be correct.) 1) Blood transfusion tubing includes a special blood filter. 2) Monitor the patient closely after blood transfusion for any changes. 3) Ensure that transfusion occurs within 5 hours after removal from refrigeration. 4) Ensure that transfusion of the blood occurs after verifying the patient's identification. 5) Remove the blood from refrigeration 1 hour before beginning the transfusion.

Blood transfusion tubing includes a special blood filter. Monitor the patient closely after blood transfusion for any changes. Ensure that transfusion of the blood occurs after verifying the patient's identification.

Several blood tests are ordered for a preschool child with severe anemia. She is crying and upset because she remembers the venipuncture done at the clinic 2 days ago. The nurse should explain that: a. Venipuncture discomfort is very brief. b. Only one venipuncture will be needed. c. Topical application of local anesthetic can eliminate venipuncture pain. d. Most blood tests on children require only a finger puncture because a small amount of blood is needed

C

The parents of a child with cancer tell the nurse that a bone marrow transplant (BMT) may be necessary. What should the nurse recognize as important when discussing this with the family? a. BMT should be done at time of diagnosis. b. Parents and siblings of child have a 25% chance of being a suitable donor. c. Finding a suitable donor involves matching antigens from the human leukocyte antigen (HLA) system. d. If BMT fails, chemotherapy or radiotherapy must be continued

C

When teaching the mother of a 9-month-old infant about administering liquid iron preparations, the nurse should include that: a. They should be given with meals. b. They should be stopped immediately if nausea and vomiting occur. c. Adequate dosage will turn the stools a tarry green color. Test Bank - Maternal Child Nursing Care by Perry (6th Edition, 2017) 714 d. Preparation should be allowed to mix with saliva and bathe the teeth before swallowing.

C

Which child should the nurse document as being anemic? a. 7-year-old child with a hemoglobin of 11.5 g/dL Test Bank - Maternal Child Nursing Care by Perry (6th Edition, 2017) 727 b. 3-year-old child with a hemoglobin of 12 g/dL c. 14-year-old child with a hemoglobin of 10 g/dL d. 1-year-old child with a hemoglobin of 13 g/dL

C

Which condition is caused by a virus that primarily infects a specific subset of T lymphocytes, the CD4 + T- cells? a. Wiskott-Aldrich syndrome b. Idiopathic thrombocytopenic purpura (ITP) c. Acquired immunodeficiency syndrome (AIDS) d. Severe combined immunodeficiency disease

C

Which statement best describes b-thalassemia major (Cooleys anemia)? a. All formed elements of the blood are depressed. b. Inadequate numbers of red blood cells are present. c. Increased incidence occurs in families of Mediterranean extraction. d. Increased incidence occurs in persons of West African descent.

C

Which statement most accurately describes the pathologic changes of sickle cell anemia? a. Sickle-shaped cells carry excess oxygen. b. Sickle-shaped cells decrease blood viscosity. c. Increased red blood cell destruction occurs. d. Decreased red blood cell destruction occurs

C

A client is admitted to the hospital for a colon resection and in preparation for surgery the physician orders neomycin. The nurse understands the main reason why this antibiotic is especially useful before colon surgery is because it: A. Will not affect the kidneys B. Acts systemically without delay C. Has limited absorption from the GI tract. D. Is effective against many different organisms

C (Because neomycin is limited absorption form the GI tract, it exerts it antibiotic effect on the intestinal mucosa. In preparation of GI surgery, the level of microbial organisms will be reduced.)

A child being treated for Acute Lymphocytic Leukemia (ALL) has a white blood cell (WBC) count of 7,000/mm3. the nursing care plan lists risk for infection as a priority nursing diagnosis, and measures are being taken to reduce the child's exposure to infection. the nurse determines that the plan has been successful when which outcome has been met? A. child's WBC count goes up. B. child's WBC count goes down. C. child's temperature remains within normal range. D. parents demonstrate good hand washing technique."

C (In leukemia, the WBCs that are present are immature and incapable of fighting infection. increases or decreases in the number of WBCs can be related to the disease process and treatment, and not related to infection. the only value that indicates the child is infection-free is the temperature. the use of proper handwashing technique is a measure or intervention used to meet a goal. but is not a goal itself. STRATEGY: the core issue of the question is knowledge of an indicator of infection in a client who is immunosuppressed from leukemia. recall that temperature and WBC counts are frequently used as indicators of infection. recall that in leukemia the WBCs are abnormal so choose the option related to temperature.)

a

In planning for a child's discharge after a sickle cell crisis, the nurse recognizes which of the following as a critical factor to include in the teaching plan? a. ingestion of large quantities of liquids to promote adequate hydration b. rigorous exercise schedule to promote muscle strength c. a high-caloric diet to improve nutrition d. at least 12 hours of sleep per night to promote adequate rest

A child is admitted to the hospital with a diagnosis of Wilm's tumor, Stage II. Which of the following statements most accurately describes this stage? A. The tumor is less than 3 cm. in size and requires no chemotherapy. B. The tumor did not extend beyond the kidney and was completely resected. C. The tumor extended beyond the kidney but was completely resected. D. The tumor has spread into the abdominal cavity and cannot be resected.

C (The staging of Wilm's tumor is confirmed at surgery as follows: Stage I, the tumor is limited to the kidney and completely resected; stage II, the tumor extends beyond the kidney but is completely resected; stage III, residual nonhematogenous tumor is confined to the abdomen; stage IV, hematogenous metastasis has occurred with spread beyond the abdomen; and stage V, bilateral renal involvement is present at diagnosis.)

A 5-year-old girl is diagnosed with iron-deficiency anemia and is to receive iron supplements. The child has difficulty swallowing tablets, so a liquid formulation is prescribed. After teaching the parents about administering the iron supplement, which statement indicates the need for additional teaching? A) "She needs to eat foods that are high in fiber so she doesn't get constipated." B) "We'll try to get her to drink lots of fluids throughout the day." C) "We will place the liquid in the front of her gums, just below her teeth." D) "We need to measure the liquid carefully so that we give her the correct amount."

C) "We will place the liquid in the front of her gums, just below her teeth."

Which finding if observed would warrant intervention regarding the administration of an antiemetic in a chemotherapy protocol for a pediatric patient? A.Providing the medication on a scheduled basis regardless of the patient's clinical symptoms. B. Administering the medication via the parenteral route prior to infusion of chemotherapy protocol. C. Providing medication with sips of water following clinical symptoms of nausea and/or vomiting. D.Administering 30 to 60 minutes prior to initiation of therapy.

C.

Administration of colony stimulating agents for the pediatric oncology patient are based on the fact that A. increase the time frame for genetic adaptation. B. delays the onset of cellular regeneration. C. it will stimulate production of blood cell components. D. increase bone marrow recovery time.

C. Colony stimulating agents used in the treatment of pediatric oncology patients help to restore functional integrity of the bone marrow leading to decreased likelihood of infections. They decrease bone marrow recovery time and stimulate bone marrow growth of specific cellular components.

Pediatric oncology patients are affected by medical management of their respective disease process and yet it is critical to include health promotion measures as part of their overall care. Which health promotion is not indicated in the plan of care? A. Continuation of dental hygiene treatment plan consistent with developmental age of child. B. Family members should receive live measles, mumps and rubella vaccinations as warranted. C. No treatment should be given if the patient has been exposed to varicella. D. The patient should not receive live attenuated vaccines during the course of chemotherapy protocol.

C. If a pediatric oncology patient has been exposed to varicella, dependent on the time frame either varicella immune zoster immunoglobulin should be administered (within 96 hours of exposure) and/or treatment with antiviral agents should be provided if the patient develops varicella. This treatment is indicated as the development of varicella can lead to increased morbidity and mortality. All of the other options should be included in a health promotion plan of care.

A pediatric patient has been diagnosed with leukemia and presents with a white blood cell (WBC) count of 80,000 mm3. In teaching a group of nursing students about the disease process, how would the nursing instructor describe the proliferation of white blood cells and their ability to fight off infection? A. The increase in WBC provides protection against viral infections but not bacterial infections. B. Although the WBC count is elevated, there are limited blast cells which leads to an increased likelihood that the patient will develop an infection. C. There is an increase in immature cells which reduce the body's ability to fight off infection. D. Although the WBC count is elevated, they are overwhelmed with mature cells that predispose the individual to develop an infection.

C. In leukemia, WBC count is elevated with an increase in blast or immature cells which limit the functional ability of WBCs being able to fight off infection.

A young pediatric oncology patient has stomatitis. Which intervention if observed by the charge nurse would warrant immediate action? A. The nurse assigned to the patient was offering mouth care using a sponge toothbrush. B. The nurse offers the patient frequent mouth rinses. C. The nurse is preparing to use viscous lidocaine to offer pain relief. D. The nurse administers sucralfate as ordered.

C. Use of viscous lidocaine is contraindicated in mucosal alterations as it can lead to potential aspiration and seizure activity. All of the other interventions are appropriate and can be used for symptomatic relief of stomatitis.

When treating nausea and vomiting as a side effect of chemotherapy and/or radiotherapy, ondansetron (Zofran) is the preferred drug of choice because? A. It has a shorter onset of action. B. It can be administered via several different routes. C. It does not cause extrapyramidal side effects. D. It has no adverse side effects if administered appropriately.

C. Zofran is a 5-hydroxytryptamine-3 receptor antagonist and is considered the antiemetic of choice for oncology patients as it produces no extrapyramidal side effects. Pharmacodynamics and pharmacokinetic features aside, the preference for this medication is due to producing no extrapyramidal side effects. Any medication even if administered properly has the potential to cause side effects. The ability to administer via different routes does not indicate a preferred drug choice.

A pediatric oncology patient has been discharged home following a course of chemotherapy. Which information should be included as part of discharge planning with regard to health promotion? A. No further treatments are needed and the patient can resume routine health assessments as developmentally appropriate. B. There are no restrictions based on activity and/or contacts with friends and family members. C. Certain restrictions will be in place related to immunizations that can be administered. D. The patient should limit fluid intake for several months in order to prevent overhydration from occurring.

C. Certain restrictions will be in place related to immunizations that can be administered. With regard to discharge planning for a pediatric oncology patient following a course of chemotherapy, certain restrictions will be in place related to immunizations that can be administered as the patient cannot receive any live viruses. Chemotherapy protocols typically involve a series of treatment therapies, so further treatment will be needed along with possible restrictions of activity and/or contacts with family members until the patient has recovered from effects of immunosuppression due to chemotherapy. Fluids should not be limited at this time as the patient should maintain or increase fluid intake so as to remain hydrated.

Which finding if observed would warrant intervention regarding the administration of an antiemetic in a chemotherapy protocol for a pediatric patient? A. Providing the medication on a scheduled basis regardless of the patient's clinical symptoms. B. Administering the medication via the parenteral route prior to infusion of chemotherapy protocol. C. Providing medication with sips of water following clinical symptoms of nausea and/or vomiting. D. Administering 30 to 60 minutes prior to initiation of therapy.

C. Providing medication with sips of water following clinical symptoms of nausea and/or vomiting. Administering an antiemetic by mouth is not the preferred route especially when the patient is having symptoms of nausea and vomiting. Preferred route is parenteral. Antiemetic medication should be provided on a scheduled basis, via parenteral route and prior to initiation of chemotherapy, typically 30 to 60 minutes. Anticipatory management of an antiemetic is part of chemotherapy and/or radiation protocols. It is typically given 30 to 60 minutes prior to the infusion.

Nursing care of the child with myelosuppression from leukemia or chemotherapeutic agents should include: A. restricting oral fluids. B. instituting strict isolation. C. using good handwashing. D. giving immunizations appropriate for age.

C. using good handwashing. There is no indication that fluids should be reduced. Strict isolation is not necessary. Good hand washing is the most effective means of preventing disease transmission. The child should not receive any live vaccines. The immune system is not capable of responding appropriately to the vaccine.

b

Care for the child with acute idiopathic thrombocytopenic purpura (ITP) includes which therapeutic intervention? a. Splenectomy b. Intravenous administration of anti-D antibody c. Use of nonsteroidal anti-inflammatory drugs (NSAIDs) d. Helping child participate in sports

The nurse understands that the scope of practice for all licensed practical/vocational nurses (LPN/LVNs) regarding blood transfusion includes which responsibilities? (Select all that apply. One, some, or all responses may be correct.) 1) Checking the blood with the registered nurse (RN) 2) Obtaining blood from the blood bank 3) Monitoring the patient during transfusion 4) Stopping the transfusion if any sign of reaction is observed 5) Starting the blood within 30 minutes of arrival to the scene

Checking the blood with the registered nurse (RN) Obtaining blood from the blood bank Monitoring the patient during transfusion Stopping the transfusion if any sign of reaction is observed

The nurse is caring for a child who has been admitted for a sickle cell crisis. Which of the following would the nurse do first to provide adequate pain management? A) Administer a nonsteroidal anti-inflammatory drug as ordered. B) Use guided imagery and therapeutic touch. C) Administer meperidine as ordered. D) Initiate pain assessment with a standardized pain scale.

D) Initiate pain assessment with a standardized pain scale.

The nurse is caring for a 3-year-old boy with a diagnosis of acute lymphocytic leukemia. The child is crying and complaining that his knees hurt. Which nursing intervention is most appropriate? 1.Administer acetaminophen to the child. 2.Involve the child in a diversional activity. 3.Ask the child if he would like a "baby aspirin." 4.Apply heat to the child's knees and elevate the knees on a pillow.

Correct Answer: 1 Rationale: Acetaminophen is acceptable and does not have anticoagulant properties. Diversional activities would not relieve the pain. Aspirin is not administered to the child with acute lymphocytic leukemia (ALL) because of its anticoagulant properties, and administering aspirin could lead to bleeding in the joints. Heat also would increase the pain by increasing circulation to the area.

A child is brought to the emergency department after falling from a high swing and landing on the back. The nurse notes that the client also has hemophilia. Based on the client's history and the nature of the injury, which should the nurse assess for first? 1.Blood in the urine 2.Oxygen saturation 3.Presence of headache 4.Presence of slurred speech

Correct Answer: 1 Rationale: Because the kidneys are located in the flank region of the body, trauma to the back area can cause hematuria, particularly in the child with hemophilia. The nurse would be most concerned about the child's airway and respiratory rate if the child sustained an injury to the neck region. Headache and slurred speech are associated with head trauma.

The nurse is reviewing the laboratory and diagnostic test results of a 5-year-old child scheduled to be seen in the clinic. The nurse notes that the health care provider documented that diagnostic studies revealed the presence of Reed-Sternberg cells. The nurse prepares to assist the health care provider to discuss which initial procedure with the parents? 1.Chemotherapy 2.Surgical biopsy 3.High-dose radiation 4.Intravenous antibiotics

Correct Answer: 1 Rationale: Hodgkin's disease is a neoplasm of lymphatic tissue. The presence of giant, multinucleated cells (Reed-Sternberg cells) is the hallmark of this disease. Initially the nurse should prepare the child for diagnostic procedures and a surgical biopsy. Once Hodgkin's disease is confirmed, induction chemotherapy is then begun as soon as the child is stable and staging of the disease has been completed. High-dose radiation may be used if the disease is detected in a single site or in full-grown adolescents but usually is not the initial treatment in small children. Hodgkin's disease is cancer, not a bacterial infection.

A 2-year-old boy with a diagnosis of hemophilia is admitted to the hospital with bleeding into the joint of the right knee. Which intervention should the nurse plan to implement with this child? 1.Measure the injured knee joint every shift. 2.Take the temperature by rectal method only. 3.Administer acetylsalicylic acid for pain control. 4.Immobilize the joint and apply moist heat to the joint.

Correct Answer: 1 Rationale: Interventions for bleeding into the joint include measuring the injured joint to assess for progression of the bleeding. This provides objective rather than subjective data, which are needed to determine if the bleeding is increasing. Rectal temperatures can cause tissue trauma, causing further bleeding. The application of heat and the administration of acetylsalicylic acid will increase bleeding.

The nurse analyzes the laboratory values of a child with leukemia who is receiving chemotherapy. The nurse notes that the platelet count is 19,500 mm3 (19.5 × 109/L). On the basis of this laboratory result, which intervention should the nurse include in the plan of care? 1.Initiate bleeding precautions. 2.Monitor closely for signs of infection. 3.Monitor the temperature every 4 hours. 4.Initiate protective isolation precautions.

Correct Answer: 1 Rationale: Leukemia is a malignant increase in the number of leukocytes, usually at an immature stage, in the bone marrow. It affects the bone marrow, causing anemia from decreased erythrocytes, infection from neutropenia, and bleeding from decreased platelet production (thrombocytopenia). If a child is severely thrombocytopenic and has a platelet count less than 20,000 mm3 (20.0 × 109/L), bleeding precautions need to be initiated because of the increased risk of bleeding or hemorrhage. Precautions include limiting activity that could result in head injury, using soft toothbrushes, checking urine and stools for blood, and administering stool softeners to prevent straining with constipation. In addition, suppositories, enemas, and rectal temperatures are avoided. Options 2, 3, and 4 are related to the prevention of infection rather than bleeding.

The nurse instructs the parents of a child with leukemia regarding measures related to monitoring for infection. Which statement, if made by the parent, indicates a need for further instructions? 1."I will take a rectal temperature daily." 2."I will inspect the skin daily for redness." 3."I will inspect the mouth daily for lesions." 4."I will perform proper hand washing techniques."

Correct Answer: 1 Rationale: Leukemia is a malignant increase in the number of leukocytes, usually at an immature stage, in the bone marrow. The risk of injury to fragile mucous membranes and resultant bleeding is so high in the child with leukemia that tympanic or axillary temperatures should be taken. In addition, rectal abscesses can occur easily to damaged rectal tissue. No rectal temperatures should be taken. In addition, oral temperature taking should be avoided, especially if the child has oral ulcers. All other options are appropriate measures to prevent infection.

The nurse is performing an assessment on a 10-year-old child suspected to have Hodgkin's disease. Which assessment findings are specifically characteristic of this disease? Select all that apply. 1.Abdominal pain 2.Fever and malaise 3.Anorexia and weight loss 4.Painful, enlarged inguinal lymph nodes 5.Painless, firm, and movable adenopathy in the cervical area

Correct Answer: 1,5 Rationale: Hodgkin's disease (a type of lymphoma) is a malignancy of the lymph nodes. Specific clinical manifestations associated with Hodgkin's disease include painless, firm, and movable adenopathy in the cervical and supraclavicular areas and abdominal pain as a result of enlarged retroperitoneal nodes. Hepatosplenomegaly also is noted. Although fever, malaise, anorexia, and weight loss are associated with Hodgkin's disease, these manifestations are seen in many disorders.

The nurse has reviewed the health care provider's prescriptions for a child suspected of a diagnosis of neuroblastoma and is preparing to implement diagnostic procedures that will confirm the diagnosis. What should the nurse expect to do next to assist in confirming the diagnosis? 1.Collect a 24-hour urine sample. 2.Perform a neurological assessment. 3.Assist with a bone marrow aspiration. 4.Send to the radiology department for a chest x-ray.

Correct Answer: 1 Rationale: Neuroblastoma is a solid tumor found only in children. It arises from neural crest cells that develop into the sympathetic nervous system and the adrenal medulla. Typically, the tumor infringes on adjacent normal tissue and organs. Neuroblastoma cells may excrete catecholamines and their metabolites. Urine samples will indicate elevated vanillylmandelic acid levels. A bone marrow aspiration will assist in determining marrow involvement. A neurological examination and a chest x-ray may be performed but will not confirm the diagnosis.

The nurse is monitoring a 3-year-old child for signs and symptoms of increased intracranial pressure (ICP) after a craniotomy. The nurse plans to monitor for which early sign or symptom of increased ICP? 1.Vomiting 2.Bulging anterior fontanel 3.Increasing head circumference 4.Complaints of a frontal headache

Correct Answer: 1 Rationale: The brain, although well protected by the solid bony cranium, is highly susceptible to pressure that may accumulate within the enclosure. Volume and pressure must remain constant within the brain. A change in the size of the brain, such as occurs with edema or increased volume of intracranial blood or cerebrospinal fluid without a compensatory change, leads to an increase in ICP, which may be life-threatening. Vomiting, an early sign of increased ICP, can become excessive as pressure builds up and stimulates the medulla in the brainstem, which houses the vomiting center. Children with open fontanels (posterior fontanel closes at 2 to 3 months; anterior fontanel closes at 12 to 18 months) compensate for ICP changes by skull expansion and subsequent bulging fontanels. When the fontanels have closed, nausea, excessive vomiting, diplopia, and headaches become pronounced, with headaches becoming more prevalent in older children.

The nurse is providing instructions to the mother of a 3-year-old child with hemophilia regarding care of the child. Which statement by the mother indicates a need for further teaching? 1."I need to cancel the upcoming dental appointment that I made for my child." 2."If my child gets a cut, I should hold pressure on it until the bleeding stops." 3."I should check the house and remove any household items that can easily fall over." 4."I should move furniture with sharp corners out of the way and pad the corners of the furniture."

Correct Answer: 1 Rationale: The nurse needs to stress the importance of immunizations, dental hygiene, and routine well-child care. The remaining options are appropriate care measures. The mother is instructed regarding actions in the event of blunt trauma, especially trauma involving the joints, and is told to apply prolonged pressure to superficial wounds until the bleeding has stopped.

The home care nurse is providing safety instructions to the mother of a child with hemophilia. Which instruction should the nurse include to promote a safe environment for the child? 1.Eliminate any toys with sharp edges from the child's play area. 2.Allow the child to use play equipment only when a parent is present. 3.Allow the child to play indoors only, and avoid any outdoor play or playgrounds. 4.Place a helmet and elbow pads on the child every day as soon as the child awakens.

Correct Answer: 1 Rationale: The nurse should instruct the mother to remove toys with sharp edges that may cause injury from the child's play area. It is not necessary to restrict play if safety measures have been implemented. It is not necessary that the child be restricted from outdoor play activity, but the activities that the child participates in should be monitored. Requiring that the child wear a helmet and elbow pads immediately on awakening and throughout the day is not necessary; however, these items should be worn during activities that could cause injury.

The mother of a 4-year-old child tells the pediatric nurse that the child's abdomen seems to be swollen. During further assessment of subjective data, the mother tells the nurse that the child is eating well and that the activity level of the child is unchanged. The nurse, suspecting the possibility of Wilms' tumor, should avoid which during the physical assessment? 1.Palpating the abdomen for a mass 2.Assessing the urine for the presence of hematuria 3.Monitoring the temperature for the presence of fever 4.Monitoring the blood pressure for the presence of hypertension

Correct Answer: 1 Rationale: Wilms' tumor is the most common intraabdominal and kidney tumor of childhood. If Wilms' tumor is suspected, the tumor mass should not be palpated by the nurse. Excessive manipulation can cause seeding of the tumor and spread of the cancerous cells. Hematuria, fever, and hypertension are clinical manifestations associated with Wilms' tumor.

The nurse is conducting staff in-service training on von Willebrand's disease. Which should the nurse include as characteristics of von Willebrand's disease? Select all that apply. 1.Easy bruising occurs. 2.Gum bleeding occurs. 3.It is a hereditary bleeding disorder. 4.Treatment and care are similar to that for hemophilia. 5.It is characterized by extremely high creatinine levels. 6.The disorder causes platelets to adhere to damaged

Correct Answer: 1,2,3,4,6 Rationale: von Willebrand's disease is a hereditary bleeding disorder characterized by a deficiency of or a defect in a protein termed von Willebrand factor. The disorder causes platelets to adhere to damaged endothelium. It is characterized by an increased tendency to bleed from mucous membranes. Assessment findings include epistaxis, gum bleeding, easy bruising, and excessive menstrual bleeding. An elevated creatinine level is not associated with this disorder.

A 14-year-old child is admitted to the hospital with a diagnosis of acute lymphocytic leukemia. She is receiving a combination chemotherapeutic regimen that includes cyclophosphamide. The nurse plans care understanding that which are associated with this medication? Select all that apply. 1.It is platelet sparing. 2.It causes constipation. 3.It causes hemorrhagic cystitis. 4.It causes bone marrow depression. 5.Increased fluid intake is necessary.

Correct Answer: 1,3,4,5 Rationale: Cyclophosphamide is an alkylating agent used as a chemotherapeutic agent in children with leukemia and other cancers. It also causes hemorrhagic cystitis; therefore, increased fluid intake is necessary. It does not cause constipation. Its side/adverse effects include bone marrow depression (BMD), but it is platelet sparing.

Which clinical manifestations would the nurse associate with a patient diagnosed with leukemia? (Select all that apply. One, some, or all responses may be correct.) 1) Increased white blood cells 2) Decreased white blood cells 3) Possible link to benzene exposure 4) Possible link to intrinsic factor exposure 5) A hemolytic genetic disorder inherited from both parents

Increased white blood cells Possible link to benzene exposure

Nursing care of the child with myelosuppression from leukemia or chemotherapeutic agents should include which intervention? 1.Restrict oral fluids. 2.Use good hand washing technique. 3.Give immunizations appropriate for age. 4.Institute strict isolation with no visitors allowed.

Correct Answer: 2 Rationale: A child with myelosuppression is at risk for infection. Good hand washing technique is necessary to prevent the spread of infection. Restricting oral fluids would not be an intervention to reduce the risk of infection and could actually be harmful to the child. Live virus vaccines are not given when the child is myelosuppressed, so assessment of the child's immune status should be done before administration of immunizations appropriate for age. Strict isolation without visitors is not warranted, although visitors should wear a mask and gloves while in the child's room.

A 13-year-old child is diagnosed with Ewing's sarcoma of the femur. After a course of radiation and chemotherapy, it was decided that leg amputation is necessary. After the amputation, the child becomes very frightened because of aching and cramping felt in the missing limb. Which nursing statement is most appropriate to assist in alleviating the child's fear? 1."The pain medication that I give you will take these feelings away." 2."This aching and cramping is normal and temporary and will subside." 3."This pain is not real pain, and relaxation exercises will help it go away." 4."This normally occurs after the surgery, and we will teach you ways to deal with it."

Correct Answer: 2 Rationale: After amputation, phantom limb pain is a temporary condition that some children experience. This sensation of burning, aching, or cramping in the missing limb is distressing to the child. The child needs to be reassured that the condition is normal and only temporary. All other options are not appropriate responses to the child, as they are incorrect or inappropriate statements.

The nurse is monitoring a child for bleeding after surgery for removal of a brain tumor. The nurse checks the head dressing for the presence of blood and notes a colorless drainage on the back of the dressing. Which intervention should the nurse perform immediately? 1.Reinforce the dressing. 2.Notify the health care provider (HCP). 3.Document the findings and continue to monitor. 4.Circle the area of drainage and continue to monitor.

Correct Answer: 2 Rationale: Colorless drainage on the dressing in a child after craniotomy indicates the presence of cerebrospinal fluid and should be reported to the HCP immediately. Options 1, 3, and 4 are not the immediate nursing intervention because they do not address the need for immediate intervention to prevent complications.

An 11-year-old child is admitted to the hospital in vaso-occlusive sickle cell crisis. The nurse plans for which priority treatments in the care of the child? 1.Splenectomy, correction of acidosis 2.Adequate hydration, pain management 3.Frequent ambulation, oxygen administration 4.Passive range-of-motion exercises, adequate hydration

Correct Answer: 2 Rationale: During vaso-occlusive sickle cell crisis, the care focuses on adequate hydration and pain management. Adequate hydration with intravenous normal saline and oral fluids maintains blood flow and decreases the severity of the vaso-occlusive crisis. Analgesics for pain management are necessary during a vaso-occlusive crisis. Splenectomy would not be done with a vaso-occlusive crisis. Acidosis is not present. Oxygen can be administered to increase tissue perfusion but is not the priority treatment for a vaso-occlusive crisis. Passive range of motion is not recommended; bed rest is prescribed initially.

The nurse is providing home care instructions to the mother of an infant who has just been found to have hemophilia. The nurse should tell the mother that care of the infant should include which appropriate measure? 1.Use aspirin for pain relief. 2.Pad crib rails and table corners. 3.Use a soft toothbrush for dental hygiene. 4.Use a generous amount of lubricant when taking a temperature rectally.

Correct Answer: 2 Rationale: Establishment of an age-appropriate, safe environment is of paramount importance for hemophiliacs. Providing a safe environment for an infant includes padding table corners and crib rails, providing extra padding on clothes to protect the joints, observing a mobile infant at all times, and keeping items that can be pulled down onto the infant out of reach. Use of a soft toothbrush is an appropriate measure for a child with hemophilia but is not typically necessary for an infant. Rectal temperature measurements and the use of aspirin are contraindicated in hemophiliacs because of the risk of bleeding.

The nursing student is assigned to care for a child with hemophilia. The nursing instructor reviews the plan of care with the student. Which intervention on the student written plan of care requires correction? 1.Measure circumference of injured joints. 2.Blood transfusion of packed red blood cells. 3.Monitor temperature with oral thermometers. 4.Intravenous administration of recombinant factor.

Correct Answer: 2 Rationale: Hemophilia is a lifelong hereditary blood disorder associated with deficiency of clotting factors. It is inherited in a recessive manner via a genetic defect on the X chromosome. Hemophilia A results from a deficiency of factor VIII. Hemophilia B (Christmas disease) is a deficiency of factor IX. Blood product transfusion is not the treatment of choice over administering recombinant factors intravenously. Measuring circumference of injured joints is appropriate to assess for enlarging hematomas or bleeding under the skin. The nurse should avoid taking rectal temperatures to decrease the risk for injury.

A 9-year-old child with leukemia is in remission and has returned to school. The school nurse calls the mother of the child and tells the mother that a classmate has just been diagnosed with chickenpox. The mother immediately calls the clinic nurse because the leukemic child has never had chickenpox. Which is an appropriate response by the clinic nurse to the mother? 1."There is no need to be concerned." 2."Bring the child into the clinic for a vaccine." 3."Keep the child out of school for a 2-week period." 4."Monitor the child for an elevated temperature, and call the clinic if this happens."

Correct Answer: 2 Rationale: Immunocompromised children are unable to fight varicella adequately. Chickenpox can be deadly to the immunocompromised child. If an immunocompromised child who has not had chickenpox is exposed to someone with varicella, the child should receive varicella-zoster immune globulin within 96 hours of exposure. All other options are incorrect because they do nothing to minimize the chances of developing the disease.

The nurse is caring for a 16-year-old boy with acute myelogenous leukemia who is having chemotherapy and who has incomplete records for varicella zoster immunization. Which of the following will be the priority nursing diagnosis? A) Pain related to adverse effects of treatment verbalized by the child B) Nausea related to side effects of chemotherapy verbalized by the child C) Constipation related to the use of opioid analgesics for pain D) Risk for infection related to neutropenia and immunosuppression

D) Risk for infection related to neutropenia and immunosuppression

The nurse is instructing the parents of a child with iron deficiency anemia regarding the administration of a liquid oral iron supplement. Which instruction should the nurse tell the parents? 1.Administer the iron at mealtimes. 2.Administer the iron through a straw. 3.Mix the iron with cereal to administer. 4.Add the iron to formula for easy administration.

Correct Answer: 2 Rationale: In iron deficiency anemia, iron stores are depleted, resulting in a decreased supply of iron for the manufacture of hemoglobin in red blood cells. An oral iron supplement should be administered through a straw or medicine dropper placed at the back of the mouth because the iron stains the teeth. The parents should be instructed to brush or wipe the child's teeth or have the child brush the teeth after administration. Iron is administered between meals because absorption is decreased if there is food in the stomach. Iron requires an acid environment to facilitate its absorption in the duodenum. Iron is not added to formula or mixed with cereal or other food items.

A child with sickle cell anemia who is in vaso-occlusive crisis is admitted to the hospital. Which health care provider prescription would assist in reversing the vaso-occlusive crisis? 1.Monitor pulse oximetry. 2.Begin intravenous fluids. 3.Administer oxygen by face mask. 4.Monitor vital signs and respiratory status.

Correct Answer: 2 Rationale: Increased fluid volume reduces the viscosity of the blood, preventing further vascular occlusion and further sickling caused by dehydration. Pulse oximetry and vital sign monitoring may be components of care, but they are actions that relate to monitoring the client versus treating. The intravenous fluids, however, will treat the condition. Vaso-occlusive crisis treatment includes analgesic and fluid administration. Oxygen may help relieve symptoms of respiratory distress, but analgesics and fluids treat the condition.

A 4-year-old child is admitted to the hospital for abdominal pain. The mother reports that the child has been pale and excessively tired and is bruising easily. On physical examination, lymphadenopathy and hepatosplenomegaly are noted. Diagnostic studies are being performed because acute lymphocytic leukemia is suspected. The nurse determines that which laboratory result confirms the diagnosis? 1.Lumbar puncture showing no blast cells 2.Bone marrow biopsy showing blast cells 3.Platelet count of 350,000 mm3 (350 × 109/L) 4.White blood cell count 4,500 mm3 (4.5 × 109/L)

Correct Answer: 2 Rationale: Leukemia is a malignant increase in the number of leukocytes, usually at an immature stage, in the bone marrow. The confirmatory test for leukemia is microscopic examination of bone marrow obtained by bone marrow aspirate and biopsy, which is considered positive if blast cells are present. An altered platelet count occurs as a result of the disease, but also may occur as a result of chemotherapy and does not confirm the diagnosis. The white blood cell count may be normal, high, or low in leukemia. A lumbar puncture may be done to look for blast cells in the spinal fluid that indicate central nervous system disease.

The nurse provides a teaching session to the nursing staff regarding osteosarcoma. Which statement by a member of the nursing staff indicates a need for information? 1."The femur is the most common site of this sarcoma." 2."The child does not experience pain at the primary tumor site." 3."Limping, if a weight-bearing limb is affected, is a clinical manifestation." 4."The symptoms of the disease in the early stage are almost always attributed to normal growing pains."

Correct Answer: 2 Rationale: Osteosarcoma is the most common bone cancer in children. Cancer usually is found in the metaphysis of long bones, especially in the lower extremities, with most tumors occurring in the femur. Osteosarcoma is manifested clinically by progressive, insidious, and intermittent pain at the tumor site. By the time these children receive medical attention, they may be in considerable pain from the tumor. Options 1, 3, and 4 are accurate regarding osteosarcoma.

The pediatric nurse specialist provides a teaching session to the nursing staff regarding osteosarcoma. Which statement by a member of the nursing staff indicates a need for information? 1."The femur is the most common site of this sarcoma." 2."The child does not experience pain at the primary tumor site." 3."Limping, if a weight-bearing limb is affected, is a clinical manifestation." 4."The symptoms of the disease in the early stage are almost always attributed to normal growing pains."

Correct Answer: 2 Rationale: Osteosarcoma is the most common bone cancer in children. Cancer usually is found in the metaphysis of long bones, especially in the lower extremities, with most tumors occurring in the femur. Osteosarcoma is manifested clinically by progressive, insidious, and intermittent pain at the tumor site. By the time these children receive medical attention, they may be in considerable pain from the tumor. The statements in the remaining options are accurate regarding osteosarcoma.

The pediatric nursing instructor asks a nursing student to prioritize care for a child diagnosed with sickle cell disease. Which student response correctly identifies the priority of care? 1.Fatigue 2.Hypoxia 3.Delayed growth 4.Avascular necrosis

Correct Answer: 2 Rationale: Sickle cell disease is a group of diseases termed hemoglobinopathies, in which hemoglobin A is partly or completely replaced by abnormal sickle hemoglobin S. It is caused by the inheritance of a gene for a structurally abnormal portion of the hemoglobin chain. Hemoglobin S is sensitive to changes in the oxygen content of the red blood cell. Hypoxia causes the cells to assume a sickle shape, and the cells become rigid and clumped together, obstructing capillary blood flow and leading to a vaso-occlusive crisis. All the clinical manifestations of sickle cell anemia result from the sickled cells being unable to flow easily through the microvasculature, and their clumping obstructs blood flow. With reoxygenation most of the sickled red blood cells resume their normal shape. Fatigue is a result of hypoxia; hypoxia should be addressed first. Avascular necrosis of the hips and shoulders and delayed growth are general manifestations of sickle cell disease.

The nurse is providing home care instructions to the mother of a child receiving radiation therapy. Which statement by the mother indicates a need for further teaching? 1."I should dress my child in loose-fitting clothing." 2."I won't need to limit the amount of sun that my child gets." 3."My child may experience fatigue and need more rest periods." 4."I need to try to provide food and fluids to prevent dehydration."

Correct Answer: 2 Rationale: Sun protection is essential during radiation treatments. The child should not be exposed to sun during these treatments because of the risk of an alteration of skin integrity. The statements in the remaining options reflect appropriate measures for the child during radiation therapy

The nurse is caring for a 9-year-old child with leukemia who is hospitalized for the administration of chemotherapy. The nurse would monitor the child specifically for central nervous system involvement by checking which item? 1.Pupillary reaction 2.Level of consciousness 3.The presence of petechiae in the sclera 4.Color, motion, and sensation of the extremities

Correct Answer: 2 Rationale: The central nervous system (CNS) status is monitored in the child with leukemia because of the risk of infiltration of blast cells into the CNS. The nurse should check the child's level of consciousness (LOC) and should also monitor for signs of irritability, vomiting, and lethargy. Changes in pupillary reaction are specific to conditions related to increased intracranial pressure. The presence of petechiae in the sclera is an objective sign that may be noted in leukemia but is not specifically related to the CNS. Color, motion, and sensation of the extremities relate to a neurovascular assessment and are not specifically related to CNS status.

The nursing student is presenting a clinical conference and discusses the cause of β-thalassemia. The nursing student informs the group that a child at greatest risk of developing this disorder is which of these? 1.A child of Mexican descent 2.A child of Mediterranean descent 3.A child whose intake of iron is extremely poor 4.A breast-fed child of a mother with chronic anemia

Correct Answer: 2 Rationale: β-Thalassemia is an autosomal recessive disorder characterized by the reduced production of 1 of the globin chains in the synthesis of hemoglobin (both parents must be carriers to produce a child with β-thalassemia major). This disorder is found primarily in individuals of Mediterranean descent. Options 1, 3, and 4 are incorrect.

Which specific nursing interventions are implemented in the care of a child with leukemia who is at risk for infection? Select all that apply. 1.Maintain the child in a semiprivate room. 2.Reduce exposure to environmental organisms. 3.Use strict aseptic technique for all procedures. 4.Ensure that anyone entering the child's room wears a mask. 5.Apply firm pressure to a needle-stick area for at least 10 minutes.

Correct Answer: 2,3,4 Rationale: Leukemia is a malignant increase in the number of leukocytes, usually at an immature stage, in the bone marrow. It affects the bone marrow, causing anemia from decreased erythrocytes, infection from neutropenia, and bleeding from decreased platelet production (thrombocytopenia). A common complication of treatment for leukemia is overwhelming infection secondary to neutropenia. Measures to prevent infection include the use of a private room, strict aseptic technique, restriction of visitors and health care personnel with active infection, strict hand washing, ensuring that anyone entering the child's room wears a mask, and reducing exposure to environmental organisms by eliminating raw fruits and vegetables from the diet and fresh flowers from the child's room and by not leaving standing water in the child's room. Applying firm pressure to a needle-stick area for at least 10 minutes is a measure to prevent bleeding.

The nurse is collecting data on a 12-month-old child with iron deficiency anemia. Which finding should the nurse expect to note in this child? 1.Cyanosis 2.Bronze skin 3.Tachycardia 4.Hyperactivity

Correct Answer: 3 Rationale: Clinical manifestations of iron deficiency anemia will vary with the degree of anemia but usually include extreme pallor with a porcelain-like skin, tachycardia, lethargy, and irritability.

A 10-year-old child with hemophilia A has slipped on the ice and bumped his knee. The nurse should prepare to administer which prescription? 1.Injection of factor X 2.Intravenous infusion of iron 3.Intravenous infusion of factor VIII 4.Intramuscular injection of iron using the Z-track method

Correct Answer: 3 Rationale: Hemophilia refers to a group of bleeding disorders resulting from a deficiency of specific coagulation proteins. The primary treatment is replacement of the missing clotting factor; additional medications, such as agents to relieve pain, may be prescribed depending on the source of bleeding from the disorder. A child with hemophilia A is at risk for joint bleeding after a fall. Factor VIII would be prescribed intravenously to replace the missing clotting factor and minimize the bleeding. Factor X and iron are not used to treat children with hemophilia A.

The nurse is caring for a child with a diagnosis of hemophilia, and hemarthrosis is suspected because the child is complaining of pain in the joints. Which measure should the nurse expect to be prescribed for the child? 1.Range-of-motion exercises to the affected joint 2.Application of a heating pad to the affected joint 3.Application of a bivalved cast for joint immobilization 4.Nonsteroidal antiinflammatory drugs for the pain

Correct Answer: 3 Rationale: In an acute period, immobilization of the joint would be prescribed. Range-of-motion exercise during the acute period can increase the bleeding and would be avoided at this time. Heat will increase blood flow to the area, so it would promote increased bleeding to the area. Nonsteroidal antiinflammatory drugs (NSAIDs) can prolong bleeding time and would not be prescribed for the child.

The pediatric nurse clinician is discussing the pathophysiology related to childhood leukemia with a class of nursing students. Which statement made by a nursing student indicates a need for further teaching of the pathophysiology of this disease? 1.The platelet count is decreased. 2.Red blood cell production is affected. 3.Reed-Sternberg cells are found on biopsy. 4.Normal bone marrow is replaced by blast cells.

Correct Answer: 3 Rationale: In leukemia, normal bone marrow is replaced by malignant blast cells. As the blast cells take over the bone marrow, eventually red blood cell and platelet production is affected, and the child becomes anemic and thrombocytopenic. The Reed-Sternberg cell is found in Hodgkin's disease.

A child is seen in the health care clinic for complaints of fever. On data collection, the nurse notes that the child is pale, tachycardic, and has petechiae. Aplastic anemia is suspected. The nurse should prepare the child to obtain which specimen that will confirm the diagnosis? 1.Platelet count 2.Granulocyte count 3.Red blood cell count 4.Bone marrow biopsy

Correct Answer: 4 Rationale: Although the diagnosis of aplastic anemia may be suspected from the child's history and from the results of a complete blood count, a bone marrow biopsy must be performed to confirm the diagnosis.

The nurse is reviewing the record of a 10-year-old child suspected of having Hodgkin's disease. Which characteristic manifestation should the nurse anticipate to be documented in the assessment notes? 1.Fever 2.Malaise 3.Painful lymph nodes in the supraclavicular area 4.Painless and movable lymph nodes in the cervical area

Correct Answer: 4 Rationale: Clinical manifestations specifically associated with Hodgkin's disease include painless, firm, and movable adenopathy in the cervical and supraclavicular areas. Hepatosplenomegaly is also noted. Although anorexia, weight loss, fever, and malaise are associated with Hodgkin's disease, these manifestations are vague and can be seen in many disorders.

A 6-year-old child with leukemia is hospitalized and is receiving combination chemotherapy. Laboratory results indicate that the child is neutropenic, and protective isolation procedures are initiated. The grandmother of the child visits and brings a fresh bouquet of flowers picked from her garden, and asks the nurse for a vase for the flowers. Which response should the nurse provide to the grandmother? 1."I have a vase in the utility room, and I will get it for you." 2."I will get the vase and wash it well before you put the flowers in it." 3."The flowers from your garden are beautiful, but should not be placed in the child's room at this time." 4."When you bring the flowers into the room, place them on the bedside stand as far away from the child as possible."

Correct Answer: 3 Rationale: Leukemia is a malignant increase in the number of leukocytes, usually at an immature stage, in the bone marrow. It affects the bone marrow, causing anemia from decreased erythrocytes, infection from neutropenia, and bleeding from decreased platelet production (thrombocytopenia). For a hospitalized neutropenic child, flowers or plants should not be kept in the room because standing water and damp soil harbor Aspergillus and Pseudomonas aeruginosa, to which the child is susceptible. In addition, fresh fruits and vegetables harbor molds and should be avoided until the white blood cell count increases.

In caring for a child diagnosed with Hodgkin's disease. Which oncologic emergency should the nurse be most concerned about? 1.Hyperleukocytosis 2.Spinal cord compression 3.Superior vena cava syndrome 4.Disseminated intavascular coagulation

Correct Answer: 3 Rationale: Pediatric oncologic emergencies include tumor lysis syndrome, hyperleukocytosis, superior vena cava syndrome, spinal cord compression, and disseminated intravascular coagulation. Because Hodgkin's disease causes a space-occupying lesion in the chest, superior vena cava syndrome is the most likely emergency that will occur with this type of malignancy. This complication could lead to airway compromise and respiratory failure. The other complications are possible, due to issues with immune response, treatment response, and obstruction, but are less likely to occur due to lesion location in Hodgkin's disease.

A 6-year-old child has just been diagnosed with localized Hodgkin's disease, and chemotherapy is planned to begin immediately. The mother of the child asks the nurse why radiation therapy was not prescribed as a part of the treatment. What is the nurse's best response? 1."It's very costly, and chemotherapy works just as well." 2."I'm not sure. I'll discuss it with the health care provider." 3."Sometimes age has to do with the decision for radiation therapy." 4."The health care provider would prefer that you discuss treatment options with the oncologist."

Correct Answer: 3 Rationale: Radiation therapy is usually delayed until a child is 8 years old, whenever possible, to prevent retardation of bone growth and soft tissue development. Options 1, 2, and 4 are inappropriate responses to the mother and place the mother's question on hold.

The nurse is collecting data on a 9-year-old child suspected of having a brain tumor. Which question should the nurse ask to elicit data related to the classic symptoms of a brain tumor? 1."Do you have trouble seeing?" 2."Do you feel tired all the time?" 3."Do you throw up in the morning?" 4."Do you have headaches late in the day?"

Correct Answer: 3 Rationale: The classic symptoms of children with brain tumors are headache and morning vomiting related to the child getting out of bed. Headaches worsen on arising but improve during the day. Fatigue may occur but is a vague symptom. Visual changes may occur, including nystagmus, diplopia, and strabismus, but these signs are not the hallmark symptoms with a brain tumor.

A child arrives at the emergency department with a nosebleed. On assessment, the nurse is told by the mother that the nosebleed began suddenly and for no apparent reason. What is the initial nursing action? 1.Insert nasal packing. 2.Prepare a nasal balloon for insertion. 3.Ask the child to sit down and lean forward, and apply pressure to the nose. 4.Place the child in a semi Fowler's position, and apply ice packs to the nose.

Correct Answer: 3 Rationale: The initial nursing action for a child with a nosebleed is to have him or her sit down, ask the child to lean forward, and apply pressure to the nose for 5 to 10 minutes. Ice or cool compresses may also be applied to the nose and face. Placing the child in semi Fowler's position would cause swallowing of blood. Inserting nasal packing and preparing a nasal balloon are not appropriate initial interventions. A nasal packing or nasal balloon may be used if conservative measures fail.

The nurse is monitoring for bleeding in a child following surgery for removal of a brain tumor. The nurse checks the head dressing and notes the presence of dried blood on the back of the dressing. The child is alert and oriented, and the vital signs and neurological signs are stable. Which nursing action is most appropriate initially? 1.Prepare to change the dressing. 2.Recheck the dressing in 1 hour. 3.Check the operative record to determine whether a drain is in place. 4.Document the findings and notify the health care provider immediately.

Correct Answer: 3 Rationale: The initial nursing action is to determine whether a drain is in place because the drainage seen on the dressing could be attributed to this. The nurse would not change the dressing without a health care provider's prescription. Rechecking the dressing is an appropriate action, but it is not the initial action. The findings would be documented; however, there is no reason to notify the health care provider immediately. The initial action would be to further assess the cause of the drainage.

The nurse is reviewing the laboratory results of a child with aplastic anemia and notes that the white blood cell count is 2000 mm3 (2 × 109/L) and that the platelet count is 150,000 mm3 (150 × 109/L). Which intervention should the nurse incorporate into the plan of care? 1.Avoid unnecessary injections. 2.Encourage quiet play activities. 3.Maintain strict neutropenic precautions. 4.Encourage the child to use a soft toothbrush.

Correct Answer: 3 Rationale: The normal white blood cell (WBC) count ranges from 5000 to 10,000 mm3 (5 to 10 × 109/L)and the normal platelet count ranges from 150,000 to 400,000 mm3 (150 to 400 × 109/L). Strict neutropenic procedures would be required if the WBC count were low to protect the child from infection. Precautionary measures to prevent bleeding should be taken when a child has a low platelet count. These include no injections, no rectal temperatures, use of a soft toothbrush, and abstinence from contact sports or activities that could cause an injury.

A 12-year-old child with newly diagnosed thalassemia is brought to the clinic exhibiting delayed sexual maturation, fatigue, anorexia, pallor, and complaints of headache. The child seems listless and small for age and has frontal bossing. What should the nurse expect to note on review of the results of the laboratory tests? 1.Macrocytosis and hyperchromia 2.Excessive red blood cell production 3.Excessive mature erythrocyte proliferation 4.Deficient production of functional hemoglobin

Correct Answer: 4 Rationale: Defective hemoglobin is produced as a result of genetically deficient beta-polypeptide. This hemoglobin is unstable, disintegrates, and damages the erythrocytes. Rapid destruction of the red cells stimulates rapid production of immature red cells, and the net gain is less than optimally functioning red cells. Iron from the red blood cell destruction is stored in the tissues, causing multiple problems. In thalassemia, immature erythrocytes proliferate, not mature ones. This is a progressive anemia. The nurse also would note microcytosis and hypochromia.

The nurse analyzes the laboratory results of a child with hemophilia. The nurse understands that which result will most likely be abnormal in this child? 1.Platelet count 2.Hematocrit level 3.Hemoglobin level 4.Partial thromboplastin time

Correct Answer: 4 Rationale: Hemophilia refers to a group of bleeding disorders resulting from a deficiency of specific coagulation proteins. Results of tests that measure platelet function are normal; results of tests that measure clotting factor function may be abnormal. Abnormal laboratory results in hemophilia indicate a prolonged partial thromboplastin time. The platelet count, hemoglobin level, and hematocrit level are normal in hemophilia.

The nurse is monitoring the laboratory values of a child with leukemia who is receiving chemotherapy. The nurse prepares to implement bleeding precautions if the child becomes thrombocytopenic and the platelet count is less than how many cells/mm3? 1.200,000 mm3 (200 × 109/L) 2.180,000 mm3 (180 × 109/L) 3.160,000 mm3 (160× 109/L) 4.150,000 mm3 (150 × 109/L)

Correct Answer: 4 Rationale: If a child is thrombocytopenic, precautions need to be taken because of the increased risk of bleeding. The precautions include limiting activity that could result in head injury, using soft toothbrushes, checking urine and stools for blood, and administering stool softeners to prevent straining with constipation. Additionally, suppositories and rectal temperatures are avoided. The normal platelet count ranges from 150,000 to 400,000 mm3 (150 to 400 × 109/L).

A child is scheduled for allogeneic bone marrow transplantation (BMT). The parent of the child asks the nurse about the procedure. The nurse should provide which description about the BMT? 1.Aspiration of bone marrow from the child 2.Obtaining bone marrow from the child's twin 3.Obtaining bovine (cow) bone marrow and administering it to the child 4.Obtaining bone marrow from a donor who matches the child's tissue type

Correct Answer: 4 Rationale: In allogeneic BMT, a donor who matches the child's tissue type is found. That bone marrow is then given to the child. In autologous BMT, the child undergoes general anesthesia for aspiration of his or her bone marrow, which is then processed in the laboratory and frozen until that marrow needs to be infused back into the child. Syngeneic BMT is done when the child has an identical twin. Administering bovine bone marrow to the child is not used in BMT.

Laboratory studies are performed for a child suspected to have iron deficiency anemia. The nurse reviews the laboratory results, knowing that which result indicates this type of anemia? 1.Elevated hemoglobin level 2.Decreased reticulocyte count 3.Elevated red blood cell count 4.Red blood cells that are microcytic and hypochromic

Correct Answer: 4 Rationale: In iron deficiency anemia, iron stores are depleted, resulting in a decreased supply of iron for the manufacture of hemoglobin in red blood cells. The results of a complete blood cell count in children with iron deficiency anemia show decreased hemoglobin levels and microcytic and hypochromic red blood cells. The red blood cell count is decreased. The reticulocyte count is usually normal or slightly elevated.

A child with a diagnosis of sickle cell anemia and vaso-occlusive crisis is complaining of severe pain, selecting number 8 on the 1 to 10 pain scale. Which medication would the nurse expect to be prescribed for pain control? 1.Ibuprofen 2.Meperidine 3.Acetaminophen 4.Morphine sulfate

Correct Answer: 4 Rationale: Morphine sulfate is the medication of choice for severe pain for the child with sickle cell anemia. Opioids such as morphine sulfate provide systemic relief. Ibuprofen decreases inflammation locally. Meperidine has neurological adverse effects and can cause seizures and should be avoided. Acetaminophen would not provide adequate pain relief.

A diagnostic workup is being performed on a 1-year-old child with suspected neuroblastoma. The nurse reviews the results of the diagnostic tests and understands that which finding is most specifically related to this type of tumor? 1.Positive Babinski's sign 2.Presence of blast cells in the bone marrow 3.Projectile vomiting, usually in the morning 4.Elevated vanillylmandelic acid urinary levels

Correct Answer: 4 Rationale: Neuroblastoma is a solid tumor found only in children. It arises from neural crest cells that develop into the sympathetic nervous system and the adrenal medulla. Typically, the tumor compresses adjacent normal tissue and organs. Neuroblastoma cells may excrete catecholamines and their metabolites. Urine samples will indicate elevated vanillylmandelic acid levels. The presence of blast cells in the bone marrow occurs in leukemia. Projectile vomiting occurring most often in the morning and a positive Babinski's sign are clinical manifestations of a brain tumor.

The nurse is caring for a child with hemophilia and is reviewing the results that were sent from the laboratory. Which result should the nurse expect in this child? 1.Shortened prothrombin time (PT) 2.Prolonged PT 3.Shortened partial thromboplastin time (PTT) 4.Prolonged PTT

Correct Answer: 4 Rationale: PTT measures the activity of thromboplastin, which is dependent on intrinsic factors. In hemophilia, the intrinsic clotting factor VIII (antihemophilic factor) is deficient, resulting in a prolonged PTT. The results in the remaining options are incorrect. The PT may not necessarily be affected in this disorder.

Chemotherapy dosage is frequently based on total body surFace area (BSA), so it is important for the nurse to do which of the following before administering chemotherapy? A. Measure abdominal girth B. Claculate BMI C. Ask the client about his/her height and weight D. Weigh and measure the client on the day of medication administration

D (To ensure that the client receives optimal doses of chemotherapy, dosing is usually based on the total Body surface area(BSA) which requires accurate height and weight before each med administration. Simply asking the client about height/weight may lead to inaccuracies in determining BSA. Calculating BMI and measuring abdominal girth does not provide the data needed.)

A child in whom sickle cell anemia is suspected is seen in a clinic, and laboratory studies are performed. The nurse checks the laboratory results, knowing that which value would be increased in this disease? 1.Platelet count 2.Hematocrit level 3.Hemoglobin level 4.Reticulocyte count

Correct Answer: 4 Rationale: Sickle cell anemia is a group of diseases termed hemoglobinopathies, in which hemoglobin A is partly or completely replaced by abnormal sickle hemoglobin S. It is caused by the inheritance of a gene for a structurally abnormal portion of the hemoglobin chain. Hemoglobin S is sensitive to changes in the oxygen content of the red blood cell. Insufficient oxygen causes the cells to assume a sickle shape, and the cells become rigid and clumped together, obstructing capillary blood flow. A diagnosis is established on the basis of a complete blood count, examination for sickled red blood cells in the peripheral smear, and hemoglobin electrophoresis. Laboratory studies will show decreased hemoglobin level and hematocrit, a decreased platelet count, an increased reticulocyte count, and the presence of nucleated red blood cells. Reticulocyte counts are increased in children with sickle cell disease because the life span of their sickled red blood cells is shortened.

The clinic nurse instructs parents of a child with sickle cell anemia about the precipitating factors related to sickle cell crisis. Which, if identified by the parents as a precipitating factor, indicates the need for further instruction? 1.Stress 2.Trauma 3.Infection 4.Fluid overload

Correct Answer: 4 Rationale: Sickle cell crises are acute exacerbations of the disease, which vary considerably in severity and frequency; these include vaso-occlusive crisis, splenic sequestration, hyperhemolytic crisis, and aplastic crisis. Sickle cell crisis may be precipitated by infection, dehydration, hypoxia, trauma, or physical or emotional stress. The mother of a child with sickle cell disease should encourage fluid intake of 1½ to 2 times the daily requirement to prevent dehydration.

The pediatric nurse educator is providing a teaching session to nursing staff about hemophilia. Which statement should the nurse educator include? 1."Acetylsalicylic acid is given for pain control." 2."Hemarthrosis is the result of synovial cavity aspiration." 3."Total joint rest along with ice pack application continues for 72 hours after factor VIII is administered." 4."Affected prepubescent girls should be counseled concerning menorrhagia, which may be life-threatening."

Correct Answer: 4 Rationale: The female offspring of an affected male and a carrier female is at risk for hemorrhage once puberty is attained and menstrual cycles begin, and depending on the severity of the hemophilia, a hysterectomy or ablation may be performed. The remaining options are incorrect statements. Aspirin is not routinely given to young children and would not be given to a child with a bleeding disorder because of its effects on platelet aggregation. Hemarthrosis is the result of bleeding into the joint cavity, not of aspiration. Seventy-two hours is too long for the joint to be rested because maintenance of mobility is a primary concern once the bleeding episode has been arrested.

The nurse provides instructions to the mother of a child with sickle cell disease. Which statement by the mother indicates a need for further teaching? 1."I need to be sure that my child has adequate rest periods." 2."I will take my child's temperature and watch for a fever." 3."I need to encourage my child to drink large amounts of fluids." 4."I know my child must spend as much time as possible in the sun."

Correct Answer: 4 Rationale: The nurse should instruct the mother to encourage fluid intake 1.5 to 2 times the daily requirements. Adequate rest periods should be provided, and the child should not be exposed to cold or heat stress. The mother should be taught how to take the child's body temperature and how to use a thermometer properly. Sources of infection should be avoided, as should prolonged exposure to the sun.

Oral iron supplements are prescribed for a 6-year-old child with iron deficiency anemia. Which beverage is the best option to recommend with iron administration? 1.Milk 2.Water 3.Apple juice 4.Orange juice

Correct Answer: 4 Rationale: Vitamin C (ascorbic acid) increases the absorption of iron by the body. The mother should be instructed to administer the medication with a citrus fruit or juice high in vitamin C. From the options presented, the correct option is the only one that identifies the food highest in vitamin C.

The nurse on the pediatric unit is caring for a child with hemophilia who has been in a motor vehicle crash. Which assessment finding, if noted in the child, indicates the need for follow-up? 1.The child maintains affected joints in an immobilized position and denies pain at this time. 2.The child's urine is noted to be clear and light yellow and is negative for red blood cells. 3.The child maintains bruised joints in an elevated position; the bruises noted are beginning to turn yellow. 4.The child is drowsy and difficult to arouse; previously the child was able to respond to questions effectively.

Correct Answer: 4 Rationale: When caring for a child with hemophilia who has sustained injuries, the nurse should monitor for signs of internal bleeding. One sign of internal bleeding is change in level of consciousness, which could indicate intracranial hemorrhage. Additional signs of bleeding include pain, tenderness, and bruising of the affected area and hematuria. Denial of pain of affected joints, clear and light yellow urine that is negative for red blood cells, and bruises that are beginning to turn yellow are not signs of internal or external bleeding.

A child with β-thalassemia is receiving long-term blood transfusion therapy for the treatment of the disorder. Chelation therapy is prescribed as a result of too much iron from the transfusions. Which medication should the nurse anticipate to be prescribed? 1.Fragmin 2.Meropenem 3.Metoprolol 4.Deferoxamine

Correct Answer: 4 Rationale: β-Thalassemia is an autosomal recessive disorder characterized by the reduced production of 1 of the globin chains in the synthesis of hemoglobin (both parents must be carriers to produce a child with β-thalassemia major). The major complication of long-term transfusion therapy is hemosiderosis. To prevent organ damage from too much iron, chelation therapy with either Exjade or deferoxamine may be prescribed. Deferoxamine is classified as an antidote for acute iron toxicity. Fragmin is an anticoagulant used as prophylaxis for postoperative deep vein thrombosis. Meropenem is an antibiotic. Metoprolol is a beta blocker used to treat hypertension.

A child is diagnosed with Wilms' tumor. In planning teaching interventions, what key point should the nurse emphasize to the parents? "1. Do not put pressure on the abdomen. 2. Frequent visits from friends and family will improve morale. 3. Appropriate protective equipment should be worn for contact sports. 4. Encourage the child to remain active."

Correct answer: 1. Do not put pressure on the abdomen. Palpation of Wilms' tumor can cause rupture and spread of cancerous cells. Frequent visitation might allow the child to be exposed to more infections, and activity and sports are discouraged because of the risk of rupture of the encapsulated tumor.

"A 15-year-old has been admitted to the hospital with the diagnosis of acute lymphocytic leukemia. Which of the following signs and symptoms require the most immediate nursing intervention? (Choices were deleted)

Correct: 2. Fever and petechiae associated with acute lymphocytic leukemia indicate a suppression of normal white blood cells and thrombocytes by the bone marrow and put the client at risk for other infections and bleeding. The nurse should initiate infection control and safety precautions to reduce these risks. Fatigue is a common symptom of leukemia due to red blood cell suppression. Although the client should be told about the need for rest and meal planning, such teaching is not the priority intervention. Swollen glands and lethargy may be uncomfortable but they do not require immediate intervention. An enlarged liver and spleen do require safety precautions that prevent injury to the abdomen; however, these precautions are not the priority.

. Myelosuppression associated with chemotherapeutic agents or some malignancies such as leukemia can cause bleeding tendencies because of a/an: a. Decrease in leukocytes. c. Vitamin C deficiency. b. Increase in lymphocytes. d. Decrease in blood platelets.

D

A child with leukemia is receiving triple intrathecal chemotherapy consisting of methotrexate, cytarabine, and hydrocortisone. The purpose of this is to prevent: a. Infection. b. Brain tumor. c. Drug side effects. d. Central nervous system (CNS) disease

D

An acquired hemorrhagic disorder that is characterized by excessive destruction of platelets is: a. Aplastic anemia. b. Thalassemia major. c. Disseminated intravascular coagulation. d. Idiopathic thrombocytopenic purpura

D

You are working with the parents of a pediatric oncology patient who has successfully responded to therapy but is now experiencing body image changes as a result of hair loss due to chemotherapy regimen. The parents are upset about this change in their child's appearance. Which response would be appropriate with regard to the parent's concern? A. As the therapy has been successful, it is important to focus on that rather than body image changes. B. Tell the parents that the child's hair will grow back soon. C. Tell the parents that the child's hair will grow back stronger and healthier. D. Acknowledge the parent's concern and focus on available options that may be used to help with body image concerns.

D

A 4-year-old has a right nephrectomy to remove a Wilms tumor. The nurse knows that it is essential to: A. Request a low-salt diet B. Restrict fluids C. Educate the family regarding renal transplants D. Prevent urinary tract infections

D (Because the child has only one remaining kidney, it is important to prevent urinary tract infections. Answers A, B, and C are not necessary, so they are incorrect.)

A nurse analyzes the lab values of a child with leukemia who is receiving chemotherapy. The nurse notices that the platelet count is 19,500 cell/mm3. Based on this lab value which intervention would the nurse document in her plan of care. A. Monitor closely for signs of infection. B. Temp every four hours. C. Isolation precautions D. Use a small toothbrush for mouth care"

D (Leukemia is a malignant increase in the number of leukocytes, usually at an immature stage, in the bone marrow. It affects the bone marrow, causing from decreased erythrocytes, infection from neutropenia, and bleeding from decreased platelet production. If the platelet count is les than 20,000 than bleeding precautions need to be taken.)

An example of a disease process that is based on a "two-hit" hypothesis leading to a cancer diagnosis is: A. Fanconi anemia B. Wiskott Aldrich syndrome C. Klinefelter syndrome D. Retinoblastoma

D. Retinoblastoma is an example of "two-hit" hypothesis of inheritance leading to development of cancer states. Wiskott Aldrich syndrome is an example of an immunodeficiency state may place the individual at increased risk to develop certain cancers. Fanconi anemia and Klinefelter syndrome are examples of chromosomal abnormalities which can potentially lead to development of cancer.

The best approach that would facilitate improved outcomes when using surgical treatment for operable cancers is A. when there is evidence of adjacent tissue involvement. B. performing amputation rather than attempting resection. C. using multiple excisions to remove the tumor. D. if the tumor is encapsulated and localized.

D. Tumors that are localized and encapsulated represent the best approach for improved outcomes for the surgical cancer patient as this indicates that the tumor is not showing evidence of metastasis. Evidence of adjacent tissue involvement means that the tumor has already metastasized. Resection of bone rather than amputation is associated with improved outcomes. Minimal incision surgical approach is favored to improve functioning and help maintain cosmesis.

A critical concept that needs to be maintained during intravenous administration of chemotherapy for a pediatric patient is? A. Positioning the patient in a semi-fowler's position. B. Not use an infusion device but rather allow for a free-flow line. C. Continue the infusion regardless if the patient develops a rash. D. Maintaining the integrity of the parenteral access line.

D. Administration of chemotherapy via parenteral access requires that the integrity of the access line be maintained and monitored by the nurse. If there is any indication that the site as infiltrated, then the infusion must be immediately stopped. Patient positioning is variable depending on patient comfort. An infusion device must be used as this is considered to be a titratable infusion. If the patient develops a rash in response to chemotherapy, this may be an indication of a hypersensitivity reaction. Intervention is required with notification of health care provider and discontinuing the infusion.

You are working with the parents of a pediatric oncology patient who has successfully responded to therapy. The parents have questions regarding what to expect as the child continues to grow and develop throughout the life cycle. Which response would be appropriate with regard to the parent's concern? A. As the therapy has been successful, growth and development should proceed along a normal sequence. B. It may be a good idea to schedule your child for repeat imaging studies on a yearly basis so as to make sure that the child remains in remission. C. There may be anticipated growth and developmental delays associated with chemotherapy treatments but they are typically self-limiting in nature. D. Genetic counseling may be something to consider as the child reaches adulthood and is considering having children his/herself if the type of cancer that the child had was inherited.

D. Even though medical treatment has been noted as being successful, continued observation and medical follow up is indicated. Growth and development should be monitored in accordance with recommended pediatric screening guidelines. Although imaging studies may be required at some point in time for follow up, yearly imaging studies may not be needed. Genetic counseling when the child reaches adulthood should be considered especially if the type of cancer was inherited. Growth and developmental delays are not considered to be normal and may not be self-limiting.

The nurse is assessing a pediatric oncology patient's nutritional status. Which diagnostic tests would provide best practice approach? A. Albumin, blood urea nitrogen (BUN) and daily weight B. Skinfold assessments and daily weight C. Intake and output with daily calorie count D. Serum prealbumin, albumin and transferrin

D. No one diagnostic test or measurement provides enough evidence to evaluate the nutritional well-being of an individual patient. BUN provides evidence of hydration status but typically should be viewed using a BUN creatinine ratio to provide detailed information about a patient's renal status. Skinfold assessments while important again do not provide enough evidence even with the addition of a daily weight to evaluate one's nutritional status. Intake and output measurements in combination with daily calorie count are representative of hydration and nutritional support but do not provide information relative to nutritional body stores.

A pediatric oncology patient has developed a nose bleed. Which testing parameter would be indicted in order to decide if medical treatment is needed? A. Chest X-ray B. CT of the nose C. Lumbar puncture D. Platelet count

D. Pediatric oncology patients are at an increased risk for hemorrhage and bleeding. Evidence of a nose bleed may indicate thrombocytopenia and as such a platelet count should be obtained. Imaging studies such as CT and chest x-ray will not provide information related to hemostasis. An invasive procedure such as a lumbar puncture would be indicated if there was possibility of an infectious process.

Which findings are consistent with tumor lysis syndrome? A. Hypercalcemia and hyperkalemia B. Hypochloremia and hypokalemia C. Hyponatremia and hyperphosphatemia D. Hyperuricemia and hyperkalemia

D. The hallmark characteristics of tumor lysis syndrome are: hyperuricemia, hypocalcemia, hyperphosphatemia, and hyperkalemia.

Which findings are consistent with tumor lysis syndrome? A. Hypercalcemia and hyperkalemia B. Hypochloremia and hypokalemia C. Hyponatremia and hyperphosphatemia D. Hyperuricemia and hyperkalemia

D. The hallmark characteristics of tumor lysis syndrome are: hyperuricemia, hypocalcemia, hyperphosphatemia, and hyperkalemia.

You are working with the parents of a pediatric oncology patient who has successfully responded to therapy but is now experiencing body image changes as a result of hair loss due to chemotherapy regimen. The parents are upset about this change in their child's appearance. Which response would be appropriate with regard to the parent's concern? A. As the therapy has been successful, it is important to focus on that rather than body image changes. B. Tell the parents that the child's hair will grow back soon. C. Tell the parents that the child's hair will grow back stronger and healthier. D. Acknowledge the parent's concern and focus on available options that may be used to help with body image concerns.

D. Acknowledge the parent's concern and focus on available options that may be used to help with body image concerns. Even though medical treatment has been noted as being successful, continued observation and medical follow up is indicated. Growth and the fact that a child has lost his/her hair may be traumatic for both the child and parents. Acknowledging the parent's concerns and helping to focus on available options that may be used to help with body image concerns is an appropriate response. Focusing on the success of the overall treatment with the exclusion of the parent's concerns is not appropriate. Telling the parents that the child's hair will grow back and/or be even stronger does not acknowledge their concern.

What is a primary reason for using parenteral administration of chemotherapy agents for cancer patients? A. Chemotherapy medications are only available in parenteral forms. B. Decreased likelihood of infiltration or extravasation. C. Parenteral routes lead to decreased absorption of medication. D. Allowing for infusion of therapies without having to perform multiple venipuntures.

D. Allowing for infusion of therapies without having to perform multiple venipuntures. Use of parenteral medication allows for infusion of therapies without having to perform multiple venipunctures. Chemotherapy medications can be given by multiple routes: by mouth, topically, and parenteral. With parenteral administration, there is an increased likelihood that infiltration or extravasation may occur due to the nature of the type of solution combined with the type of venous access. Use of the parenteral route leads to increased absorption of the medication.

What instructions does the nurse provide for following a bone marrow procedure being performed on a pediatric patient for detection of leukemia? A. The patient will have to remain on a low-bacteria diet until the results are obtained. B. Patient should expect to have slight swelling at the site which will lessen within 24 hours of the procedure. C. Round the clock pain medication will be prescribed for the first 24 hours following the procedure. D. No activity restrictions are provided.

D. No activity restrictions are provided. Typically, there are no activity restrictions following a bone marrow procedure. The patient can expect some local soreness at the site but this is usually self-limiting without the need for round the clock pain medication. Dietary restrictions are not typically seen following a bone marrow procedure unless the patient is immunosuppressed as a result of the disease process and/or therapeutic regimen. Swelling at the site is not typically seen following a bone marrow procedure.

A schoolage child comes to the clinic for evaluation of excessive bruising. The primary care pediatric nurse practitioner notes a history of an upper respiratory infection 2 weeks prior. The physical exam is negative for hepatosplenomegaly and lymphadenopathy. Blood work reveals a platelet count of 60,000/mm3 with normal PT and aPTT. How will the nurse practitioner manage this child's condition? A. Admit to the hospital for IVIG therapy. B. Begin a short course of corticosteroid therapy. C. Refer to a pediatric hematologist. D. Teach to avoid NSAIDs and contact sports.

D. Teach to avoid NSAIDs and contact sports.

Which responses would the nurse utilize when a patient diagnosed with leukemia asks what could have caused the disease? (Select all that apply. One, some, or all responses may be correct.) 1) Exposure to ionizing radiation 2) Human T-lymphotropic virus 1 3) An abnormal number of chromosomes 4) Enlargement of the lymph glands 5) Chemicals that are toxic to the bone marrow

Exposure to ionizing radiation Human T-lymphotropic virus 1 An abnormal number of chromosomes Chemicals that are toxic to the bone marrow

b

In anticipation of the admission of a child with hereditary spherocytosis (HS) who is experiencing an aplastic crisis, what action should the nurse plan? a. Secure an isolation room. b. Prepare for a transfusion of packed red blood cells. c. Anticipate preoperative preparation for a splenectomy. d. Gather equipment and medication for treatment of shock.

Which type of anemia would the nurse associate with the patient experiencing neurologic clinical manifestations? 1) Aplastic anemia 2)Sickle cell anemia 3) Megaloblastic anemia 4) Iron deficiency anemia

Megaloblastic anemia

When a middle-aged adult with a history of anemia tells the nurse about "having trouble getting through the day lately," which symptom would the nurse expect the patient to report to the primary health care provider? 1) Lymphadenopathy 2) Shortness of breath 3) Nausea and vomiting 4) Fatigue upon awakening

Shortness of breath

Which characteristics would the nurse associate with an assigned patient with sickle cell anemia? (Select all that apply. One, some, or all responses may be correct.) 1) The disorder is hereditary. 2) Sickle cell anemia is an acquired anemia. 3) The disease is usually asymptomatic. 4) The disease is a result of abnormal hemoglobin. 5) Generally, the disorder results in increased hemoglobin levels.

The disorder is hereditary. The disease is a result of abnormal hemoglobin.

b

The nurse is caring for a school-age child with severe anemia and activity intolerance. What diversional activity should the nurse plan for this child? a. Playing a musical instrument b. Playing board or card games c. Participating in a game of table tennis d. Participating in decorating the hospital room

d

The nurse is instructing a new mother in how to prevent iron-deficiency anemia in her new premature infant when she takes her home. The mother intends to breastfeed. Which of the following statements reflects a need for further teaching? a. "I will use only breast milk or formula as a source of milk for my baby until she is at least 12 months old" b. "My baby will need to have iron supplements introduced when she is 2 months old" c. "As my baby is able to tolerate other foods, such as cereal, I should limit her formula intake to about 1 liter per day to encourage intake of iron-rich cereals" d. "I will need to add iron supplements to my baby's diet when she is 6 months old"

b

The nurse is instructing the parents of a child with iron deficiency anemia regarding the administrating of a liquid oral iron supplement. Which instruction should the nurse tell the parents? a. administer the iron at mealtimes b. administer the iron through a straw c. mix the iron with cereal to administer d. add the iron to formula for easy administration

d

The nurse is preparing to administer a unit of packed red blood cells to a hospitalized child. What is an appropriate action that applies to administering blood? a. Take the vital signs every 15 minutes while blood is infusing. b. Use blood within 1 hour of its arrival from the blood bank. c. Administer the blood with 5% glucose in a piggyback setup. d. Administer the first 50 ml of blood slowly and stay with the child.

c

The nurse is providing home care instructions to the parents of a 10 year old child with hemophilia. Which sport activity should the nurse suggest for this child? a. soccer b. basketball c. swimming d. field hockey

c d e

The nurse is teaching parents of a child being discharged from the hospital after a splenectomy about the risk of infection. What should the nurse include in the teaching session? Select all that apply a. Avoid obtaining the pneumococcal vaccination for the child. b. Avoid obtaining the meningococcal vaccination for the child. c. The child should receive prophylactic penicillin for certain procedures. d. Have the child immunized with the Haemophilus influenzae type b vaccination. e. Notify the health care provider if your child develops a fever of 38.5 C (101.3 F).

c

The nurse is teaching the family of a child, age 8 years, with moderate hemophilia about home care. What should the nurse tell the family to do to minimize joint injury? a. Administer nonsteroidal anti-inflammatory drugs (NSAIDs). b. Administer DDAVP (synthetic vasopressin). c. Provide intravenous (IV) infusion of factor VIII concentrates. d. Encourage elevation and application of ice to the involved joint.

b

The nurse is teaching the parents of a child with sickle cell disease. To instruct them on how to prevent sickle cell crisis, the nurse should include which instructions? a. exercise in cool temperatures b. drink at least 2 quarts of fluids per day c. avoid contact sports d. take anti-inflammatory medications before exercising

b

The parents of a child hospitalized with sickle cell anemia tell the nurse that they are concerned about narcotic analgesics causing addiction. The nurse would explain what concerning narcotic analgesics? a. They are often ordered but not usually needed. b. When they are medically indicated, children rarely become addicted. c. They are given as a last resort because of the threat of addiction. d. They are used only if other measures, such as ice packs, are ineffective.

d

The school nurse is discussing prevention of acquired immunodeficiency syndrome with some adolescents. What is appropriate to include? a. The virus is easily transmitted. b. It is only transmitted through blood. c. Condoms should be used if adolescents are homosexual. d. Recreational drug users should not share needles or other equipment.

a

Therapeutic management of a 6-year-old child with hereditary spherocytosis (HS) should include which therapeutic intervention? a. Perform a splenectomy. b. Supplement the diet with calcium. c. Institute a maintenance transfusion program. d. Increase intake of iron-rich foods such as meat.

d

Therapeutic management of sickle cell crisis generally includes which of the following? a. long term oxygen use to enable the oxygen to reach the sickled RBCs b. decrease in fluids to increase hemoconcentration c. diet high in iron to decrease anemia d. bed rest to minimize energy expenditure

d

To control pain related to vasoocclusive sickle cell crisis, which of the following can the nurse expect to be included in the care plan? a. administration of long-term oxygen b. application of cold compresses to the area c. administration of meperidine (Demerol) titrated to a therapeutic level d. codeine added to acetaminophen or ibuprofen if neither one of these is effective in relieving the pain alone

d

Two year old Karen Lumia is HIV infected. Her mother is concerned about placing Karen in daycare and is discussing this with the nurse at a routine pediatric follow up visit. Which of the following is the best information to provide Karen's mother? a. the risk for HIV transmission is significant in daycare centers. Karen should not go to daycare until she is older b. it will be alright for Karen to attend the daycare, but Karen's mother must tell the daycare that Karen is infected c. Karen can go to the daycare but will not be allowed to participate in sports or physical activity that could lead to injury d. Karen should be admitted to the daycare without restrictions and allowed to participate in all activities as her health permits

c

What condition is an acquired hemorrhagic disorder that is characterized by excessive destruction of platelets? a. Aplastic anemia b. Thalassemia major c. Idiopathic thrombocytopenic purpura d. Disseminated intravascular coagulation

a

What explanation provides the rationale for why iron-deficiency anemia is common during infancy? a. Cows milk is a poor source of iron. b. Iron cannot be stored during fetal development. c. Fetal iron stores are depleted by 1 month of age. d. Dietary iron cannot be started until 12 months of age.

c

What information should the nurse include when teaching the mother of a 9-month-old infant about administering liquid iron preparations? a. Give with meals. b. Stop immediately if nausea and vomiting occur. c. Adequate dosage will turn the stools a tarry green color. d. Allow preparation to mix with saliva and bathe the teeth before swallowing.

b

What is a priority nursing consideration when caring for a child with sickle cell anemia? a. Refer the parents and child for genetic counseling. b. Teach the parents and child how to recognize the signs and symptoms of crises. c. Help the child and family adjust to a short-term disease. d. Observe for complications of multiple blood transfusions.

a

When discussing hemophilia with the parents of a child recently diagnosed with this disease, the nurse tells the parents that a. hemophilia is an X-linked disorder in which the mother is the carrier of the illness but is not affected by it b. hemophilia is a recessive disorder carried by either the mother or father c. all daughters of the parents will be carriers d. each of their sons has a 75% chance of being affected

a

When teaching the parents of 4 year old Tony how to administer the iron supplement ordered for his iron deficiency, the nurse should include which of the following in the teaching plan? a. give the iron twice daily in divided doses with orange juice b. give the iron twice daily with milk c. administer the oral liquid iron preparation with the use of a syringe or medicine dropper directly into each side of the mouth in the cheek areas d. make certain the parents have at least a 3 month supply of the iron preparation on hand so they will not run out

b

When the hemoglobin level falls sufficiently to produce clinical manifestations of anemia, the patient experiences a. cyanosis b. tissue hypoxia c. nausea and vomiting d. feelings of anxiety

a b c d

Which of the following activities should a nurse suggest for a client diagnosed with hemophilia? Select all that apply a. swimming b. golf c. hiking d. fishing e. soccer

b c e

Which of the following describes idiopathic thrombocytopenia purpura? Select all that apply a. ITP is a congenital hematological disorder b. ITP causes excessive destruction of platelets c. children with ITP have normal bone marrow d. platelets are small in ITP e. purpura is observed in ITP

d

Which of the following diagnostic tests can distinguish between those children with sickle cell trait and those with sickle cell disease? a. CBC with differential b. sickledex c. bleeding time d. hemoglobin electrophoresis

a c d e

Which of the following factors needs to be included in a teaching plan for a child with sickle cell disease? Select all that apply a. the child needs to be taken to a physician when sick b. the parent should make sure the child sleeps in an air conditioned room c. emotional stress should be avoided d. it is important to keep the child well hydrated e. it is important to make sure the child gets adequate nutrition

a

Which of the following is an acquired hemorrhagic disorder characterized by thrombocytopenia, absence of severe signs of bleeding, and normal bone marrow with a normal or increased number of immature magakaryocytes and eosinophils? a. immune thrombocytopenia b. disseminated intravascular coagulation c. acute-onset neutropenia d. hemoch-schonlein purpura

a

Which of the following statements about chronic benign neutropenia is true? a. nursing care management includes educating parents to keep their child away from crowded areas and individuals who are ill b. diagnosis is usually made when the child is seen with weight loss and fatigue c. the absolute neutrophil count is usually 1500/mm or less at the time of diagnosis d. children do not receive chronic childhood immunization because of the abnormal cellular immunity and antineutrophil antibodies associated with this disorder

b c e

You are discharging a patient with hemophilia. Which of the following responses by the parents indicate an understanding of this disorder? Select all that apply a. "my child should remain active to decrease joint problems, and most children with hemophilia can participate in the same activities as peers" b. "care should be taken to avoid bleeding of gums, and softening of the toothbrush in warm water before brushing or using a sponge-tipped disposable toothbrush may be helpful" c. "signs of internal bleeding should be recognized, such as headache, slurred speech, loss of consciousness, and black tarry stools" d. "if there is bleeding in a joint, elevation, ice, and rest should help and may prevent the need for factor VIII replacement" e. "all of my son's teachers need to be aware of what to do if he gets a bloody nose"

Which statement by a mother may indicate a cause for her 9-month-old's iron deficiency anemia? a. "Formula is so expensive. We switched to regular milk right away." b. "She almost never drinks water." c. "She doesn't really like peaches or pears, so we stick to bananas for fruit." d. "I give her a piece of bread now and then. She likes to chew on it."

a. "Formula is so expensive. We switched to regular milk right away."

In which of the conditions are all the formed elements of the blood simultaneously depressed? a. Aplastic anemia b. Sickle cell anemia c. Thalassemia major d. Iron deficiency anemia

a. Aplastic anemia Aplastic anemia refers to a bone marrow-failure condition in which the formed elements of the blood are simultaneously depressed. Sickle cell anemia is a hemoglobinopathy in which normal adult hemoglobin is partly or completely replaced by abnormal sickle hemoglobin. Thalassemia major is a group of blood disorders characterized by deficiency in the production rate of specific hemoglobin globin chains. Iron deficiency anemia results in a decreased amount of circulating red cells.

In which of the conditions are all the formed elements of the blood simultaneously depressed? a. Aplastic anemia b. Sickle cell anemia c. Thalassemia major d. Iron deficiency anemia

a. Aplastic anemia Aplastic anemia refers to a bone marrow-failure condition in which the formed elements of the blood are simultaneously depressed. Sickle cell anemia is a hemoglobinopathy in which normal adult hemoglobin is partly or completely replaced by abnormal sickle hemoglobin. Thalassemia major is a group of blood disorders characterized by deficiency in the production rate of specific hemoglobin globin chains. Iron deficiency anemia results in a decreased amount of circulating red cells.

What should be included in the nursing care of a 12-year-old child receiving radiation therapy for Hodgkin's disease? (Select all that apply.) a. Application of sunblock b. Appetite stimulation c. Conservation of energy d. Provision for expressions of anger e. Preparation for premature sexual development

a. Application of sunblock b. Appetite stimulation c. Conservation of energy d. Provision for expressions of anger

On admission, a child with leukemia has widespread purpura and a platelet count of 19,000/mm3. What is the priority nursing intervention? a. Assessing neurological status b. Inserting an intravenous line c. Monitoring vital signs during platelet transfusions d. Providing family education about how to prevent bleeding

a. Assessing neurological status When platelets are low, the greatest danger is spontaneous intracranial bleeding. Neurological assessments are therefore a priority of care.

The school nurse is informed that a child with human immunodeficiency virus (HIV) will be attending school soon. Which is an important nursing intervention? a. Carefully follow universal precautions. b. Determine how the child became infected. c. Inform the parents of the other children. d. Reassure other children that they will not become infected.

a. Carefully follow universal precautions. Universal precautions are necessary to prevent further transmission of the disease. It is not the role of the nurse to determine how the child became infected. Informing the parents of other children and reassuring children that they will not become infected is a violation of the child's right to privacy.

The nurse is preparing to give oral care to a school-age child with mucositis secondary to chemotherapy administered to treat leukemia. Which preparations should the nurse use for oral care on this child? (Select all that apply.) a. Chlorhexidine gluconate (Peridex) b. Lemon glycerin swabs c. Antifungal troches (lozenges) d. Lip balm (Aquaphor) e. Hydrogen peroxide

a. Chlorhexidine gluconate (Peridex) c. Antifungal troches (lozenges) d. Lip balm (Aquaphor) ANS: A, C, D Preparations that may be used to prevent or treat mucositis include chlorhexidine gluconate (Peridex) because of its dual effectiveness against candidal and bacterial infections, antifungal troches (lozenges) or mouthwash, and lip balm (e.g., Aquaphor) to keep the lips moist. Agents that should not be used include lemon glycerin swabs (irritate eroded tissue and can decay teeth), hydrogen peroxide (delays healing by breaking down protein), and milk of magnesia (dries mucosa).

The nurse is planning care for an adolescent with AIDS. Which is the priority nursing goal? a. Preventing infection b. Preventing secondary cancers c. Restoring immunologic defenses d. Identifying source of infection

a. Preventing infection Because the child is immunocompromised in association with HIV infection, the prevention of infection is paramount. Although certain precautions are justified in limiting exposure to infection, these must be balanced with the concern for the child's normal developmental needs. Preventing secondary cancers is not currently possible. Current drug therapy is affecting the disease progression; although not a cure, these drugs can suppress viral replication, preventing further deterioration. Case finding is not a priority nursing goal.

A young child with leukemia has anorexia and severe stomatitis. The nurse should suggest that the parents try which intervention? a. Relax any eating pressures. b. Firmly insist that child eat normally. c. Begin gavage feedings to supplement diet. d. Serve foods that are either hot or cold.

a. Relax any eating pressures. A multifaceted approach is necessary for children with severe stomatitis and anorexia. First, the parents should relax eating pressures. The nurse should suggest that the parents try soft, bland foods; normal saline or bicarbonate mouthwashes; and local anesthetics. The stomatitis is a temporary condition. The child can resume good food habits as soon as the condition resolves.

suddenly begins to wheeze and have severe urticaria. Which is the most appropriate nursing action? a. Stop drug infusion immediately. b. Recheck rate of drug infusion. c. Observe child closely for next 10 minutes. d. Explain to child that this is an expected side effect.

a. Stop drug infusion immediately. If an allergic reaction is suspected, the drug should be immediately discontinued. Any drug in the line should be withdrawn, and a normal saline infusion begun to keep the line open. Rechecking the rate of drug infusion, observing the child closely for next 10 minutes, and explaining to the child that this is an expected side effect can all be done after the drug infusion is stopped and the child is evaluated.

The nurse is conducting a staff in-service on inherited childhood blood disorders. Which statement describes severe combined immunodeficiency syndrome (SCIDS)? a. There is a deficit in both the humoral and cellular immunity with this disease. b. Production of red blood cells is affected with this disease. c. Adult hemoglobin is replaced by abnormal hemoglobin in this disease. d. There is a deficiency of T and B lymphocyte production with this disease.

a. There is a deficit in both the humoral and cellular immunity with this disease. Severe combined immunodeficiency syndrome (SCIDS) is a genetic disorder that results in deficits of both humoral and cellular immunity. Wiskott-Aldrich is an X-linked recessive disorder with selected deficiencies of T and B lymphocytes. Fanconi syndrome is a hereditary disorder of red cell production. Sickle cell disease is characterized by the replacement of adult hemoglobin with an abnormal hemoglobin S.

A young child with human immunodeficiency virus (HIV) is receiving several antiretroviral drugs. The purpose of these drugs is to: a. cure the disease. b. delay disease progression. c. prevent spread of disease. d. treat Pneumocystis carinii pneumonia.

b. delay disease progression. Although not a cure, these antiviral drugs can suppress viral replication, preventing further deterioration of the immune system and delaying disease progression. At this time, cure is not possible. These drugs do not prevent the spread of the disease. P. carinii prophylaxis is accomplished with antibiotics.

Meperidine (Demerol) is not recommended for children in sickle cell crisis because it: a. may induce seizures. b. is easily addictive. c. is not adequate for pain relief. d. is given by intramuscular injection. ANS: A A metabolite of meperidine, normeperidine, is a central nervous system stimulant that produces anxiety, tremors, myoclonus, and generalized seizures when it accumulates with repetitive dosing. Patients with sickle cell disease are particularly at risk for normeperidine-induced seizures. Meperidine is no more addictive than other narcotic agents. Meperidine is adequate for pain relief. It is available for IV infusion.

a. may induce seizures. A metabolite of meperidine, normeperidine, is a central nervous system stimulant that produces anxiety, tremors, myoclonus, and generalized seizures when it accumulates with repetitive dosing. Patients with sickle cell disease are particularly at risk for normeperidine-induced seizures. Meperidine is no more addictive than other narcotic agents. Meperidine is adequate for pain relief. It is available for IV infusion.

Which statement made by a parent indicates an understanding of health maintenance of a child with sickle cell disease? a. "I should give my child a daily iron supplement." b. "It is important for my child to drink plenty of fluids." c. "He needs to wear protective equipment if he plays contact sports." d. "He shouldn't receive any immunizations until he is older."

b. "It is important for my child to drink plenty of fluids." Prevention of dehydration, which can trigger the sickling process, is a priority goal in the care of a child with sickle cell disease.

A young boy will receive a bone marrow transplant (BMT). This is possible because one of his older siblings is a histocompatible donor. Which is this type of BMT called? a. Syngeneic b. Allogeneic c. Monoclonal d. Autologous

b. Allogeneic Allogeneic transplants are from another individual. Because he and his sibling are histocompatible, the BMT can be done. Syngeneic marrow is from an identical twin. There is no such thing as a monoclonal BMT. Autologous refers to the individual's own marrow.

The nurse is planning care for a school-age child admitted to the hospital with hemophilia. Which interventions should the nurse plan to implement for this child? (Select all that apply.) a. Finger sticks for blood work instead of venipunctures b. Avoidance of IM injections c. Acetaminophen (Tylenol) for mild pain control d. Soft tooth brush for dental hygiene e. Administration of packed red blood cells

b. Avoidance of IM injections c. Acetaminophen (Tylenol) for mild pain control d. Soft tooth brush for dental hygiene

Chelation therapy is begun on a child with -thalassemia major. The purpose of this therapy is to: a. treat the disease. b. eliminate excess iron. c. decrease risk of hypoxia. d. manage nausea and vomiting.

b. eliminate excess iron. A complication of the frequent blood transfusions in thalassemia is iron overload. Chelation therapy with deferoxamine (an iron-chelating agent) is given with oral supplements of vitamin C to increase iron excretion. Chelation therapy treats the side effect of the disease management. Decreasing the risk of hypoxia and managing nausea and vomiting are not the purposes of chelation therapy.

Which is caused by a virus that primarily infects a specific subset of T lymphocytes, the CD4+ T cells? a. Wiskott-Aldrich syndrome b. Idiopathic thrombocytopenic purpura c. Acquired immunodeficiency syndrome (AIDS) d. Severe combined immunodeficiency disease

c. Acquired immunodeficiency syndrome (AIDS) AIDS is caused by the human immunodeficiency virus (HIV), which primarily attacks the CD4+ T cells. Wiskott-Aldrich syndrome, idiopathic thrombocytopenic purpura, and severe combined immunodeficiency disease are not viral illnesses.

The nurse is planning care for a school-age child admitted to the hospital with hemophilia. Which interventions should the nurse plan to implement for this child? (Select all that apply.) a. Finger sticks for blood work instead of venipunctures b. Avoidance of IM injections c. Acetaminophen (Tylenol) for mild pain control d. Soft tooth brush for dental hygiene e. Administration of packed red blood cells

b. Avoidance of IM injections c. Acetaminophen (Tylenol) for mild pain control d. Soft tooth brush for dental hygiene ANS: B, C, D Nurses should take special precautions when caring for a child with hemophilia to prevent the use of procedures that may cause bleeding, such as IM injections. The subcutaneous route is substituted for IM injections whenever possible. Venipunctures for blood samples are usually preferred for these children. There is usually less bleeding after the venipuncture than after finger or heel punctures. Neither aspirin nor any aspirin-containing compound should be used. Acetaminophen is a suitable aspirin substitute, especially for controlling mild pain. A soft toothbrush is recommended for dental hygiene to prevent bleeding from the gums. Packed red blood cells are not administered. The primary therapy for hemophilia is replacement of the missing clotting factor. The products available are factor VIII concentrates.

A boy with leukemia screams whenever he needs to be turned or moved. Which is the most probable cause of this pain? a. Edema b. Bone involvement c. Petechial hemorrhages d. Changes within the muscles

b. Bone involvement The invasion of the bone marrow with leukemic cells gradually causes a weakening of the bone and a tendency toward fractures. As leukemic cells invade the periosteum, increasing pressure causes severe pain. Edema, petechial hemorrhages, and changes within the muscles would not cause severe pain.

Which should the nurse teach about prevention of sickle cell crises to parents of a preschool child with sickle cell disease? (Select all that apply.) a. Limit fluids at bedtime. b. Notify the health care provider if a fever of 38.5° C (101.3° F) or greater occurs. c. Give penicillin as prescribed. d. Use ice packs to decrease the discomfort of vasoocclusive pain in the legs. e. Notify the health care provider if your child begins to develop symptoms of a cold.

b. Notify the health care provider if a fever of 38.5° C (101.3° F) or greater occurs. c. Give penicillin as prescribed. e. Notify the health care provider if your child begins to develop symptoms of a cold. ANS: B, C, E The most important issues to teach the family of a child with sickle cell anemia are to (1) seek early intervention for problems, such as a fever of 38.5° C (101.3° F) or greater; (2) give penicillin as ordered; (3) recognize signs and symptoms of splenic sequestration, as well as respiratory problems that can lead to hypoxia; and (4) treat the child normally. The nurse emphasizes the importance of adequate hydration to prevent sickling and to delay the adhesion-stasis-thrombosis-ischemia cycle. It is not sufficient to advise parents to "force fluids" or "encourage drinking." They need specific instructions on how many daily glasses or bottles of fluid are required. Many foods are also a source of fluid, particularly soups, flavored ice pops, ice cream, sherbet, gelatin, and puddings. Increased fluids combined with impaired kidney function result in the problem of enuresis. Parents who are unaware of this fact frequently use the usual measures to discourage bedwetting, such as limiting fluids at night. Enuresis is treated as a complication of the disease, such as joint pain or some other symptom, to alleviate parental pressure on the child. Ice should not be used during a vasoocclusive pain crisis because it vasoconstricts and impairs circulation even more.

Parents of a school-age child with hemophilia ask the nurse, "Which sports are recommended for children with hemophilia?" Which sports should the nurse recommend? (Select all that apply.) a. Soccer b. Swimming c. Basketball d. Golf e. Bowling

b. Swimming d. Golf e. Bowling ANS: B, D, E Because almost all persons with hemophilia are boys, the physical limitations in regard to active sports may be a difficult adjustment, and activity restrictions must be tempered with sensitivity to the child's emotional and physical needs. Use of protective equipment, such as padding and helmets, is particularly important, and noncontact sports, especially swimming, walking, jogging, tennis, golf, fishing, and bowling, are encouraged. Contact sport such as soccer and basketball are not recommended.

A school-age child is admitted in vasoocclusive sickle cell crisis. The child's care should include: a. correction of acidosis. b. adequate hydration and pain management. c. pain management and administration of heparin. d. adequate oxygenation and replacement of factor VIII.

b. adequate hydration and pain management.

A school-age child is admitted in vasoocclusive sickle cell crisis. The child's care should include: a. correction of acidosis. b. adequate hydration and pain management. c. pain management and administration of heparin. d. adequate oxygenation and replacement of factor VIII.

b. adequate hydration and pain management. The management of crises includes adequate hydration, minimization of energy expenditures, pain management, electrolyte replacement, and blood component therapy if indicated. Hydration and pain control are two of the major goals of therapy. The acidosis will be corrected as the crisis is treated. Heparin and factor VIII is not indicated in the treatment of vasoocclusive sickle cell crisis. Oxygen may prevent further sickling, but it is not effective in reversing sickling because it cannot reach the clogged blood vessels.

Which should the nurse include when teaching the mother of a 9-month-old infant about administering liquid iron preparations? a. They should be given with meals. b. They should be stopped immediately if nausea and vomiting occur. c. Adequate dosage will turn the stools a tarry green color. d. Allow preparation to mix with saliva and bathe the teeth before swallowing.

c. Adequate dosage will turn the stools a tarry green color. The nurse should prepare the mother for the anticipated change in the child's stools. If the iron dose is adequate, the stools will become a tarry green color. The lack of the color change may indicate insufficient iron. The iron should be given in two divided doses between meals when the presence of free hydrochloric acid is greatest. Iron is absorbed best in an acidic environment. Vomiting and diarrhea may occur with iron administration. If these occur, the iron should be given with meals, and the dosage reduced, then gradually increased as the child develops tolerance. Liquid preparations of iron stain the teeth. They should be administered through a straw and the mouth rinsed after administration.

The nurse is teaching parents of an infant about the causes of iron deficiency anemia. Which statement best describes iron deficiency anemia in infants? a. It is caused by depression of the hematopoietic system. b. It is easily diagnosed because of an infant's emaciated appearance. c. Clinical manifestations are similar regardless of the cause of the anemia. d. Clinical manifestations result from a decreased intake of milk and the premature addition of solid foods.

c. Clinical manifestations are similar regardless of the cause of the anemia. In iron deficiency anemia, the child's clinical appearance is a result of the anemia, not the underlying cause. Usually the hematopoietic system is not depressed in iron deficiency anemia. The bone marrow produces red cells that are smaller and contain less hemoglobin than normal red cells. Children who are iron deficient from drinking excessive quantities of milk are usually pale and overweight. They are receiving sufficient calories, but are deficient in essential nutrients. The clinical manifestations result from decreased intake of iron-fortified solid foods and an excessive intake of milk.

A newly married couple is seeking genetic counseling because they are both carriers of the sickle cell trait. How can the nurse best explain the children's risk of inheriting this disease? a. Every fourth child will have the disease; two others will be carriers. b. All of their children will be carriers, just as they are. c. Each child has a one in four chance of having the disease and a two in four chance of being a carrier. d. The risk levels of their children cannot be determined by this information.

c. Each child has a one in four chance of having the disease and a two in four chance of being a carrier.

An adolescent will receive a bone marrow transplant (BMT). The nurse should explain that the bone marrow will be administered by which route? a. Bone grafting b. Bone marrow injection c. IV infusion d. Intra-abdominal infusion

c. IV infusion Bone marrow from a donor is infused intravenously, and the transfused stem cells will repopulate the marrow. Because the stem cells migrate to the recipient's marrow when given intravenously, this is the method of administration.

An adolescent will receive a bone marrow transplant (BMT). The nurse should explain that the bone marrow will be administered by which route? a. Bone grafting b. Bone marrow injection c. IV infusion d. Intra-abdominal infusion

c. IV infusion Bone marrow from a donor is infused intravenously, and the transfused stem cells will repopulate the marrow. Because the stem cells migrate to the recipient's marrow when given intravenously, this is the method of administration.

The nurse is conducting a staff in-service on sickle cell anemia. Which describes the pathologic changes of sickle cell anemia? a. Sickle-shaped cells carry excess oxygen. b. Sickle-shaped cells decrease blood viscosity. c. Increased red blood cell destruction occurs. d. Decreased adhesion of sickle-shaped cells occurs.

c. Increased red blood cell destruction occurs. The clinical features of sickle cell anemia are primarily the result of increased red blood cell destruction and obstruction caused by the sickle-shaped red blood cells. Sickled red cells have decreased oxygen-carrying capacity and transform into the sickle shape in conditions of low oxygen tension. When the sickle cells change shape, they increase the viscosity in the area where they are involved in the microcirculation. Increased adhesion and entanglement of cells occurs.

The nurse is conducting a staff in-service on sickle cell anemia. Which describes the pathologic changes of sickle cell anemia? a. Sickle-shaped cells carry excess oxygen. b. Sickle-shaped cells decrease blood viscosity. c. Increased red blood cell destruction occurs. d. Decreased adhesion of sickle-shaped cells occurs.

c. Increased red blood cell destruction occurs. The clinical features of sickle cell anemia are primarily the result of increased red blood cell destruction and obstruction caused by the sickle-shaped red blood cells. Sickled red cells have decreased oxygen-carrying capacity and transform into the sickle shape in conditions of low oxygen tension. When the sickle cells change shape, they increase the viscosity in the area where they are involved in the microcirculation. Increased adhesion and entanglement of cells occurs.

The nurse is planning activity for a 4-year-old child with anemia. Which activity should the nurse plan for this child? a. Game of "hide and seek" in the children's outdoor play area b. Participation in dance activities in the playroom c. Puppet play in the child's room d. A walk down to the hospital lobby

c. Puppet play in the child's room Because the basic pathologic process in anemia is a decrease in oxygen-carrying capacity, an important nursing responsibility is to assess the child's energy level and minimize excess demands. The child's level of tolerance for activities of daily living and play is assessed, and adjustments are made to allow as much self-care as possible without undue exertion. Puppet play in the child's room would not be overly tiring. Hide and seek, dancing, and walking to the lobby would not conserve the anemic child's energy.

Which is most descriptive of the pathophysiology of leukemia? a. Increased blood viscosity occurs. b. Thrombocytopenia (excessive destruction of platelets) occurs. c. Unrestricted proliferation of immature white blood cells (WBCs) occurs. d. First stage of coagulation process is abnormally stimulated.

c. Unrestricted proliferation of immature white blood cells (WBCs) occurs.

A 3-year-old child with sickle cell disease is admitted to the hospital in sickle cell crisis with severe abdominal pain. Which type of crisis is the child most likely experiencing? a. Aplastic b. Hyperhemolytic c. Vaso-occlusive d. Splenic sequestration

c. Vaso-occlusive Vaso-occlusive crisis, or painful crisis, is caused by obstruction of blood flow by sickle cells, infarctions, and some degrees of vasospasm.

Parents of a hemophiliac child ask the nurse, "Can you describe hemophilia to us?" Which response by the nurse is descriptive of most cases of hemophilia? a. Autosomal dominant disorder causing deficiency in a factor involved in the blood-clotting reaction b. X-linked recessive inherited disorder causing deficiency of platelets and prolonged bleeding c. X-linked recessive inherited disorder in which a blood-clotting factor is deficient d. Y-linked recessive inherited disorder in which the red blood cells become moon-shaped

c. X-linked recessive inherited disorder in which a blood-clotting factor is deficient

Parents of a hemophiliac child ask the nurse, "Can you describe hemophilia to us?" Which response by the nurse is descriptive of most cases of hemophilia? a. Autosomal dominant disorder causing deficiency in a factor involved in the blood-clotting reaction b. X-linked recessive inherited disorder causing deficiency of platelets and prolonged bleeding c. X-linked recessive inherited disorder in which a blood-clotting factor is deficient d. Y-linked recessive inherited disorder in which the red blood cells become moon-shaped

c. X-linked recessive inherited disorder in which a blood-clotting factor is deficient ANS: C The inheritance pattern in 80% of all of the cases of hemophilia is X-linked recessive. The two most common forms of the disorder are factor VIII deficiency, hemophilia A or classic hemophilia; and factor IX deficiency, hemophilia B or Christmas disease. The inheritance pattern is X-linked recessive. The disorder involves coagulation factors, not platelets, and does not involve red cells or the Y chromosomes.

Iron dextran is ordered for a young child with severe iron deficiency anemia. Nursing considerations include to: a. administer with meals. b. administer between meals. c. inject deeply into a large muscle. d. massage injection site for 5 minutes after administration of drug.

c. inject deeply into a large muscle. Iron dextran is a parenteral form of iron. When administered intramuscularly, it must be injected into a large muscle. Iron dextran is for intramuscular or intravenous (IV) administration. The site should not be massaged to prevent leakage, potential irritation, and staining of the skin.

Several blood tests are ordered for a preschool child with severe anemia. The child is crying and upset because of memories of the venipuncture done at the clinic 2 days ago. The nurse should explain: a. venipuncture discomfort is very brief. b. only one venipuncture will be needed. c. topical application of local anesthetic can eliminate venipuncture pain. d. most blood tests on children require only a finger puncture because a small amount of blood is needed.

c. topical application of local anesthetic can eliminate venipuncture pain. Preschool children are concerned with both pain and the loss of blood. When preparing the child for venipuncture, the nurse will use a topical anesthetic to eliminate any pain. This is a traumatic experience for preschool children. They are concerned about their bodily integrity. A local anesthetic should be used, and a bandage should be applied to maintain bodily integrity. The nurse should not promise one attempt in case multiple attempts are required. Both finger punctures and venipunctures are traumatic for children. Both require preparation.

The nurse is reviewing first aid with a group of school nurses. Which statement made by a participant indicates a correct understanding of the information? a. "If a child loses a tooth due to injury, I should place the tooth in warm milk." b. "If a child has recurrent abdominal pain, I should send him or her back to class until the end of the day." c. "If a child has a chemical burn to the eye, I should irrigate the eye with normal saline." d. "If a child has a nosebleed, I should have the child sit up and lean forward."

d. "If a child has a nosebleed, I should have the child sit up and lean forward." If a child has a nosebleed, the child should lean forward, not lie down. A tooth should be placed in cold milk or saliva for transporting to a dentist. Recurrent abdominal pain is a physiologic problem and requires further evaluation. If a chemical burn occurs in the eye, the eye should be irrigated with water for 20 minutes.

Which child should the nurse document as being anemic? a. 7-year-old child with a hemoglobin of 11.5 g/dl b. 3-year-old child with a hemoglobin of 12 g/dl c. 14-year-old child with a hemoglobin of 10 g/dl d. 1-year-old child with a hemoglobin of 13 g/dl

d. 1-year-old child with a hemoglobin of 13 g/dl Anemia is a condition in which the number of red blood cells, or hemoglobin concentration, is reduced below the normal values for age. Anemia is defined as a hemoglobin level below 10 or 11 g/dl. The child with a hemoglobin of 10 g/dl would be considered anemic. The normal hemoglobin for a child after 2 years of age is 11.5 to 15.5 g/dl.

The nurse is conducting a staff in-service on childhood blood disorders. Which describes the pathology of idiopathic thrombocytopenic purpura? a. Bone marrow failure in which all elements are suppressed b. Deficiency in the production rate of globin chains c. Diffuse fibrin deposition in the microvasculature d. An excessive destruction of platelets

d. An excessive destruction of platelets

Which immunization should not be given to a child receiving chemotherapy for cancer? a. Tetanus vaccine b. Inactivated poliovirus vaccine c. Diphtheria, pertussis, tetanus (DPT) d. Measles, rubella, mumps

d. Measles, rubella, mumps The vaccine used for measles, mumps, and rubella is a live virus and can result in an overwhelming infection. Tetanus vaccine, inactivated poliovirus vaccine, and diphtheria, pertussis, tetanus (DPT) are not live virus vaccines.

The nurse is conducting a staff in-service on childhood blood disorders. Which describes the pathology of idiopathic thrombocytopenic purpura? a. Bone marrow failure in which all elements are suppressed b. Deficiency in the production rate of globin chains c. Diffuse fibrin deposition in the microvasculature d. An excessive destruction of platelets

d. An excessive destruction of platelets Idiopathic thrombocytopenic purpura is an acquired hemorrhagic disorder characterized by an excessive destruction of platelets, discolorations caused by petechiae beneath the skin, and a normal bone marrow. Aplastic anemia refers to a bone marrow-failure condition in which the formed elements of the blood are simultaneously depressed. Thalassemia major is a group of blood disorders characterized by deficiency in the production rate of specific hemoglobin globin chains. Disseminated intravascular coagulation is characterized by diffuse fibrin deposition in the microvasculature, consumption of coagulation factors, and endogenous generation of thrombin and plasma.

The child receiving a transfusion complains of back pain and itching. What is the best initial action by the nurse? a. Notify the charge nurse. b. Disconnect intravenous lines immediately. c. Give diphenhydramine (Benadryl). d. Clamp off blood and keep line open with normal saline.

d. Clamp off blood and keep line open with normal saline.

Which is the most effective pain-management approach for a child who is having a bone marrow aspiration? a. Relaxation techniques b. Administration of an opioid c. EMLA cream applied over site d. Conscious or unconscious sedation

d. Conscious or unconscious sedation

Which is the most effective pain-management approach for a child who is having a bone marrow aspiration? a. Relaxation techniques b. Administration of an opioid c. EMLA cream applied over site d. Conscious or unconscious sedation

d. Conscious or unconscious sedation Effective pharmacologic and nonpharmacologic measures should be used to minimize pain associated with procedures. For bone marrow aspiration, conscious or unconscious sedation should be used. Relaxation, opioids, and EMLA can be used to augment the conscious or unconscious sedation.

Which is a common clinical manifestation of Hodgkin disease? a. Petechiae b. Bone and joint pain c. Painful, enlarged lymph nodes d. Enlarged, firm, nontender lymph nodes

d. Enlarged, firm, nontender lymph nodes Asymptomatic, enlarged, cervical or supraclavicular lymphadenopathy is the most common presentation of Hodgkin disease. Petechiae are usually associated with leukemia. Bone and joint pain are not likely in Hodgkin disease. The enlarged nodes are rarely painful.

Which is a common clinical manifestation of Hodgkin disease? a. Petechiae b. Bone and joint pain c. Painful, enlarged lymph nodes d. Enlarged, firm, nontender lymph nodes

d. Enlarged, firm, nontender lymph nodes NON TENDER!NOT PAIN FUL

The nurse is preparing a child for possible alopecia from chemotherapy. Which should be included? a. Explain to child that hair usually regrows in 1 year. b. Advise child to expose head to sunlight to minimize alopecia. c. Explain to child that wearing a hat or scarf is preferable to wearing a wig. d. Explain to child that when hair regrows, it may have a slightly different color or texture.

d. Explain to child that when hair regrows, it may have a slightly different color or texture.

The nurse is preparing a child for possible alopecia from chemotherapy. Which should be included? a. Explain to child that hair usually regrows in 1 year. b. Advise child to expose head to sunlight to minimize alopecia. c. Explain to child that wearing a hat or scarf is preferable to wearing a wig. d. Explain to child that when hair regrows, it may have a slightly different color or texture.

d. Explain to child that when hair regrows, it may have a slightly different color or texture. Alopecia is a side effect of certain chemotherapeutic agents. When the hair regrows, it may be a different color or texture. The hair usually grows back within 3 to 6 months after cessation of treatment. The head should be protected from sunlight to avoid sunburn. Children should choose the head covering they prefer.

The nurse is recommending how to prevent iron deficiency anemia in a healthy, term, breast-fed infant. Which should be suggested? a. Iron (ferrous sulfate) drops after age 1 month b. Iron-fortified commercial formula by age 4 to 6 months c. Iron-fortified infant cereal by age 2 months d. Iron-fortified infant cereal by age 4 to 6 months

d. Iron-fortified infant cereal by age 4 to 6 months

The nurse is recommending how to prevent iron deficiency anemia in a healthy, term, breast-fed infant. Which should be suggested? a. Iron (ferrous sulfate) drops after age 1 month b. Iron-fortified commercial formula by age 4 to 6 months c. Iron-fortified infant cereal by age 2 months d. Iron-fortified infant cereal by age 4 to 6 months

d. Iron-fortified infant cereal by age 4 to 6 months Breast milk supplies inadequate iron for growth and development after age 5 months. Supplementation is necessary at this time. The mother can supplement the breastfeeding with iron-fortified infant cereal. Iron supplementation or the introduction of solid foods in a breast-fed baby is not indicated. Providing iron-fortified commercial formula by age 4 to 6 months should be done only if the mother is choosing to discontinue breastfeeding.

Which immunization should not be given to a child receiving chemotherapy for cancer? a. Tetanus vaccine b. Inactivated poliovirus vaccine c. Diphtheria, pertussis, tetanus (DPT) d. Measles, rubella, mumps

d. Measles, rubella, mumps

Which clinical manifestation should the nurse expect when a child with sickle cell anemia experiences an acute vasoocclusive crisis? a. Circulatory collapse b. Cardiomegaly, systolic murmurs c. Hepatomegaly, intrahepatic cholestasis d. Painful swelling of hands and feet; painful joints

d. Painful swelling of hands and feet; painful joints A vasoocclusive crisis is characterized by severe pain in the area of involvement. If in the extremities, painful swelling of the hands and feet is seen; if in the abdomen, severe pain resembles that of acute surgical abdomen; and if in the head, stroke and visual disturbances occur. Circulatory collapse results from sequestration crises. Cardiomegaly, systolic murmurs, hepatomegaly, and intrahepatic cholestasis result from chronic vasoocclusive phenomena.

Several complications can occur when a child receives a blood transfusion. Which is an immediate sign or symptom of an air embolus? a. Chills and shaking b. Nausea and vomiting c. Irregular heart rate d. Sudden difficulty in breathing

d. Sudden difficulty in breathing Signs of air embolism are sudden difficulty breathing, sharp pain in the chest, and apprehension. Air emboli should be avoided by carefully flushing all tubing of air before connecting to patient. Chills, shaking, nausea, and vomiting are associated with hemolytic reactions. Irregular heart rate is associated with electrolyte disturbances and hypothermia.

A child with leukemia is receiving triple intrathecal chemotherapy consisting of methotrexate, cytarabine, and hydrocortisone. The purpose of this is to prevent: a. infection. b. brain tumor. c. drug side effects. d. central nervous system (CNS) disease.

d. central nervous system (CNS) disease. For certain children, CNS prophylactic therapy is indicated. This drug regimen is used to prevent CNS leukemia and will not prevent infection or drug side effects. If the child has a brain tumor in addition to leukemia, additional therapy would be indicated.

An 8-year-old girl is receiving a blood transfusion when the nurse notes that she has developed precordial pain, dyspnea, distended neck veins, slight cyanosis, and a dry cough. These manifestations are most suggestive of: a. air emboli. b. allergic reaction. c. hemolytic reaction. d. circulatory overload.

d. circulatory overload.

An 8-year-old girl is receiving a blood transfusion when the nurse notes that she has developed precordial pain, dyspnea, distended neck veins, slight cyanosis, and a dry cough. These manifestations are most suggestive of: a. air emboli. b. allergic reaction. c. hemolytic reaction. d. circulatory overload.

d. circulatory overload. The signs of circulatory overload include distended neck veins, hypertension, crackles, dry cough, cyanosis, and precordial pain. Signs of air embolism are sudden difficulty breathing, sharp pain in the chest, and apprehension. Allergic reactions are manifested by urticaria, pruritus, flushing, asthmatic wheezing, and laryngeal edema. Hemolytic reactions are characterized by chills, shaking, fever, pain at infusion site, nausea, vomiting, tightness in chest, flank pain, red or black urine, and progressive signs of shock and renal failure.

Myelosuppression, associated with chemotherapeutic agents or some malignancies such as leukemia, can cause bleeding tendencies because of a(n): a. decrease in leukocytes. b. increase in lymphocytes. c. vitamin C deficiency. d. decrease in blood platelets.

d. decrease in blood platelets.

Myelosuppression, associated with chemotherapeutic agents or some malignancies such as leukemia, can cause bleeding tendencies because of a(n): a. decrease in leukocytes. b. increase in lymphocytes. c. vitamin C deficiency. d. decrease in blood platelets.

d. decrease in blood platelets. The decrease in blood platelets secondary to the myelosuppression of chemotherapy can cause an increase in bleeding. The child and family should be alerted to avoid risk of injury. Decrease in leukocytes, increase in lymphocytes, and vitamin C deficiency would not affect bleeding tendencies.


संबंधित स्टडी सेट्स

Project Management Ch 10/11 Extra practice

View Set

Management Quiz 1: Musculoskeletal Problems

View Set

Unit 1 Water, Thermoregulation, Plant Transport

View Set

Anatomy - Axial Skeleton Chapter 7

View Set

Comprehensive Stress Management- Chapter 1, Comprehensive Stress Management-Chapter 2, Comprehensive Strees Managment-Chapter3

View Set

The Spinal Cord and Spinal Nerves

View Set

Computing - Factors affecting CPU performance

View Set